The Ultimate Behavioural Sciences Guide (Psych x Soc)

¡Supera tus tareas y exámenes ahora con Quizwiz!

What refers to some people not having the ability to stop themselves from falling asleep, and genetics are strong indicator of this sleep disorder? (A) Narcolepsy (B) Sleep Apnea (C) Insomnia (D) Sleep debt

(A) Narcolepsy Narcolepsy refers to some people not having the ability to stop themselves from falling asleep, and genetics are strong indicator of this sleep disorder.

Robert writes in his dissertation that countries can be broken down into different types that are hierarchical in nature. Robert's Dissertation lends support to which Globalization Theory? (A) World Systems Theory (B) Modernization Theory (C) Dependency Theory (D) Butterfly Effect Theory

(A) World Systems Theory World Systems Theory divides the world into three types of countries.

Which subcategory of Schizophrenia symptoms involves issues with attention, organizing, or planning abilities? (A) None of the following (B) Cognitive (C) Negative (D) Positive

(B) Cognitive The Cognitive subcategory of Schizophrenia symptoms involves issues with attention, organizing and planning abilities.

Which of the following is not one of the three temperaments assigned to babies? (A) Difficult (B) Extrovert (C) Withdrawn (D) Easy

(B) Extrovert These temperaments can be assigned at very young ages, before environments could influence temperament.

As John looks at a tree in the distance, his eyes are relaxed. However, when he begins to look at a pencil on the desk in front of him, his eyes must turn towards the pencil. The relaxing and turning of John's eye muscles allow John to have sense of depth. Which binocular cue does this refer to? (A) Motion Parallax (B) Shading and Contour (C) Convergence (D) Retinal Disparity

(C) Convergence Eye convergence refers to how we use our eye muscles, relaxing them when looking at something far; turning them towards an object when looking at something close

To trigger the Fight or Flight response, the adrenal glands release catecholamines. Which of the following are catecholamines? I. Epinephrine II. Cortisol III. Norepinephrine (A) I only (B) II only (C) I and III only (D) I, II and III

(C) I and III only Epeniphrine and Norepinephrine (also called adrenaline and noradrenaline) are catecholamines.

Mary has a subconscious wish of kissing Brad Pitt's knee. Instead of acknowledging her strange desire, she unconsciously pushes it down into her unconscious. Which defense mechanism is Mary most likely implementing? (A) Regression (B) Suppression (C) Repression (D) Denial

(C) Repression Mary is pushing an undesirable wish down into her unconscious, which is exactly what happens in Repression.

Think about your high school graduation. What type of memory is that? (A) Procedural Memory (B) Semantic Memory (C) Priming (D) Episodic Memory

(D) Episodic Memory

How would you define norms?

*Norms* are defined as unwritten societal rules that define the boundaries of acceptable behaviour. They describe moral standards and ethical rules that individuals internalize, and thus serve as a means of social control.

The two functions of the thyroid gland are setting basal metabolic rate, and calcium homeostasis. What hormones are responsible for both?

BMR - triiodothyronine (T3) and thyroxine (T4) Calcium homeostasis - calcitonin

Albert Bandura's Bobo doll experiment is famous for coming up with the social cognitive theory of personality. What were his crucial ideas?

Bandura's concepts of *reciprocal determinism* and *locus of control* were crucial to the social cognitive perspective. Reciprocal determinism - idea that out thoughts, feelings, behaviour and environment all interact with each other to determine our actions in a given situation. Locus of control - level of control people feel they have over the environment, or the environment has over them.

True or False? Although Taboos are based on highly-cherished values, they are not punishable by law.

False. Because Taboos are based on highly-cherished values, they are often punishable by law.

Your Mom starts to call you every so often. After she talks to you, she will send you a $5 Amazon gift card immediately via email, which makes you happy. After a while you start feeling happy as soon as your mom calls. What is the Neutral Stimulus, Conditioned Stimulus, Unconditioned Stimulus, Conditioned Response and Unconditioned Response in this example?

In this example, your mom's phone call is the Neutral Stimulus. The Amazon gift card is the Unconditioned Stimulus, which elicits the Unconditioned Response of you feeling happy. Once you begin to associate gift cards with your mom calling, your mom calling becomes the Conditioned Stimulus, which elicits a Conditioned Response of you feeling happy. What a tricky mother you have!

Virat Kohli consistently hits the ball with the middle of the bat compared to how he was 4 years ago. Is this an example of proprioception or kinesthesia?

Kinesthesia - hand-eye coordination improvements and awareness + behavioural

Describe Piaget's pre-operational stage

Pre-operational stage: *Children ~2-7 years* - characterized by symbolic thinking (including ability to speak), *egocentrism* and *centration* (tendency to focus on only one aspect of a phenomenon). Concept of *pretend-play* develops.

Describe how Sensitization and Habituation may relate to trying to sleep in a new noisy neighborhood.

Sensitization (or dehabituation) would entail you having a harder and harder time trying to sleep because the noises start bothering you more and more. Habituation would entail you having and easier and easier time trying to sleep because the noises begin to blend into the background and affect you less and less over time.

Describe the term and what kind of cue it is: Shading, lighting and contours

Shading/contours - monocular cue - recognition of shading and lighting to attain depth information

Suppose you want to teach a kid to do a handstand. Describe shaping and how it could be useful in doing so.

Shaping is successively, gradually reinforcing behaviours that approximate (/come close to) the target behaviour. The target behaviour = headstand. You would reinforce the steps, so first you'd reinforce showing up to the class. Then, you'd reinforce the position with hands on the mat, and stop reinforcing the previous behaviour. Then, you'd reinforce the position with forearms on the mat, and stop reinforcing the hands on mat step. Finally, you'd reinforce lifting up the legs until the target behaviour is performed.

Jenny ends up deciding on a flavor of ice cream. Do you expert her to feel more or less satisfied about her choice than someone choosing from 5 flavors of ice cream?

She would be expected to feel less satisfied. Those who experience Tyranny of Choice are expected to feel less satisfied about their final choice.

Describe social constructionism and examples of social constructs

Social constructionism - focuses on how people form and put together social constructs and realities - constructs arise from communication and agreement on significance of certain concepts - can be intangible (honor, justice) or even physical objects (money - paper and coinage money have value because we give it value). Examples include money, nations, even gender

Sound waves hit the outer ear - how do they reach the inner ear?

Sound waves hit the pinna and travel down the external auditory meatus (auditory canal) and hit the ear drum (tympanic membrane), causing it to vibrate. This causes the 3 bone structure (malleus, incus, stapes) to vibrate back and forth. The stapes is attached to a the oval window, a membrane-covered opening that leads into the inner ear.

True or false: According to Selye's theory of stress and general adaptive syndrome, when you see something as having both good and bad consequences, it's called an approach-avoidance conflict, and both physical and social stressors can cause conflict.

TRUE!

True or false: The biological and trait perspectives are closely linked. In twin studies with identical twins raised separately, one of the supporting findings for these theories was that traits such as social potency and traditionalism were common in both twins.

TRUE: The biological and trait perspectives are closely linked. In twin studies with identical twins raised separately, one of the supporting findings for these theories was that traits such as social potency and traditionalism were common in both twins. social potency = degree to which a person assumes leadership in mastery roles in social situations traditionalism = tendency to follow authority

True or false: the PEN model by Eysenck was expanded to the *Big Five*, which describes majority of people using the dimensions of five traits: openness, conscientiousness, extraversion, agreeableness and neuroticism

TRUE: the PEN model by Eysenck was expanded to the *Big Five*, which describes majority of people using the dimensions of five traits: openness, conscientiousness, extraversion, agreeableness and neuroticism acronym - OCEAN

According to Freud, what characterizes the phallic/oedipal stage of his theory?

The *phallic (Oedipal) stage* (3-5yo) centers on the resolution of the *Oedipus complex* (male children) or the *Electra conflict* (females). Libidinal energy is focused on the genitals, and in Freud's view, the male child envies his father for intimate relationship with the mother. The child develops guilty feelings of wanting to eliminate the father and a supposed fear of castration, and deals with these by identifying with his father, establishing sexual identity and internalizing moral values. Results in *sublimating* of libidinal energy, which leads to the latency stage. For females, there is no fear of castration but a supposed *penis envy*

What does the problem solving theory of dreams state?

The *problem-solving theory of dreams* states that dreams are a way to solve problems while you are sleeping - untethered by real world rules, and allow interpretation of obstacles differently than when alert and awake.

True or False? Psychoactive drugs have the ability to alter our consciousness, perceptions, increase our mood, calm us down, and make us feel more alert.

True

True or false: PTH increases gut absorption of Ca by producing Vitamin D in the kidneys, which helps reabsorption of calcium in small intestine

True. PTH increases gut absorption of Ca by producing Vitamin D in the kidneys, which helps reabsorption of calcium in small intestine

In an autopsy of someone who committed suicide, fewer than normal serotonin and norepinephrine receptors have been found in tissue staining. Is this characteristic of clinical depression?

Yes, studies have shown fewer than average NE and serotonin receptors in patients with depression.

How does a dermatome encode a location of stimulus?

Each *dermatome* is an area of the skin supplied by mainly a single spinal nerve - whichever dermatome is firing indicates the position.

What is encoding?

Encoding refers to the process of putting new info into memory.

Define functional fixedness

Functional fixedness: The inability to consider how to use an object in a nontraditional manner - not thinking outside the box

Jimmy is a wealthy land owner and has many slaves. How might Jimmy use the Just World Hypothesis to justify his position (Social Class)?

He might say that the slaves got what they deserved and he did too, which is why he is wealthy.

Describe illness anxiety disorder

Illness anxiety disorder is characterized by being consumed with thoughts about having or developing a serious medical condition - individuals may be quick to become alarmed about health and excessively check selves for signs of illness

Sam is goes to a car dealership to buy a car, and while he is impressed with the cars and the prices, he does not like the way the salesman is dressed or the way he talks to Sam. Which attitude theory best explains Sam's thought process? (A) Theory of Planned Behavior (B) Elaboration Likelihood Model (C) Attitude-to-behavior Process Model (D) Prototype Willingness Model (PWM)

(B) Elaboration Likelihood Model Sam is persuaded by quality (Central Route Processing) but distracted and put off by the superficial cues (Peripheral Route Processing). This is closely related to the Elaboration Likelihood Model.

______________ taste buds are folded structures located primarily in the lateral (side) portions of the tongue. (A) Fungiform (B) Foliate (C) Circumvallate (D) Filiform

(B) Foliate Foliate tasted buds are folded structures located primarily on the lateral (side) portions of the tongue.

Where in the body is the 30C squalene converted to Cholesterol? (A) Kidney (B) Adrenal Gland (C) Pancreas (D) Liver

(D) Liver Squalene is converted to cholesterol in the Liver.

In the menstrual cycle, estrogen causes the release of GnRH, which will trigger LH release, which will increase estrogen production. Is this an example of positive or negative feedback?

Because this cycle will increase the production of the primary messenger, it is a positive feedback loop.

Define culture

Culture is a way of life shared by a group of people - refers to knowledge, beliefs and values.

John's girlfriend cheats on him, yet when he finds out he is surprisingly mellow and not angry the way he was expected to be. What sort of impairment in the limbic system would explain this?

Damage to the amygdala, which controls anger/aggression and fear/anxiety emotions. Destruction of the amygdala results in a mellow attitude.

Which of the anterior pituitary hormones are direct? tropic?

Direct hormones - Prolactin, GH, endorphins Tropic - TSH (T3 and T4), ACTH, FSH and LH

True or False? Maslow believed that most people will eventually reach self-actualization.

False. Maslow believed only about 1% of people will reach self-actualization. Maslow also stated that people who self-actualize are more likely to have *peak experiences*: profound and deeply moving experiences that have important and lasting effects

True or False. Milgram's study is considered ethical as evidenced by its many replications across cultures.

False. Milgram's study is considered unethical despite its many replications across cultures.

What is sensory adaptation?

The change in the sensitivity of your perception of a sensation Senses of hearing, touch, smell, even proprioception (sense of balance) all adapt to changes in perception by desensitizing

When talking about biases, what is the disconfirmation principle?

The disconfirmation principle states when a potential solution fails during testing it should be discarded - solution does not work. Confirmation bias may prevent one from doing this.

What is the endocrine system?

The endocrine system is a system of organs called glands which release chemicals called hormones into the bloodstream to initiate an effect.

True or false: Endorphines decrease the perception of pain - mask it and can even induce sense of euphoria

True

What refers to the persistent difficulty in falling asleep or staying asleep? (A) Narcolepsy (B) Sleep Apnea (C) Insomnia (D) Sleep debt

(C) Insomnia. Insomnia is difficulty falling asleep or staying asleep, and is related to anxiety and depression.

After a 2 year old still has not started speaking, worried parents go to see a specialist. The specialist suggests that they start bringing their child to a popular park. Which language development theory would this specialist most agree with? (A) Nativist approach (B) Learning reinforcement (C) Interactionist approach (D) Universal approach

(C) Interactionist approach This therapist would support the interactionist approach because his recommended course of action is to increase the child's interaction with other children. Social interactionist approach - social interactions drive language development.

Which of the following examples best represents cognitive dissonance? (A) Jessica spends lots of time alone with her boyfriend instead of socializing, and would rather spend time with him than other friends. (B) Raven telling others that her gastroparesis means she cannot eat meat, but she eats hamburgers. (C) Jason knows Paul is manipulating all their mutual friends, but convinces himself that Paul wouldn't manipulate him. (D) Matt believes everything Raven says because he is blinded by love.

(C) Jason knows Paul is manipulating all their mutual friends, but convinces himself that Paul wouldn't manipulate him. (A) shows no dissonance (B) is simply lying (D) is only one thought, so there cannot be dissonance

Which of the following would be most intriguing to a sociologist that specializes in Feminism? (A) Jill asks Jack to trade sandwiches at lunch time. (B) Sam tells Alyssa that he likes her. (C) Jenny receives a smaller allowance than her twin brother Dewey. (D) At recess, Mary taunts Harold, saying, "men go to Jupiter to get more Stupider."

(C) Jenny receives a smaller allowance than her twin brother Dewey. Feminists are interested in Gender inequalities. This example shows a distinct inequality in which a girl is treated differently than a boy in a way that would be seen as unfair.

Sarah is in an abusive relationship with her boyfriend. She tries to tell him to stop at first, but this just makes him even worse. She feels trapped in the relationship and gives up on trying to make things better, ending up extremely depressed. Which term best describes Sarah's situation? (A) External Locus of Control (B) Internal Locus of Control (C) Learned Helplessness (D) Tyranny of Choice

(C) Learned Helplessness Learned helplessness occurs when one feels helpless in She has learned that her actions are fruitless; thus, she has learned to feel and act helpless.

After olfactory cells synapse to their specific glomerulus, which structure, that projects to the brain, do the cells then travel to? (A) Olfactory epithelium (B) Cribriform Plate (C) Mitral/Tufted Cell (D) Amygdala

(C) Mitral/Tufted Cell

Researchers are trying to determine if musical tastes are genetic or environmental. Which of the following design setups would suggest that variance is almost completely caused by environments? (A) Monozygotic twins raised together (B) Dizygotic twins raised separately (C) Monozygotic twins raised separately (D) Dizygotic twins raised together

(C) Monozygotic Twins raised separately To limit variance caused by heredity, monozygotic twins are used. To amplify variance caused by environment, the twins are raised in separate households, like in an adoption study.

When describing major motor milestones in infants, which is the appropriate unit of time to judge on? (A) Days (B) Weeks (C) Months (D) Years

(C) Months For infants, major motor milestones are often talked about in terms of months because of how fast they develop!

Prenatal development begins with the process of ____________ at 3-4 weeks gestational stage - this is when the ectoderm overlying the notochord begins to furrow and forms a ______________ surrounded by two neural folds. (A) Neural groove, neurulation (B) Neurulation, forebrain (C) Neurulation, neural groove (D) Neural groove, basal plate

(C) Neurulation, neural groove Prenatal development begins with the process of neurulation at 3-4 weeks gestational stage - this is when the ectoderm overlying the notochord begins to furrow and forms a neural groove surrounded by two neural folds.

You see two motorcycles. One looks smaller than the other and you assume that it must be further away in the distance. What monocular cue is likely responsible for this? Explain. (A) Relative size (B) Interposition (C) Relative height (D) Shading and contour

(A) relative size Relative size tells us objects that are smaller are perceived as being further away.

Describe the two main types of strategies used in signal detection theory.

Conservative strategy - always say no unless 100% certain of a signal. Downside is you'll always get correct rejections, but may have some miss's. Liberal strategy - you'll always get the hits but then may also get some false alarms

Sally is touching a hot stove. She thinks that the action potential that sent the signal to her brain must be really big and strong. How would you respond to Sally's reasoning?

Inform her that action potentials are actually always the same size. The intensity of her pain is encoded by frequencies with which nerve cells fire, which sends many action potentials to her brain over a short time period, rather than one large action potential signal.

Describe the term and what kind of cue it is: shape constancy

Shape constancy - monocular cue - ability to recognize similar/same shapes even when in different orientation

Draw or visualize an example of The Law of Similarity. Describe this law.

The Law of Similarity - items or objects that are similar are grouped together.

What is the purpose of the anterior, posterior and lateral/horizontal canals (aka semicircular canals)? By what mechanism do they work?

The three ear canals are at right angles to each other and each are in line with one of the axes along our head (x, y or z). The canals are filled with a fluid called endolymph, and rotations of our head along particular axes causes the endolymph in the corresponding semicircular canal to shift - allows us to sense what plane out head is rotating along, as well as strength of the said rotation. Works towards our proprioception (sense of balance and position).

True or False? Constancy is a perceptual category in which our perception of an object remains the same even if the image cast on the retina is different.

True

How does the sociological use of the term Bureaucracy compare to our everyday use of the term?

The sociological use of the term Bureaucracy refers to the way an organization is organized (rules, structure, and ranking) whereas in our everyday life we only use the term to refer to overly rigid and structured organizations such as the DMV.

Parts of the eye: aqueous humor

water and liquid that fills the anterior chamber of the eye

Which of the following are not one of the three components of emotion? (A) Logical (B) Physiological (C) Cognitive (D) Behavioral

(A) Logical

Which of the following mental disorders does not fall under Cluster A, characterized as "odd" or "eccentric"? (A) Paranoid (B) Borderline (C) Schizotypal (D) Schizoid

(B) Borderline

Which of the following is not a sociocultural factor for eating habits? (A) Time of day (B) Insulin levels (C) Occasions (D) Appeal

(B) Insulin Levels Common sociocultural factors affecting eating habits include occasions, time, desire, appeal, and availability.

What is the usual order of sleep stages? (A) N1->N2->N3->REM (B) N1->N2->N3->N2->REM (C) N1->N3->N2->REM (D) N1->N2->N3->N2->N1->REM

(B) N1->N2->N3->N2->REM

Microscopic findings of Alzheimer's disease include: (A) Prions (B) Neurofibrillary tangles and β-amyloid plaques (C) Retroviruses in the brain (D) Foreign bacterium in the hippocampus

(B) Neurofibrillary tangles and β-amyloid plaques

Which of the following are direct hormones from the anterior pituitary gland? I. TSH II. Prolactin III. Growth Hormone (A) II only (B) I and III (C) II and III (D) I, II and III

(C) II and III Think of the mnemonic device "FLAT PEG": FLAT are the tropic hormones that stimulate endocrine glands: FSH, LH, ACTH, and TSH (FLAT). PEG are the direct hormones: Prolactin, endorphins, and growth hormone.

What is the point at which the optic nerve from each eye converges? (A) Blind Spot (B) Retina (C) Optic Chiasm (D) Visual Field

(C) Optic chiasm

Which of the following terms refers to when a recent exposure to a stimulus will unconsciously influence a response to a later stimulus? (A) procedural memory (B) semantic memory (C) priming (D) episodic memory

(C) priming

Which of the following are not a type of Cone? (A) Red (B) Blue (C) Yellow (D) Green

(C) yellow

After a severe biking accident, Lamelo suffers from anterograde amnesia (unable to form new memories). Lamelo can still access old memories. What area has Lamelo likely damaged? (A) Amygdala (B) Thalamus (C) Hypothalamus (D) Hippocampus

(D) Hippocampus The Hippocampus is in charge of converting short-term memories into long-term memories. A damaged hippocampus does not impair the ability to access long term memories

Andrew, was a "PROTAGONIST (ENFJ-A)" on the MBTI - Which of the following are a key part of my personality according to this? I. Thinking II. Judging III. Intuitive (A) I Only (B) II Only (C) I and II Only (D) II and III Only

(D) II and III Only

We know what confirmation bias is. What is hindsight bias?

*Hindsight bias* - "I knew it all along" effect; creeping determinism

What is place theory?

*Place theory* is a theory of hearing which states that our perception of sound depends on where each component frequency produces vibrations along the basilar membrane

Notice how sometimes in group projects, one member may do all of the work while the others contribute very little. What social phenomenon explains this?

*Social loafing* is the tendency of individuals to put in less effort in a group setting than individually.

Sally has a bad attitude towards spiders. Describe this by each of the three components of attitude

Affective - Sally is scared of spiders (fear being the emotion) Behaviour - because of her fear, Sally will avoid spiders and going near them (action) Cognitive - Sally believes spiders are dangerous. All three contribute to form Sally's negative attitude towards spiders.

According to Freud, what characterizes the latency stage of his theory?

After the libidinal energy of the oedipal stage is sublimated, the latency stage begins where there is no real focus of the libido, and this lasts until puberty.

Define Alertness

Alertness is a state of consciousness where we are awake and able to think cognitively, properly - process, perceive, access information and express information verbally.

A brain injury that damages or disrupts neurological connections in the prefrontal cortex results in a coma. What does this tell you about the prefrontal cortex and the state of alertness?

Alertness is maintained by neurological circuits in the prefrontal cortex - fibers from the prefrontal cortex communicate with a neural structure in the brainstem known as the reticular formation to keep the cortex alert and awake.

When graphing Signal detection theory, D represents the distance between the peaks of the noise curve and the signal curve. As D decreases, what happens to the frequency of misses/false alarms?

As D decreases, the range of signals and thresholds also decreases, and false alarms and misses would become more common.

In terms of homeostasis and the body's ability to adapt, why do regular drug users need a higher dose of drugs over time?

As the body adapts to drug use, it begins to prepare itself before the drug user takes the drug. Taking external and environmental cues, the brain begins to prepare the body and preemptively lowers heart rate, etc. - thus, the same dose of drugs over time is not as effective and drug users require more of the drug to achieve the same high.

Is the explanation of your computer screen a brute or institutional fact? What about quarks and subatomic particles?

Computer screen explanation is an institutional fact, as it is dependent on the movement of subatomic particles and metals. Because they do gown to quarks as we know right now, explanation of quarks would be brute facts in this case.

Substance use can lead to problems such as substance induced disorders. What falls under this category?

Conditions caused by substances that can range from substance induced mood disorders (mania, depression), disorders relating to anxiety, sleep, sexual function, even psychosis

Cultural differences include everything from typical jobs, to diet, to time to eat lunch, etc. When traveling outside one's home country to another, these differences can seem dramatic and overwhelming - what is this phenomenon referred to as?

Culture shock!

Compare: - Deductive reasoning - Inductive reasoning

Deductive reasoning (top-down) - starts from a set of general rules and draws conclusions from info given Inductive reasoning (bottom up) - create a theory via generalizations - starts with specific instances and draws conclusions from them

Compare delirium versus dementia.

Delirium is for reversible episodes of neurocognitive malfunctions due to drugs, improper blood concentrations, or even infection. Dementia is a major neurocognitive disorder that can lead to loss of functions

Delusional thought in schizophrenia can be described in categories: delusion of reference, delusion of persecution, and delusion of grandeur. Describe these three

Delusion of reference - belief that common environmental elements are directed toward individual (TV character talking directly to patient) Delusion of persecution - belief that the person is being deliberately targetted, interfered with, plotted against, discriminated against or threatened. Delusion of grandeur - belief that the person is remarkable in some significant way

Hormones can also be classified by target tissue. Differentiate between *direct hormones* and *tropic hormones*

Direct hormones - secreted and act directly on target tissue - e.g. insulin released by pancrease induces glucose uptake by muscles. Tropic (Indirect) hormones - require intermediary action - e.g. GnRH stimulates LH and FSH

Stressors are classified as causing either distress or eustress - differentiate between them

Distress occurs when experiencing unpleasant stressors. Eustress is the result of positive conditions.

How does a taste bud distinguish between different tastes?

Each taste bud has all the different types of cells specialized to each of the tastes. They all have dedicated axons that lead to the brain and remain separate all the way, and synapse at different parts of the brain. (This is known as the *labelled lines model*)

The pancreas has exocrine and endocrine functions. What is its exocrine function?

Exocrine tissues in pancrease secrete substances directly into ducts - numerous digestive enzymes

True or false: part of the hypothalamus is the suprachiasmatic nucleus which controls blood osmolarity.

FALSE. The *suprachiasmatic nucleus* IS INDEED part of the hypothalamus, but it *receives info from the retinae to help control sleep-wake cycles*.

True or false: most of the taste buds are concentrated on the posterior part of the tongue (back of the tongue)

False - most taste buds are concentrated on the anterior (front) of the tongue.

True or False? The faster a drug takes affect on the body, the less likely someone will be dependent on the drug.

False. The faster a drug takes affect on the body, the MORE likely someone will be dependent on the drug.

True or false: The hypophyseal portal system is used by the hypothalamus to communicate with the posterior pituitary gland.

False. The hypophyseal portal system is used by the hypothalamus to communicate with the ANTERIOR pituitary gland.

Differentiate between Freud's and Jung's idea of libido

Freud believed libido was physic energy rooted in sexuality; Jung believed libido was psychic energy in general.

Libido is the natural energy source that fuels the mechanisms of the mind. When Libido becomes fixed at a certain stage of development, what does Freud propose occurs at this point?

Freud would propose that you would end up stuck in that stage of development. So if your libido was stuck at an oral stage of development, you would have oral traits and personalities such as being very talkative, maybe smoking, etc.

T3 and T4 are synthesized by iodination of AA tyrosine in the follicular cells of the thyroid. The numbers 3 and 4 correspond to the number of iodine atoms attached to each tyrosine. Increased levels of T3 and T4 leads to cellular respiration - what does this entail?

Greater amounts of T3 and T4 leads to greater amount of protein and fatty acid turnover by speeding of synthesis and degradation. High plasma levels of thyroid hormones leads to decreased TSH and TRH synthesis - negative feedback in excess of T3 and T4

How did Hans and Sybil Eysenck come up with their trait theory? (hint: PEN model)

Hans and Sybil Eysenck used factor analysis to group behaviours that typically occur together and assigned labels to those groups. They described three traits in the *PEN Model* - *Psychotism* (measure of social deviance and nonconformity); *Extraversion* (tolerance for social interaction + stimulation); and *Neuroticism* (measure of emotional arousal in stress) E.g. people who are more reserved and less outspoken tended to enjoy solitary activity and avoid stimulation.

Can a phobia be considered an anxiety disorder?

If the phobia is causing distress or disability, then yes. It will be focused anxiety presenting with a specific pattern.

Why do we not realize how strange our dreams are when we are in REM sleep?

In REM sleep, the activity of the prefrontal cortex in the brain decreases. The prefrontal cortex is responsible for logical thinking, therefore our dreams in REM sleep can defy logic and not seem strange.

Compare the two types of Unintentional Discrimination: Side-effect Discrimination and Past-in-present Discrimination. Give an example of each.

In Side-effect Discrimination, discrimination in one setting leads to Unintentional Discrimination in another setting. For instance, a couple is discriminated against and unable to move into an apartment in a nice neighborhood and end up living in a poorer neighborhood, where they are unintentionally discriminated against by the government in that they received worse educations for their children in that area. In Past-in-present discrimination, discrimination in the past leads to Unintentional Discrimination in the present. For instance, an all-black hospital becomes a all-races hospital, yet the doctors continue to unintentionally treat the blacks differently than the whites who go there.

Compare the Information Processing Model of the nervous system to how a computer works.

In both the Information Processing Model and a computer, input is taken from the environment, processed, and outputs are produced.

Serotonin is a neurotransmitter that is responsible for satiation and content. What happens to Serotonin levels when Dopamine levels increase due to heightened euphoria and pleasure?

Increase in dopamine levels causes subsequent decline in serotonin levels. Because serotonin is responsible for feelings like satiation, it explains why drugs can be so addictive and cause dependence.

What is somatic symptom disorder?

Individuals with somatic symptom disorder have one or more somatic symptoms (which can be any symptom, may or may not be linked to a condition) that is accompanied by a disproportionately high level of concern about its seriousness, devotion of excess time and energy to it, or elevated levels of stress.

Compare the effects of inhibiting versus destroying the Amygdala.

Inhibiting the amygdala will simply result in someone entering a mellow state. Destroying the amygdala will cause hyperorality, hypersexuality and disinhibited behavior, as seen in Kluver-Bucy Syndrome.

You are trying to solve a difficult physics practice question. How might insight learning help you in this instance.

Insight learning is immediate, clear learning or understanding that takes place without overt trial-and-error testing - an insight! Maybe you will go take a shower, and then randomly the solution will come to you as an insight.

Cluster B (dramatic/emotional) is defined by four types of personality disorder. Describe narcissistic personality disorder

Narcissistic personality disorder - characterized by sense of grandoise and self importants, feelings of entitlement and constant need for admiration and attention. Ego-centric.

What does the Nature versus Nurture debate claim about motor development.

Nature: Since kids all around the world and blind kids all have similar timelines, it may not be learned from watching/nurture. Nurture: Allowing more space and time with parents to develop motor skills and practice can help.

What are the physiological effects of opiates/opioids? What are some examples?

Opiates can lower CNS activity, heart rate and BP - they are analgesics (pain reducing) E.gs - morphine, heroin, codeine,

What defines stress?

Our response to challenging events (physio-, cognitive or behavioural) defines stress

Describe the third step of the phototransduction cascade. Transducin binds to PDE, what happens next?

PDE is triggered to convert cGMP, necessary for Na+ channels, to GMP. There is hence a decline in [cGMP] and a rise in [GMP]

Cluster A (Odd and eccentric) is defined by which three types of personality disorder? (Important image)

Paranoid, schizotypal and schizoid.

Peptide hormones are charged while steroids aren't (derived from nonpolar molecules) - what does this entail about their water solubility and travel through the bloodstream?

Peptide hormones can travel through the bloodstream without carriers because they are water soluble. Steroid hormones are nonpolar and thus not soluble in water - they require protein carriers to travel through the bloodstream. They are generally inactive when attached and must dissociate from the carrier protein to function. Carrier proteins can be specific or nonspecific.

Which of the following reflexes newborns are born with are permanent reflexes? Neonatal reflexes? - Breathing reflex - Rooting refelx - Eye blink reflex - Pupillary reflex - Swimming Reflex - Moro Reflex (flailing arms out when startled - Galant reflex (when stroked on side, baby will roll onto that side - Swallowing reflex - Palmar grasp reflex -Stepping reflex

Permanent reflexes: Breathing, eye blink, pupillary, swallowing Neonata reflexes: rooting, swimming, moro, galant, palmar grasp, stepping

Each of the tropic hormones causes release of another hormone from an endocrine gland that has a negative feedback effect. What does this mean about receptors on the hypothalamus and pituitary?

Receptors for end hormones like cortisol must be present on the hypothalamus and pituitary for negative feedback to be possible. CRF from hypothalamus causes release of ACTH from pituitary gland - ACTH causes the adrenal cortex to release cortisol. When cortisol levels get too high, it must have negative feedback capabilities - therefore receptors for cortisol must be present on the hypothalamus and pituitary.

Describe the following retrieval cues: Serial position effect

Retrieval cue referring to how learning lists. Learners will have much higher recall for the first and last few items on a list than any other - tendency to remember early and late items is known as the *primacy* and *recency* effect (respectively)

Describe the sympathetic NS vs. parasympathetic NS in terms of: - Pupil dilation - Salivation - Breathing (respiratory) rate - Heart rate - Glucose release (blood sugar) - Adrenaline release - Digestion

Sympathetic NS: Pupils dilate; decreased salivation and digestion; increased breathing and heart rates, glucose release/blood sugar, and adrenaline/epinephrine release. Parasympathetic: Pupils constrict; increased salivation and digestion; decreased heart and breathing rates, adrenaline release and glucose release (or increased glucose storage)

Draw or visualize an example of The Law of Closure. Describe this law.

The Law of Closure - objects grouped together are seen as a whole. We tend to ignore gaps and complete contour lines

Draw or visualize an example of The Law of Continuity. Describe this law.

The Law of Continuity - objects that form a path are perceived as units; lines are seen as following the smoothest path.

Draw or visualize an example of The Law of Pragnanz. Describe this law.

The Law of Pragnanz states that reality is usually reduced to its simplest form. In this image, you see five circles instead of more complex potential shapes.

How might Cultural Universals relate to the Theory of Evolution?

The Theory of Evolutions states that physical traits that are selected for/are favorable to a species will become more common/prominent over time. It is the same with culture. Aspects of culture that are favored in a society will become more prevalent over time. Perhaps these Cultural Universals were selected for over time.

Describe how the education system functions as a social institution

The education system aims to arm the populaton with information, not only learning and cognitive skills but also the *hidden curriculum* of transmitting social norms, ethics, attitudes, beliefs to students. *Teacher expectancy* refers to the idea that teachers expect certain things from different students (e.g. males vs females), and that they tend to get what the expect from the students. They put students into categories based on perceived intrinsic abilities, as well as other things like gender, race, ethnicity, etc. creating inequalities between school districts.

How would the feminist perspective describe portrayal of information through mass media?

The feminist perspective would describe the mass media as reinforces dominant ideology on the basis of gender stereotypes and discrimination such as subordination of women, gender segregation in ads (pens, pens for her).

Where in the retina is the image at its sharpest?

The fovea, due to high concentration of cone cells.

What is the halo effect?

The halo effect is a cognitive bias where judgements about a specific aspect of a person can be influenced by overall impression of the individual - tendency for general impression to influence other, specific evaluations. The halo effect explains why people are often inaccurate when evaluating people they like or dislike a lot. Physical attractiveness is strongly linked to the halo effect.

How is the lack of a reflex in an infant clinically significant in a pediatrician checkup?

The lack of a reflex can signal that a certain area or group of muscles is not developing properly.

Why do some astronauts or even scuba divers feel 'disoriented' at times?

The lack of gravity in space, or the effect of buoyancy in deep waters, will cause the carbonate crystals in the otolithic system to not function properly, as gravity is an essential component in its shifting and subsequent pulling on hair cells to indicate orientation and position.

B. F. Skinner proposed that operant conditioning and reinforcement account for an infants ability of language development. Which theory does this refer to?

The learning (behaviouralist) theory of language development. Over time infants perceive sounds that are reinforced as having value, whereas those that aren't do not. Does not explain how children can learn new words or form sentences they haven't heard before

What is the macula? Fovea? Difference?

The macula is the name given to a avascular region in the posterior region of the eye. The fovea is a dimple-like area in the center of the macula, densely populated with cones, which allows detailed vision and in colour.

A man is playing a violent video game when a researcher uses a probe to activate the man's Amygdala. How will this influence the man's violent behavior?

The man is likely to become more violent as Amygdala activation is associated with anger/aggression.

Describe the Modernization theory of globalization

The modernization theory states that countries follow the same/similar path of going from a traditional country to a modernized one, and looks at internal social dynamics as countries adapt to tech changes and advancements, and the political + social changes that occur.

A common issue with flash photography is the "red eye effect". What part of the eye anatomy causes the red eye effect, and how?

The red eye effect phenomenon is due to light reflecting back off the retina, which is a reddish colour.

When it comes to behaviour influencing attitudes, what is role-playing phenomenon?

The role-playing phenomenon states that people alter their behaviour when in a new role to fit that role, which may eventually influence their attitudes and normalize that role. E.g. first time parents feel unusual at it initially but eventually develop new attitudes, "mature"; new professional behaviours in a new job, making you eventually having a new attitude

When the mother is present, how will a securely attached child typically behave compared to an insecurely attached child?

The securely attached child will feel safe to explore, knowing it has a Secure Base. The insecurely attached child, on the other hand, will stay close to its mother.

A child is unable to ride a bike alone but able to do so with the help of his father. His older brother, however, is well versed with how to ride a bike. What would Vygotsky's theory state about these two?

The skill of riding the bike is within the child's zone of proximal development, but not for the older brother, for whom the skill is fully developed.

What is top-down processing?

Top down processing uses your background knowledge to influence your perception e.g. in the following image, there are only circles with lines inside, but we can see a cube in it due to our previous knowledge. Even though there is no cube, we perceive one due to our background knowledge

Define the methods of problem solving: - Trial & Error - Algorithms - Heuristics

Trial and error - less sophisticated - various solutions to a problem are tried until one works Algorithms - formula or procedure for solving a certain type of problem - e.g. mathemetical or instructions set Heuristics are simplified principles to make decisions - rules of thumb - e.g. ismeans-end analysis (starts with the biggest problem -> smaller); working backwards (working way back from start point of a maze to find the beginning).

What is the crash?

When the body adjusts its homeostatic conditions with cues and the brain expects incoming drugs, but does not receive them, the time spent staying in preemptive preparation is the crash.

A major problem in recovering from drugs is the dependence that the nervous system can develop for outside sources of dopamine and/or serotonin. What happens in the neural synapses to cause this natural decrease in dopamine/serotonin production?

When the drug is taken initially, the synapses are flooded by excess dopamine/serotonin. With this overstimulation, the brain acts by shutting down some membrane receptors - can lower production of these neurotransmitters and/or the number of channels in the membrane, decreasing sensitivity.

According to the DSM-5, is Sexual Dysfunction considered a mental disorder?

Yes, Sexual Dysfunction is considered a mental disorder because it involves Distress and Disabilities from abnormalities or performance of sexual acts.

How does the Hypothesis of Relative Deprivation suggest about what causes frustration?

You become frustrated when what you actually get is less than what you expected to get/feel entitled to.

If a person strongly believes in the spotlight and resource models, what would their opinion be on multi-tasking?

That the ability to multi-task is ineffective, and attending to one set of stimuli is more effective.

A Population Pyramid can tell you what information?

A *Population Pyramid* can tell you the population of a community based on age and gender.

What is a glomerulus?

A glomerulus is a bundle of nerves that can be thought of as a destination for olfactory cells sensitive to the same molecule.

Parts of the eye: retina

A screen of neural elements (photoreceptors) and blood vessels in the back of the eye - converts photons into electrical signals, considered part of the CNS

In the image behind this flashcard, try to match each letter with the number provided: 1. Iris 2. Lens 3. Aqueous Humor 4. Vitreous Humor 5. Ciliary muscles 6. Optic nerve 7. Retina 8. Pupil 9. Fovea 10. Cornea

A-10 B-3 C-8 D-1 E-2 F-5 G-4 H-7 I-9 J- 6

Amino acid derivative hormones are less common than the other two but very important. What can be said about them, and what are examples?

AA-derivative hormones include important hormones such as epinephrine, norepinephrine, triiodothryonine, thyroxine. They are derived from one or two AA's with a few modifications. Chemistry is not predictable - catecholamines (epin/norepinephrine) bind to G protein coupled receptors, thyroid hormones bind intracellularly.

When one becomes addicted to a drug and suddenly goes through a period of not having the drug, what does the individual experience? A) Tolerance B) Withdrawal C) Intoxication D) Relapse

B) Withdrawal

Which Signal detection strategy corresponds to a C>1, and C<1?

C>1 = conservative C<1 = liberal C=0 is ideal

Describe Piaget's formal operational stage

Formal operational stage: *~12+ years* - characterized by the ability to engage in *abstract thinking and moral reasoning*

Sensation occurs when a physical stimuli is converted into a neural impulse. In terms of the human eye, what is the physical stimuli and what cells convert that into a neural impulse?

Light is the stimulus - it enters the eye and is converted to a neural impulse by photoreceptors (cones and rods)

Describe learned behaviours

Learned behaviours are not inherited and develop in response to environmental stimuli, usually for some adaptive advantage. They are learned, extrinsic, permutable (pattern/sequence that is changeable over time), adaptable (can be modified), progressive.

Parts of the eye: lens

Light hits the lens which is convex - the lens can change shape and get thinner or thicker

Whereas classical conditioning is concerned with instincts and biological responses, what is *operant conditioning*?

Operant conditioning links voluntary behaviour with consequences in an effort to alter the frequency of those behaviours. Op-conditioning is associated with BF Skinner, the father of *behaviourism* (theory believing all behaviours are conditioned).

Compare Race and Ethnicity.

Race is a physical characteristic that has been deemed to put you in one group versus another. It can be seen as socially-constructed. Examples include Black, White, Hispanic, etc. Ethnicity is the cultural group that you belong to. It may also relate to your country of origin. Examples include Jewish, Japanese-American, Irish, etc.

Describe the term and what kind of cue it is: relative height

Relative height - monocular cue - objects that are higher in the visual field are more distant

Describe how religion functions as a social institution

Religion is considered to be a pattern of social activities based on a set of beliefs and practices. *Religiosity* refers to how religious one considers him or herself to be: strength of beliefs, engagement in practices, attitudes about religion itself.

Describe the term and what kind of cue it is: retinal disparity

Retinal disparity/stereopsis - binocular cues - eyes separated by 2.5" (disparity); 2 eyes used to provide slightly different images on the retina

What is retrieval?

Retrieval is the process of demonstrating that something learned has been retained. Many think of it as recall.

Compare Assimilation and Accommodation, in terms of mental Schema.

Schemas can be defined as frameworks we create in order to organize and understand new information (e.g. schema for 4 legged animals). Piaget theorized new information is processed via adaptation, which has two complementary processes: assimilation and accomodation. Assimilation is the process of classifying new information into existing schemas. Accomodation is the process by which the EXISTING schemas are modified to incorporate new information.

Cluster A (Odd and eccentric) is defined by three types of personality disorder. Describe schizotypal personality disorder

Schizotypal personality disorder refers to a pattern of odd or eccentric thinking - individuals may have ideas of reference (similar to delusions of reference) as well as magical thinking (supersticious, belief in clairvoyance)

Differentiate between self-esteem and self-efficacy.

Self-esteem refers to our self worth and the respect we have for ourself. Self efficacy is the belief in our ability to succeed in a certain situation.

When eating spicy foods, people can feel pain as their TrpV1 receptors are activated. Why can capsacin and similar chemicals activate these receptors?

Spicy foods contain chemical molecules like capsacin which bind to TrpV1 receptors in your tongue, causing it to fire an action potential, which causes the brain to interpret it as pain and heat. Over time, the receptors become desensitized to capsacin.

Describe the *humanistic theory* (or phenomenological)

The humanistic theory takes a person-centered approach in stating individuals have free will and the ability to strive towards self-actualization (which remember, is max potential in Maslow's hierarchy!) The theory states we are inherently good and strive to self improve. Maslow was the first humanist.

The hypothalamus connects to the pituitary gland through a stream of blood vessels to release hormones that bind to receptors in the pituitary, triggering release of other hormones. What is the name of the blood vessel system?

The hypophyseal portal system. The hypothalamus sends tropic hormones through the hypophyseal portal system to the anterior pituitary gland, where they bind to receptors and signal release of other hormones.

A woman dreams about her boss getting eaten by a giant snake. What might be the manifest content in this example? latent content?

The manifest content is what she saw - her boss getting eaten by a giant snake. The latent content could be her suppressed feelings of anger toward her boss. Perhaps subconsciously she wants her boss to be out of her life.

Drug addiction is highly related to the mesolimbic reward pathway. What is this pathway and what does it include?

The reward pathway is a dopaminergic pathway that includes the ventral tegmental area (VTA) or the brain which releases dopamine in response to a pleasurable stimulus, to the nucleus accumbens (NAc), amygdala, prefrontal cortex and hippocampus. The pathway counts for motivation and emotion, and positive reinforcement of substance use. Stimuli can include food, sex, social interactions, drugs, gambling, even love.

When traveling to the Eiffel Tower for the first time, tourists may still get a feeling of deja-vu. Why is this the case in terms of bottom-up and top-down processing?

Tourists have probably seen pictures of the Eiffel Tower, so top-down processing will quickly recognize the structure before bottom-up processing can actually process the stimuli.

Parts of the eye: vitreous humor

Transparent gel in the eye behind the lens which supports the retina

True or false: dreaming is most likely to occur during REM sleep

True. REM sleep is the stage dreaming is most likely to occur in.

What would happen if the cribriform plate were completely solid (no holes)?

We would have impaired sense of smell, as the olfactory bulb would be unable to connect to the olfactory epithelium.

When is the phototransduction cascade triggered?

When light is present and hits rod cells and turns them off

A Harlow Monkey is removed from his mother and given two options for replacement mothers, a wire cage mother with a food dispenser and a cloth mother without a food dispenser. Which mother would you expect the monkey to spend most of its time with? What does this reveal?

You would expect the monkey to spend most of its time with the cloth mother. This reveals that comfort is a huge factor in forming secure attachments.

How might you use shaping to teach your dog to do a backflip?

You would reward your dog for each successive good step in the right direction. For instance, first reward him for getting up on two feet. Then reward him for falling backwards. Then reward him for the attempt at a flip, and finally reward him for the full, successful flip.

For audition to occur, you need ______________ (the stimulus) to be present and _____________ (receptor) to transduce these waves into signals.

pressurized sound waves, hair cells

What is the range of Frequencies in terms of Hz that humans can hear?

20-20000Hz

Which of the following are the two substages of the Conventional Stage? -Self-Interest (Individualism and Exchange) -Conformity and Interpersonal Accord -Social Contract -Obedience vs Punishment -Universal (Ethical) Principles -Authority and Social Order

3. Conformity and Interpersonal Accord ("Good Boy + Girl") 4. Authority and Social Order ("Law and Order")

What are pheromones?

Excreted or secreted chemicals that triggers an innate social response in members of the same species. Linked to mating, fighting, and communication.

Describe the following retrieval cues: Context effects

Context effects is when memory is aided by being in the physical location where encoding took place

The position component of our sense of proprioception is controlled by proprioception spindles in the muscles, what about our sense of balance?

Controlled by the vestibular system - semicircular canals in the inner ear.

A mouse is on a diving board that can sometimes administer electric shocks. The researcher shocks the diving board, and the mouse jumps in the water. Before the shock, however, the researcher made a beeping noise. Next time, when the researcher makes the beeping noise, the mouse jumps off the diving board before the researcher has time to apply the electric shock. How does this example relate to Avoidance Learning, Escape Learning, and Aversive Control. respectively?

Escape Learning applies to the mouse jumping off the bridge to escape the continuous electric shock. Avoidance Learning applies to the mouse jumping off when it hears the beep noise in order to avoid getting shocked. Both Escape and Avoidance Learning are examples of Aversive Control.

What is the function of GH (growth hormone)?

Exactly what the name says - promotes growth of muscle and bone. This requires energy and uses glucose - GH inhibits glucose uptake by other tissues and stimulates breakdown of fatty acids - increasing glucose availability for muscle and bone.

Feminist theory relates to Marxist conflict theory but states a conflict of gender rather than class. Describe feminist theory and how it applies today.

Feminist theory explains social inequalities on the basis of gender, and focuses on subordination of women through social + institutional discrimination - forms can be *gender roles* (expected roles of given sex/gender), sexuality, financial opportunity, social mobility. Though feminist waves helped diversify, the effects of gender discrimination are still seen today - differential expectations of behaviour, objectifying of women, less pay in workforce, *glass ceilings* for women (invisible top, difficulty to attain high ranks as easily) vs. *glass elevators* for men (easier to achieve high ranks due to gender).

The upper membrane of the Organ of Corti contains Cilia (Hair Cells) known as the ___________.

Hair bundle

Describe the signalling pathway for the parasympathetic NS

In the parasympathetic NS, a long preganglionic neuron begins either in the brain or spinal cord, and carries the electrical signal via a long axon to the synapse with a short postganglionic neuron - synapse uses acetylcholine. The signal is then carried to the effector organ, where the synapse also uses acetylcholine. Note: both synapses use acetylcholine to carry the signal on.

What mechanistically differentiates PTSD from other anxiety disorders?

PTSD can have a clear trigger that causes the onset of the disorder, while others have more ambiguous or trivial beginnings.

Describe the three elements of emotion/emotional response: cognitive, physio-, behavioural

Physiological response - when feeling is experiences, arousal is stimulated by ANS - physiological component includes changes in heart rate, breathing rate, skin temp, blood pressure. Behavioural response - includes facial expressions and body language - e.g. smiling, hand gestures Cognitive response - this is the subjective interpretation of the feeling being experiences - largely based on memories of past experiences and perception.

What happens when light hits a rod cell? Is it turned off or on?

Rod cells are normally turned on, in the absence of light. So when light hits a rod cell, it turns it off.

What is sensory memory?

Sensory memory is a temporary register of all the senses your body is taking in. The most studied are iconic memory (visual) and echoic memory (auditory)

What are the physiological effects of Stimulants? What are some examples?

Stimulants stimulate or intensity neural activity and bodily functions, i.e. increase CNS activity, heart rate and blood pressure - also causes one to be more awake and alert. E.gs amphetamines (like meth), caffeine, nicotene, MDMA/ecstasy, cocaine

A common effect of Alzheimer's is sundowning - what does this refer to?

Sundowning is the phenomenon of increase in dysfunction in the late afternoon and evening

True or false: self-efficacy can be depressed past the point of no recovery

True. Self-efficacy can be depressed to a point beyond recovery, leading to *learned helplessness*. This is a possible model for clinical depression.

Parts of the eye: iris

coloured part of the eye - two different muscles that contract and expand that control the size of the pupil - dilator pupillae (sympathetic) and constrictor pupillae (parasympathetic)

Which of the following is not one of the major categories of stressors? (A) Significant life change (B) Relationships (C) Catastrophe (D) Daily Hassle

(B) Relationships

Which structure in the ear allows the Brain to differentiate between 2 different sounds? (A) Tympanic Membrane (B) Stapes (C) Cochlea (D) Ossicles

(C) Cochlea

Differentiate between Anorexia and Bulimia nervosa

Both Anorexia and Bulimia are Feeding and Eating Disorders that lead to underweight patients. In Anorexia, the person consumes too little food consistently, whereas in bulimia, a person will binge eat and then purge it out (throw up the food).

Compare Schizotypal and Schizoid Personality Disorders.

Both Schizotypal and Schizoid Personality Disorders are part of the "Odd and Eccentric" Cluster. *Schizoid Personality Disorder* is characterized by emotional detachment, restricted emotional expression, little interactions and poor social skills. *Schizotypal personality disorder* will both avoid close relationships AND hold magical/odd beliefs, even may have ideas similar to delusions of self-reference.

Compare how Charles Cooley and George Herbert Mead thought that other people can influence us.

Charles Cooley seemed to think that anyone can influence us at any time while Mead had a more restrictive view, thinking that only certain people can influence us at certain time points..

Define cognitive dissonance

Cognitive dissonance is the discomfort felt when holding two or more conflicting cognitiosn (ideas, values, beliefs, etc.) - can lead to alterations in attitudes or behaviours, which are done in order to reduce that dissonance

Describe the term and what kind of cue it is: colour constancy

Colour constancy - monocular cue - ability to recognize when an object or thing is of the same colour throughout despite influence of shading

True or False? Impression Management is unrelated to the Backstage Self.

False. The Backstage is where you prepare for Impression Management (get dressed, put on makeup, practice singing, etc).

A group of closet racists and a group of closet feminists are placed in a room to discuss issues about racism for an hour. After the discussion, it was found that in the first group, members became overt and more racist than they were before, whereas the other group members became even less so. What explains this?

Group polarization - tendency for group ideas to shift more to an extreme than individual ideas of members within the group.

What are the 6 components of the universal emotions?

Happiness, sadness, surprise, anger, disgust, fear

Give an example of how Culture might shape Evolution?

If we all start doing desk jobs, different phsyical characteristics and strengths such as a strong back might be favored and become more prevalent over time through natural selection.

Define imitation

Imitation is a type of individual social influence - most basic form of social behaviour, when we copy someone else, and it comes from the understanding that there is a difference between ourselves and others.

Describe factitious disorder

Individuals who fake it because they 'want to be sick' - falsification of signs and symptoms

Describe the role of insulin in the body

Insulin is released by beta cells of the islets of Langerhans - antagonistic to glucagon - secreted at high [blood glucose] - induces glucose uptake by muscles and liver cells. Insulin stimulates anabolic processes such as protein and fat synthesis.

What are intentions in the theory of planned behaviour based off and formed by?

Intentions are based off (1) our attitudes towards a certain behaviour, (2) our subjective norms (what others/society thinks of our behaviour), and (3) perceived behaviour control (how hard it is to control our behaviour).

Jacob walked on the carpet with muddy shoes on. His Mom told him that before he could eat any dinner, he would need to clean the carpet. How might this situation relate to the Drive-Reduction Theory of Motivation?

Jacob's need for food is driving him to clean the carpet. Once he finishes cleaning the carpet and eats his dinner, his need is fulfilled and his drive is reduced.

Jessica believes that she is good at playing basketball, but deep down she feels a sense of guilt about who she is. She doesn't like herself. How does Jessica's situation relate to Self-esteem and Self-efficacy?

Jessica believes that she is good at playing basketball, but deep down she feels a sense of guilt about who she is. She doesn't like herself. How does Jessica's situation relate to Self-esteem and Self-efficacy?

Jim sees an image of his friend morphed with his own image. Will Jim like this picture more or less than a normal picture of his friend? Why?

Jim will like the morphed picture more because we tend to like/be attracted to things that are like/similar to ourselves.

How does the Just World Hypothesis relate to Attribution Theory?

Just World Hypothesis would lead people to make internal attributions when evaluating situations.

Bipolar disorders are a type of mood disorder characterized by episodes of both depression and mania. Describe manic episodes

Manic episodes are characterized by abnormal and persistently elevated moods lasting a week or longer - consist of at least three of the following: increased distractibility, decreased need for sleep, high self esteem, racing thoughts, increase goal-driven activity, increased talkativeness, and potentially involvement in high-rish behaviour.

How might Media, Society, Family, and Oneself respectively influence the spreading of a Stigma that women who are overweight are unintelligent?

Media is a major source of stigma - may portray women that are overweight as being brainless in various situations. Society may believe this and avoid giving intellectual jobs to women that are overweight. One's Family may tell a girl that she needs to lose weight or study harder. Oneself may believe the Stigma placed upon them, resulting in a Self-fulfilling Prophecy.

What is memory decay?

Memory loss which is natural and not due to disorders - memories not often used or stimulated are often lost naturally over time as the neurochemical trace of a short term memory fades.

In the frame of the nature versus nurture debate, explain the significance of monozygotic and dizygotic twin studies and adoption studies.

Monozygotic twin studies have 100% of the same genes and almost 100% similar environments. Dizygotic twins have only 50% gene sharing but still almost 100% environment. In an adoption study, the monozygotic twins still share 100% of genes, but very different environments. A combination of these studies allows studying effects of only genetic differences or only environment changes.

What kind of light do rods detect?

Only light and dark - they have one pigment, which is rhodopsin - low sensitivity to detail but permits night vision

While referred to as the same class of psychoactive drugs, what is the small distinction between Opiates and Opioids?

Opiates are used to describe the drugs found naturally, while opioids are those made synthetically.

Ethology refers to the scientific and objective study of behaviour, usually in an adaptive sense. What are 'overt' behaviours?

Overt behaviours simply refer to observable behaviour. It can be broken down to innate, learned and complex behaviours.

One form of response to stress is to "tend and befriend" for things like pair bonding - what is the hormone released and responsible for this? Why is it seen more in females?

Oxytocin It is more common in women because it is more linked to estrogen.

Patients in the past were expected to play the *sick role* whereas now things have changed a little - elaborate?

Patients used to be expected to play the *sick role* which entails they are not responsible for the illness and exempt from normal societal roles. NOW - patients are expected to be more responsible via diet, exercise, seeking help early on etc. to avoid health problems better.

Compare the Pegword and Method of Loci mnemonic devices.

Pegword devices are number and verbally based (match numbers with rhyming words, "one is a gun"), whereas Method of Loci is based on location (putting items in different rooms of your house or something like that).

Describe the Solomon Asch Conformity Study.

People are asked to identify which line (A, B, or C) is equal to the line of interest. Only one person is actually a participant, and they are found to go along with the group even when they knew the group was wrong. The participant went along with the group at least once 75 percent of the time. The participant went along with the group every time 37 percent of the time. When alone however, the participant made errors less than 1 percent of the time!

Why did Urbanization start during the Industrial Revolution?

People were losing their jobs on the farms to machines. They had to flock to cities where they could get a job at a factory. Also, new technology such as sewer, water, and transportation systems made larger urban centers possible.

Hormones can be classified into three types - what are they?

Peptide hormones steroids amino-acid derivatives

Peptide hormones are charged - how do they function if they cannot pass through the plasma membrane (due to charge)?

Peptide hormons cannot pass through the plasma membrane and hence function as a *first messenger* and bind to the receptor (usually G-protein coupled) to release a second signal called a *second messenger*. At each step there is a possibility of amplification. The connection between the hormone and the effect of its second messenger - referred to as *signalling cascade*. Common peptide hormones are insulin, ADH; common second messengers are cAMP, IP3 and Calcium

The _______ funnels sound into the ___________, where it will travel to the _______________, where the sound waves are then converted into ossicle vibrations. To fill in the blanks, please choose 3 from the following options: - Semicircular Canals - Pinna - Tympanic Membrane (eardrum) - Auditory Canal - Stapes

Pinna, Auditory Canal, Tympanic membrane

Describe the following retrieval cues: Spreading activation (priming)

Priming is the effect where one stimulus or concept activates a node in our semantic memory to think of other linked concepts being unconsciously activated - also called spreading activation e.g. the concept of red could relate to similar colours like orange, and also objects like roses or fire engines

Differentiate between a role, role performance and role set in terms of a doctor.

The *role* of the doctor is to help sick people. The *role performance* is the carrying out of the behaviours associated with that role. The various roles associated with the status of being the doctor is referred to as the *role set*

Describe the second step of the phototransduction cascade. Rhodopsin has changed it's shape, and there's a molecule of interest attached to it. What happens?

Rhodopsin changes shape after 11-cis retinal -> all trans retinal. A G-protein molecule, Transducin, is attached to rhodopsin, and subsequently breaks off. Transducin has an alpha, beta and gamma subunits, and the alpha subunit binds to PDE (phosphodiesterase).

Describe the following retrieval cues: State-dependent effect

Retrieval cue referring to people who learn facts or concepts in an altered state of mind, will recall the information best when in that state of mind.

Compare retroactive and proactive interference in terms of memory.

Retroactive interference is when some new piece of information makes it harder to recall previously learned information/memories. Proactive interference is when something learned in the past interferes with learning or retrieving something learned after.

When you walk out from the bright sunlight into a dark room, you are unable to see at first, but after a few minutes your eyes adjust and you can begin to see clearly. What causes this on a molecular level?

Rods account for dark and light (or B/W) vision. Rods have slow recovery or adjustment times - in bright light, they are hyperpolarized (and turned off) to allow activation of bipolar cells; for them to return to resting membrane potential and fire another action potential, it takes more time than it does for cone cells. Hence, when you go into a dark room from bright, it takes time for rods to reactivate to be able to clearly see in the dark

Sarah struggles with an eating disorder and body dysmorphia, meaning that she sees herself as being much fatter than she actually is. Because she is not as thin as the pictures she sees in magazines, she tends to feel bad about who she is. Describe how Carl Roger's Self-esteem, Self-image, and Ideal Self relate to this example.

Sarah feels a low Self-esteem as she views herself poorly. Her Self-image is skewed from reality as she sees herself as fatter than she really is. Her ideal self is likely based on the images she sees in the magazine. It is who she wants to be/thinks she should be.

Cluster A (Odd and eccentric) is defined by three types of personality disorder. Describe schizoid personality disorder

Schizoid personality disorder is a pervasive pattern of emotional detachment from social relationships + restricted emotional expression Little social interactions + poor social skills

An individual experiences symptoms of major depressive disorder, but for some reason only during the winter season. What sort of disorder might this be categorized as? How might this be treated?

Seasonal affective disorder (SAD). Best explained as major depressive disorder with seasonal onset, with symptoms present only in the winter months. Believed to potentially be related to abnormal melatonin metabolism, and often treated with bright light therapy (patient exposed to bright light for specified amount of time each day).

Compare and contrast the mechanism and effects of the secondary messenger system and primary messenger system.

Secondary Messenger: After first hormone is bound on an extracellular receptor, a series of reactions will cause a secondary messenger like cAMP to be produced in bulk. This allows amplification! Primary messenger: The receptor is intracellular. Hormones cross the cell membrane and bind to receptor. That binding can directly effect transcription/translation. It is simpler, but does not allow for amplification.

Why is Sensory adaptation important in terms of cells becoming over excited?

Sensory adaptation is necessary as otherwise cells remaining excited for too long leads to them being over-excited, which can induce damage and cell-death.

Compare Social Stigma and Self Stigma.

Social Stigma - negative belief toward a group of people (i.e. "mentally ill are dangerous"). Self Stigma - when one internalizes the Stigma that has been placed upon them (i.e. "maybe I really am dangerous and need to be locked away").

Differentiate between the biomedical and biopsychosocial approaches to psych- disorders.

The biomedical approach assumes disorders have roots in biomedical/biological disturbances and abnormalities, and therapy focuses on interventions that rally around symptom reduction. The biopsychosocial approach assumes there are biological, psychological and social components that contribute to an individual's disorder. The biomedical approach is typically narrower than the biopsychosocial and fails to take into account psychological and social factors like lifestyle and socioeconomic status.

In terms of the difference of choroids, explain why cats have better night vision than humans?

The choroid in humans is black in colour, and so absorbs all light and reflects none. However, in cats the choroid is a shiny colour, and thus light not absorbed by the retina is reflected back onto the retina after striking the choroid and given another chance at retinal absorption, resulting in better night vision for cats.

What are the function of the ciliary body?

The ciliary body has the ciliary muscles which control shape of the lens (by pulling on suspensory ligaments), and ciliary epithelium that produces aqueous humor to bathe the eye front before draining into the canal of Schlemm

What is the absolute threshold of a sensation?

The minimum stimulus needed to activate a sensory system and reach the CNS - it is a threshold in sensation but not perception. A stimulus below this would not reach the CNS. It can also be described as the minimum intensity of a stimulus to be able to detect a stimulus ~50% of the time - lowest level of a stimulus we can generally detect.

What is the purpose of the Dependency Ratio?

The purpose of the *Dependency Ratio* is to allow you to determine what percent of the population is dependent upon those who can work. The higher the ratio, the more dependent people there are on the workforce. Dependency Ratio = (population that is <14 and <65+)/(population that is 15 - 64 years) If the Dependency Ratio was 2 that would mean that every working person will need to support two dependent/non-working individuals.

In the second version of the experiment, the children were shown a man getting punished after he beat up the Bobo Doll. Some children still copied the man, but other children didn't. These children still learned the behavior, though, as evidenced by their ability to imitate the behavior after being bribed with candy. What did this reveal about the *Learning-performance Distinction*?

This experiment revealed that just because you learn something doesn't mean you are going to actually perform what you learned. This is the essence of the *Learning-performance Distinction*.

Explain why a one-year-old child really wants to touch the outlets in my house (no aversion), but he gets scared of a barking dog (aversion).

This is because my child is predisposed by evolution to know that barking dogs are dangerous whereas outlets have not been around long enough to result in any sort of evolutionary adaptation.

Jack sees a poor man on the street. He thinks to himself, "That poor fellow. He doesn't deserve this." Is this an example of the Just World Phenomenon?

This is not an example of the Just World Phenomenon. If it was, Jack would have said, "That fellow is getting what he deserves. He must be too lazy to get a job." Think of the "Just World Phenomenon" as a "Fair World", and it can be used to justify critical judgments.

What was Triesmans's Attenuation theory? How does it alter the selective filter idea?

This theory changed the idea of a selective filter to an attenuator, which would weaken but not eliminate other information that is not receiving selective attention. Instead of a complete filter, the attenuator that will not amplify the not-attended information. This focus could then be switched.

What is the expectancy-value theory of motivation?

This theory states that the amount of motivation needed to reach a goal is proportional to both the individuals expectations of success + degree to which they value succeeding at that goal

What does it mean when you say *medicalization*?

To medicalize something (an illness, for e.g.) means to take a phenomenon that was not medical before and bring it into the realm of biomedicine.

True or false: in operant conditioning, withdrawal of consequences causes extinction of target behaviour.

True! Just like how removing a stimuli in classical conditioning can cause extinction.

True or False? The retina sends information to the back of the eye through fibers that form the optic nerve, which ultimately leads to the brain for comprehension.

True.

True or false? The environment someone is raised in can cause epigenetic changes that will affect function for the rest of their life.

True.

True or false: stigmas can expand to affect others associated with particular individuals

True. e.g. family members of an alleged murdere or rapist

True or false: a big factor for participants in the Milgram experiment was the passing of responsibility of actions to others.

True. A big factor for participants in the Milgram experiment was the passing of responsibility of actions to others. Think of the term "I was just following orders"

True or False? At the tissue level, there have been no consistent findings that are "typical" of depression sufferers.

True. At the tissue level, there has been no consistent findings that are "typical" of depression sufferers.

What is amplification?

Up regulation of a certain stimulus

What is the problem with Urban Renewal though?

When urban areas are made nicer, property values rise, and the poor are no longer able to afford housing there and are forced to leave to poorer areas. The new residents benefit while the previous residents are essentially punished and lose their home.

With Obsessive Compulsive Disorder (OCD), where does the distress or disability come into play?

With OCD, distress comes into their everyday life when their compulsions interfere with daily activities.

In signal detection theory, what do the notations d' and c mean?

d' = refers to strength of the signal c = refers to strategy

The opening or dark space in the middle of the eye, known as the pupil, adjusts its size based on the amount of light it receives. How does the pupil adjust its size in a dark environment? Light environment?

If it is dark outside, the pupil will expand to increase the amount of light able to enter the eye, while if it is light outside, the pupil will contract to decrease the amount of light able to enter the eye.

Jackson is very good at starting new things and makes his own plans for what he is going to do each day. How old is Jackson? (A) 5 years (B) 21 years (C) 48 years (D) 68 years

(A) 5 years Jackson is overcoming Initiative vs. Guilt, which occurs between the ages of 3 and 6.

Mary is trying to figure out who she is. She is experimenting with new things to figure out what she likes and where she fits in. How old is Mary? (A) 8 months (B) 2 years (C) 7 years (D) 13 years

(D) 13 years Mary is struggling with Identity vs. Role Confusion, which occurs between the ages of 12 and 18.

Lamar comes from a technologically superior culture to Jane. Both are asked to describe a new Tablet, and although Lamar can use specific tech words, Jane can only describe it as "shiny" and "bright". A specialist claims that Jane must not have thought about the Tablet in the same sense that Lamar did based on her response. Which theory of Language Development would this specialist most agree with? (A) Nativist Approach (B) Learning Reinforcement (C) Interactionist Approach (D) Universal Approach

(D) Universal Approach The Universal Approach states that thoughts precede and dictate language, so Jane not being able to come up with words to describe the advanced technology means her thoughts about them were also under-developed.

What three questions do we ask ourselves when developing our Looking Glass Self?

1. How do I appear to others? ("I walk upright") 2. What do others think of me? ("That guy is too serious") 3. How should this affect how I think about myself? ("Maybe I am a little to serious")

Having strong self-efficacy is great, but a weak sense of self-efficacy can lead to failure, avoidance, etc.; There are four ways to develop Self-efficacy. Pretend that you want to get better at playing the piano, describe how each of these things may build up your self-efficacy: 1. Mastery of Experience 2. Social Modeling 3. Social Persuasion 4. Psychological Responses

1. Mastery of Experience - You practice the piano all the time, so you start to feel more confident. 2. Social Modeling - You see your friend who has been playing the piano as long as you, play really well. 3. Social Persuasion - Your friends compliment you and even tell other people that you are good at the piano. 4. Psychological Responses - You develop the ability to decrease your stress level and anxiety before a concert.

Which of the following are the two substages of the Pre-conventional Stage? -Self-Interest (Individualism and Exchange) -Conformity and Interpersonal Accord -Social Contract -Obedience vs Punishment -Universal (Ethical) Principles -Authority and Social Order

1. Obedience vs. Punishment 2. Self-interest (Individualism and Exchange)

Match the following components of language to their meanings: - Phonology - Morphology - Semantics - Syntax - Pragmatics (A) The dependence of language on context and pre-existing knowledge. (B) The structure of words (C) How words are put together to form sentences (D) The actual sound of words (E) The association of meaning with a word

A - pragmatics B - morphology C - syntax D - phonology E - semantics

Why can a cochlear implant help patients with sensorineural hearing loss (aka nerve deafness), but not cochlear or downstream nervous issues?

A cochlear implant functions by stimulating the oval window of the cochlea when its transmitter is activated. It can overcome conduction errors from the middle ear, but can't fix errors downstream of the oval window.

What is depersonalization/derealization disorder?

A dissociative disorder where individuals feel detached from their own mind and body (depersonalization) or from their surroundings (derealization) - can be felt simultaneously.

What is a mental set?

A mental set is the tendency to approach similar problems in the same way

What is the difference between Altruism and Reciprocal Altruism?

Altruism is when you help selflessly without expecting anything in return. Reciprocal Altruism is when you help but expect a return sometime down the road.

Explain the carbon-ring based structure of steroids (triterpenes).

An isoprene has the carbon backbone of 2-methyl butyl (see image), and a triterpene has 6 isoprene groups bound to form 4 Carbon rings: 3 cyclohexane rings and one cyclopentyl ring.

Explain the carbon-ring based structure of steroids (triterpenes).

An isoprene unit has a C-backbone of 2-methyl butyl (image). A triterpene is made up of 6 isoprene groups, bound to form 4 rings of Carbon - 3 cyclohexanes and one cyclopentyl ring

Cluster B (dramatic/emotional) is defined by which four types of personality disorder? (important image)

Antisocial, borderline, histronic and narcissistic personality disorders.

When using the situational approach to attribution theory, we look at the external component of attribution which has three parts/cues: consistency, distinctiveness and consensus

Consistency - does the person usually act this way in the situation? Distinctiveness - does the person behave differently in different situation, or this particular situation distinct? Consensus - do others typically behave similarly in this situation? If you can answer yes to the second two components (distinctiveness and consensus), then you can infer the person is behaving in a particular way due to the situation and external factors.

What are the hormones released during 'fight or flight'? Compare them to what is released during 'tend and befriend'

During fight or flight, ACTH is released to stimulate release of cortisol; epinephrine and norepinephrine (aka adrenaline and noradrenaline) are also released During tend and befriend - oxytocin

What are explicit (declarative) memories?

Explicit memories are facts or events that can clearly and explicitly be described (e.g. state capitals, vocabulary etc.) - uses semantic memory (having to do with words) or episodic memory (memory for events)

True or false: as levels of plasma calcium rise, PTH secretion increases.

FALSE. PTH secretion decreases with increasing levels of plasma Ca.

True or false: Introns code for specific proteins, while exons in DNA are regulatory regions that do not and are spliced out during post-transcriptional modification.

FALSE: EXONS code for specific proteins, while INTRONS in DNA are regulatory regions that do not and are spliced out during post-transcriptional modification.

True or False? Symptoms can still be considered a mental disorder if the symptoms are being caused by drug usage.

False. Symptoms CANNOT be considered a mental disorder if the symptoms are being caused by drug usage.

Two of the negative symptoms of schizophrenia is the flat affect (emotional flattening) and inappropriate affect - describe this symptoms

Flat affect refers to the symptom where there are virtually no signs of emotional expression in the patient. Inappropriate affect - discordance with content of speech - e.g. laughing while describing someone's death

Describe the basic premise of Functionalism and how it relates to the idea of Homeostasis.

Functionalism views society like an organism. When something is wrong with part of the Society, homeostasis is lost, throwing the society out of equilibrium. It sees all aspects of Society as having a function and as necessary for the survival of the Society.

What are the physiological effects of hallucinogens? What are some examples?

Hallucinogens cause individuals to experience distorted perceptions, including hallucinations or heightened sensations that are not real. They may cause increased/decreased energy but are classified by their perceptual effects. E.gs - LSD, psilocybin, PCP

What is interference?

Interference (or the interference effect) is another reason for memory loss - retrieval error caused by existence of other similar information.

What are the roles of somatostatin?

Somatostatin is produced by delta cells of the islets of Langerhans (pancreas) - stimulated by high blood sugar and AA concentrations - inhibitor for both glucagon and insulin. Also produced by hypothalamus to decrease GH secretion.

Of the 4 main stages of sleep, which stages are characterized as Non-Rapid Eye Movement (non-REM)?

Stages 1, 2 and 3 (N1, N2, N3)

How do socioeconomic factors in society influence the education system?

Teachers put students into categories based on perceived intrinsic abilities, as well as other things like gender, race, ethnicity, residential segregation etc. creating inequalities between school districts. Schools in lower-income districts will receive less funding than schools in more affluent areas, creating a social inequality from the get-go.

Contrast the Cannon-Bard theory of emotion with the James-Lange theory of emotion.

The Cannon-Bard theory claims an event can cause a physiological response and an emotion simultaneously. The James-Lange theory claims a physiological response will occur, and then its perception will cause an emotion.

Why might you hypothesize as to why criminals and drug addicts tend to have smaller Frontal Lobes?

The Frontal Lobe is responsible for impulse control, and reduced activity may lead to these individuals being less able to control their aggressive impulses and/or drug addictions.

Describe Frued's element of the Id

The Id is the most unconscious part of our mind that develops at birth. It operates under a *pleasure principle* which demands instant gratification. The *primary process* is the id's response to frustration, which is to seek satisfaction immediately.

Within the prefrontal cortex, describe the function of the following: - dorsal prefrontal cortex - ventral prefrontal cortex - ventromedial prefrontal cortex (A) Associated with attention and cognition (B) Plays roles in decision making and controlling emotional responses from amygdala (C) Connects regions of the brain responsible for experiencing emotions

The dorsal prefrontal cortex: (A) associated wit attention and cognition The ventral prefrontal cortex: (C) connects with regions of the brain responsible for experiencing emotion The ventromedial prefrontal cortex: (B) plays roles in decision making and controlling emotional responses from the amygdala

Describe Frued's element of the ego

The ego and superego both have conscious and unconscious elements. The ego operates according to the *reality principle*, and takes reality into account to inhibit or guides the activity of the id - guidance referred to as *secondary process*. The aim of this principle is to postpone the pleasure principle until satisfaction is actually obtainable. The ego also moderates the superego

Arriving at the scene, an EMT needs to quickly assess the situation, including any bleeding, discoloration, injuries, etc. How does the parallel processing of multiple vision pathways enable them to do this quickly and efficiently?

The parallel processing of multiple pathways allows different aspects (color, form, motion) to be processed quickly at the same time, and integrated at higher vision centers.

Parts of the eye: sclera

The thick white covering layer of the eye, does not cover the cornea - supplied nutrients by two bloodvessels: choroidal and retinal vessels. Composed of protein called collagen as well as elastic fibers

Vygotsky's sociocultural theory of development studied the role that social interaction plays in the development of cognition. Describe his concept of *zone of proximal development*

Vygotsky's theory stated the engine driving cognition was the child's internalization of various aspects of culture: rules, symbols, language etc. using four elementary mental functions: attention, sensation, perception and memory. The *zone of proximal development* concept refers to those skills and abilities that are in the process of but not yet fully developed. Gaining these skills requires help of a "more knowledgeable other"

Racial segregation prior to the time of Dr King is a prime example of _____________________ in the United States. (A) Individual discrimination (B) Institutional discrimination (C) Propaganda (D) Ethnocentrism

(B) Institutional discrimination Racial segregation prior to the time of Dr King is a prime example of *institutional discrimination* in the United States. Individual discrimination refers to one person's discrimination against a particular person or group (e.g. a sexist CEO). Institutional discrimination is discrimination against a person/group by an entire institution (Apartheid)

A very ambitious father is trying to teach his 12-month-old daughter to potty-train, but without any success. How is this related to the Nature versus Nurture debate? (A) No matter how much Nature the child is born with, nothing can cause this milestone to be reached until the Nurture of the parent is appropriate (the father changes his approach to match the daughter's learning style). (B) No matter how much Nurturing the father gives, this milestone cannot be reached until Nature has appropriately taken its course (developing bladder muscles). (C) If the ambitious father keeps providing this level of Nurturing, he will overcome the daughter's Nature and get her to be potty-trained. (D) There are no Nature versus Nurture implications of potty-training.

(B) No matter how much Nurturing the father gives, this milestone cannot be reached until Nature has appropriately taken its course (developing bladder muscles).

Freddy belongs to his town's Rotary Club, which is a group of individuals that try to serve the community. Members are not reimbursed for their membership. Which type of Organization does Freddy belong to? (A) Coercive Organization (B) Normative Organization (C) Bureaucratic Organization (D) Utilitarian Organization

(B) Normative Organization Because members are united by a sense of purpose as opposed to economic reasons, this would be an example of a Normative Organization.

Which category of behavior is characterized by regulating people spacially in their environment, like Taxis? (A) Reflexes (B) Orientation behaviors (C) Fixed Action Patterns (D) None of the Above

(B) Orientation behaviors are characterized by regulating people spacially in their environment, like through Taxis or Kinesis.

Which of the following endocrine glands is most likely going to be affected after digesting a meal with a large amount of calcium? (A) Thyroid (B) Parathyroid (C) Pancreas (D) Hypothalamus

(B) Parathyroid Parathyroid hormone is partly responsible for controlling calcium levels in the blood.

In which part of cerebral cortex is the sensory strip located? (A) Frontal Lobe (B) Parietal Lobe (C) Temporal Lobe (D) Occipital Lobe

(B) Parietal lobe

Migration contributes to population growth, and can be influenced by _________________ (positive attributes attracting migration) and ________________ (negative attributes encouraging immigrants to leave). (A) Push factors, pull factors (B) Pull factors, push factors (C) Demographic transition, Malthusian theory (D) Malthusian theory, Demographic shift

(B) Pull factors, push factors Migration contributes to population growth, and can be influenced by *pull factors* (positive attributes attracting migration) and *push factors* (negative attributes encouraging immigrants to leave). Pull factor example can be "American dream" ideology. Push factor example can be Islamization of Pakistan causing foreigners to leave.

The posterior pituitary receives and stores ADH and oxytocin, which are direct hormones. It responds to increased blood osmolarity and low blood volume by: (A) Releasing oxytocin to act on uterus (B) Releasing ADH to act on kidneys (C) Releasing ADH to act on the breasts (D) Releasing oxytocin to act on kidneys

(B) Releasing ADH to act on kidneys ADH is secreted in response to low blood volume and high blood osmolarity. Its action is at the level of the collecting ducts in the kidneys - increased permeability of water = results in increased water retention, higher blood pressyre and increased blood volume.

Marge writes an article for her school newspaper entitled, "10 Things You Need to Start a Social Movement." Marge's Essay most likely lends support to which theory of Social Movements? (A) Mass Society Theory (B) Resource Mobilization Theory (C) Relative Deprivation Theory (D) Rational Choice Theory

(B) Resource Mobilization Theory Resource Mobilization Theory suggests that starting a Social Movement requires various resources such as time, money, and a charismatic leader. Perhaps Marge will mention these things in her article.

_______________ Benzodiazepines are used to treat insomnia, while _______________ Benzodiazepines are used to treat anxiety. (A) Short-Acting, Short-Acting (B) Short-Acting, Long-Acting (C) Long-Acting, Short-Acting (D) Long-Acting, Long-Acting

(B) Short-Acting, Long-Acting

Which theory of dreaming explains that dreams are our unconscious thoughts and desires that need to be interpreted? (A) Evolutionary Biology theory (B) Sigmund Freud's Dream theory (C) Memory Consolidation Theory (D) Neural Pathway Theory

(B) Sigmund Freud's Dream theory

If a subject watches a video and is later told incorrect information about the video, they are more likely to recall the video incorrectly. Which of the following is a cause of this? (A) Context Clues (B) Source monitoring error (C) Recency Effect (D) Primacy Effect

(B) Source monitoring error Source monitoring error is a confusion between semantic and episodic memory, when a piece of false or misleading information influences the memory of one's recollected experience This is an example of the misinformation effect

Which of the following is not a trait of innate (genetically encoded, not taught) behavior? (A) Intrinsic (B) Inflexible (C) Environmentally fit / well adapted (D) Stereotypical

(C) Environmentally fit / well adapted Traits of innate behavior include being Inherited, Intrinsic, Stereotypical, Inflexible and Consummate (fully developed right away)

In Africa, the Fertility Rate is close to 6, and due to high levels of disease people die at a relatively young age. What type of population pyramid would you expect to see in this situation? (A) Constrictive Population Pyramid (B) Retroactive Population Pyramid (C) Expansive Population Pyramid (D) Stationary Population Pyramid

(C) Expansive Population Pyramid

You give your sister some licorice at the end of each hour that she avoids annoying you. This example is most illustrative of which partial reinforcement schedule? (A) Fixed-ratio (B) Variable-ratio (C) Fixed-interval (D) Variable-interval

(C) Fixed-interval *Fixed-interval (FI) schedules* reinforce the first instance of a behaviour after a specified time period, e.g. every 60s. You are rewarding your sister according to a timetable that is fixed, thus it is FI schedule.

Which of the following is not one of Lazarus' three categories/responses of primary appraisal? (A) Fearful (B) Irrelevant (C) Benign/positive (D) Stressful

(A) Fearful

Which of the following examples best exhibit Darwin's theory that emotions have an adaptive or inherent value? (A) A child sees their parent bargaining with a sales clerk for a discount, and will attempt bargaining with their teacher. (B) A toddler is told to use their words to explain what is wrong, but the toddler only cries. (C) A child throws a tantrum after watching their younger sibling complain and get a toy. (D) A blind man will use similar facial expressions as the general public, even though he's never seen them.

(D) A blind man will use similar facial expressions as the general public, even though he's never seen them. The other responses would require to be learned and not innate.

Which of the following are damaged if a patient is suffering from Conduction Aphasia? (A) Broca's Area (B) Wernicke's Area (C) Both A and B (D) Arcuate Fasciculus

(D) Arcuate Fasciculus

Which of the following are damaged if a patient is suffering from Global Aphasia? (A) Broca's Area (B) Wernicke's Area (C) Both A and B (D) Arcuate Fasciculus

(D) Arcuate Fasciculus

What refers to the process in which the cochlea distinguishes between varying frequencies and is maintained by the brain? (A) Frequency Processing (B) Signal Detection Theory (C) Parallel Processing (D) Auditory Processing

(D) Auditory Processing

What is detected by the 4 Types of Somatosensation? - Thermoception - Mechanoception - Nociception - Propioception

- Thermoception = temperature - Mechanoception = pressure - Nociception = pain - Propioception = position

Describe the relationship of the Bourgeoisie and the Proletariat according to Karl Marx's Conflict Theory.

Bourgeoisie had the power and own the means of production such as a factory. The Proletariat are the workers that do the producing such as being factory workers. The Proletariat are dependent upon the Bourgeoisie for their livelihood, and likewise the Bourgeoisie were dependent on the Proletariat for their labour to function and maintain status quo.

What are examples of permanent reflexes?

Breathing reflex, eye blink reflex, pupillary reflex, swallowing reflex

Compare compliance, identification and internalization (all ways of being motivated by different influences or pressures)

Compliance is doing what you are asked regardless of whether you want to or not. Identification - when we go along with something under motivation of being like a certain famous figure Internalization - conforming not only publicly but also privately with a certain behaviour - internalizing the ideas - most powerful because not only do you, but you also want to!

Describe Piaget's concrete operational stage

Concrete operational stage: *~7-11 years* - Concept of *conservation* has developed + able to engage in logical thought and solve math problems

What are the coping mechanism types for cognitive dissonance?

Described from perspective of smoker: 1) Modifying - modifying the cognitions; "i don't really smoke that much" 2) Trivialize - trivializing/making less important their cognitions - "evidence is weak that smoking causes cancer" 3) Add - adding more cognitions - "i exercise so much it doesn't matter I smoke" 4) Denying the cognitions - denying any relations between the cognitions - "there's no evidence smoking and cancer are linked"

Differentiate between direct and indirect therapy. Which approach typically subscribes to which one (biomedical vs biopsychosocial approaches)?

Direct therapy refers to treatment that acts directly on the individual such as medications and periodic meetings with the psychologist. Indirect therapy aims to increase social support by educating and empowering family and friends of affected individual. The biomedical approach uses direct therapy; biopsychosocial uses both.

True or false: the image projected on the retina is upright

False: the image is inverted

Describe the term and what kind of cue it is: relative size

Relative size - monocular cue that allows us to perceive two things far apart as relatively the same size When objects are of relatively equal size, the one closer takes up more space in the visual field

Draw or visualize an example of The Law of Proximity. Describe this law.

The Law of Proximity states that objects that are close to each other are usually grouped together.

What is parallel processing?

The ability to detect all information relating to feature detection (color, form, motion) simultaneously.

How do trait theories define personality?

Trait theories of personality use clusters of behavioural patterns to describe individuals - the combination and interaction of various traits defines a personality unique to each individual. No two people by these theories have the same personality.

True or False? The encoding strategy of Spacing explains that learning occurs best when spread out over time, not cramming.

True. The encoding strategy of Spacing explains that learning occurs best spread out over time, not cramming.

True or false: innocent ethnic pride and violent supremacy groups are both examples of ethnocentrism.

True. They are just different ways ethnocentrism can manifest in.

True or false: Individuals with sensorineural hearing loss are unable to convert sound waves into electrical impulses

True. They are unable to trigger hair cells to do so. "nerve deafness"

The tip link of a kinocilium is attached to the gates of a K+ channel. Explain how an action potential is generated when fluid begins to move inside the cochlea.

When fluid moves in the cochlea, the tip links are pushed back and forth and subsequently stretched by movement in the cochlea. This stretching activates K+ channels they are linked to, allowing K+ to enter the cell. The entry of K+ activates Ca2+ channels inside the cell, and Ca2+ is also allowed into the cell. This causes an action potential which activates the auditory nerve.

What is *wish fulfillment* and how is it part of the Id?

Wish fulfillment refers to mental imagery like fantasy or daydreaming - it serves as a ways to fulfill satisfaction, which is the function of the Id.

Jack finds himself in a Holocaust type of situation in which he is told to kill others. After this event, he is asked if he feels responsible for his actions. He responds, "No. I was just following orders. I only acted the way I did because of my situation." When asked about his fellow soldiers who did horrible things, he responds by saying, "they, on the other hand, are terrible people and did what they did because they are flawed." Is this an example of the Fundamental Attribution Error?

Yes. This is a perfect example as Jack is attributing others' actions to their internal characteristics while attributing his own actions to his situation.

Freddie's needs are not being met fully, leading him to develop a sense of skepticism toward the world around him. How old is Freddie? (A) 8 months (B) 2 years (C) 7 years (D) 13 years

(A) 8 months Freddie is struggling with Trust vs. Mistrust, which occurs between the ages of 0 and 1.

Which of the following refers to the prejudice and discrimination on basis of a person's age? It views older individuals as frail vulnerable, less intelligent, less respect (A) Ageism (B) Lookism (C) Age cohort (D) Dependency

(A) Ageism *Ageism* is prejudice or discrimination on basis of a person's age.

Light hits a photoreceptor in the eye and causes the the cell to fire an action potential. That cell is connected to two other cells, which also fire action potentials. What does this form of up-regulation refer to? (A) Amplification (B) Sensory Adaptation (C) Habituation (D) Proprioception

(A) Amplification

In the Cochlea, what structure moves back and forth and is responsible for transmitting an electrical impulse via the auditory nerve to the brain? (A) Cilia (Hair Cells) (B) Oval Window (C) Circular Window (D) Stapes

(A) Cilia (hair cells)

Match each emotional state with the brain hemisphere that it is usually associated with: (A) Left Hemisphere (B) Right Hemisphere -Positive emotions -Negative emotions -Isolative interactions -Social interactions

(A) Excitation of the Left Hemisphere - positive emotions and social interactions associated. (B) Excitation of the Right Hemisphere - negative emotions and isolative interactions associated.

Put the following stages of a society in order according to Karl Marx's Conflict Theory: I. Capitalism II. Socialism III. Feudalism (A) III > I > II (B) III > II > I (C) I > II > III (D) II > I > III

(A) III > I > II Marxist conflict theory was based on the idea of class struggle, and how the ideas portrayed in society were the ideas of the ruling class. Began with feudalism, then capitalism, and finally socialism. The ruling class were the minority rich group known as the *bourgeoisie* and the majority poor class were the *proletariat* class.

Which route of drug entry refers to ingesting a drug, and is one of the slowest routes because of the time it takes for the drug to pass through the GI tract? (A) Oral (B) Inhalation (C) Injection (D) Transdermal

(A) Oral

Which taste cells rely on Ion Channels? (A) Salty, Sour (B) Bitter, Sweet (C) Sweet, Umami (D) Bitter, Umami

(A) Salty, Sour Salty and sour molecules bind to receptors on ion channels in tastes cells, which pumps ions to trigger an action potential to the brain.

Infants' motor developments generally occur in order of: (A) Superior to Inferior (B) Anterior to Posterior (C) Distal to Proximal (D) Fine motor to Gross Motor

(A) Superior to Inferior Infants' motor developments generally occur in order from head to to (superior to inferior).

Which of the following is not a part of the General Adaptation Syndrome (G.A.S.)? (A) Alarm (B) Action (C) Exhaustion (D) Resistance

(B) Action

The guards told themselves that the prisoners deserved what they got. This allowed them to feel better about their brutal actions. This is an example of what? (A) Deindividuation (B) Cognitive Dissonance (C) Fundamental Attribution Error (D) Confirmation Bias

(B) Cognitive Dissonance Cognitive Dissonance is when you change the way you think about a situation in order to avoid dissonance between your behavior and your beliefs. This is what the guards did to feel better about their actions.

John says that you should never use potty language in public because to do so would result in other people laughing at you. He wants to fit in. Which stage of moral development is John currently in? (A) Pre-conventional (B) Conventional (C) Post-conventional - Social Contract (D) Post-conventional - Universal Principles

(B) Conventional John is concerned about securing the approval of others, which is characteristic of the first level of the Conventional Stage.

Which route of drug entry refers to breathing in, snorting, or smoking the particular drug? This method is highly addictive and it does not take long for the drug to travel to the brain. (A) Oral (B) Inhalation (C) Injection (D) Transdermal

(B) Inhalation

What is another term for sleep walking? (A) Sleep apnea (B) Somnambulism (C) Cataplexy (D) Sleep paralysis

(B) Somnambulism. Sleep walking is also known as somnambulism, and occurs during SWS.

Before taking an math exam, girls are informed that typically girls will perform worse on math-related exams. Will the girls perform better or worse than if they had not been informed of this? This is an example of: (A) Minority Challenge (B) Stereotype Threat (C) Self-concept (D) Front-stage Self

(B) Stereotype Threat This is an example of Stereotype threat, and the girls will perform worse once they are aware of the stereotype.

What personality type is more predisposed to engage in prejudice? (A) Submissive (B) Neurotic (C) Authoritarian (D) Oppressed

(C) Authoritarian The Authoritarian Personality Type is more predisposed to engage in prejudice.

Which membrane in the eye is right behind the retina and is a network of blood vessels that provides nutrients to retinal cells as well as other cells in the eye? (A) Cornea (B) Sclera (C) Choroid (D) Optic Nerve

(C) Choroid Choroid is a membrane in the eye that is right behind the retina and is a network of blood vessels that provides nutrients to retinal cells as well as other cells in the eye

Which of the following is not a bias that can impair decision making? (A) Confirmation Bias (B) Overconfidence (C) Heuristics (D) Belief Perserverance

(C) Heuristics

Who of the following are considered proponents of the Humanistic Theory? I. Carl Rogers II. Carl Jung III. Abraham Maslow (A) I and II Only (B) II and III Only (C) I and III Only (D) I, II, and III

(C) I and III Only

Put Piaget's Four Stages of Cognitive Development in order from earliest to latest stage: I. Pre-Operational II. Formal Operational III. Sensorimotor IV. Concrete Operational (A) I > II > III > IV (B) I > III > IV > II (C) III > I > IV > II (D) III > IV > II > I

(C) III > I > IV > II 1) Around 0-2 years is the Sensorimotor stage 2) ~2-7 years is the Pre-Operational stage 3) ~7-11 years is Concrete Operational stage 4) ~12+ is Formal Operational stage

Jessica took a math test and got a terrible grade. She says, "I just didn't study hard enough." She has what kind of Locus of Control? (A) Positive (B) Negative (C) Internal (D) External

(C) Internal Jessica is attributing her failure to internal issues, which is characteristic of an Internal Locus of Control.

A trial-and-error approach where testing behaviours until a reward is yielded is: (A) Predisposition (B) Latent learning (C) Problem solving (D) Instinctive drive

(C) Problem solving

Which of the following is not one of the major structural classifications of hormones? (A) Polypeptides/proteins (B) Steroids (C) Tryptophan derivatives (D) Tyrosine derivatives

(C) Tryptophan derivatives

Because the cornea is really sensitive it needs to be protected. The ___________ is a thin layer of epithelial cells that protects the cornea from friction and helps to moisturize the cornea as well. (A) Iris (B) Pupil (C) Retina (D) Conjunctiva

(D) Conjunctiva

What made the model T difference from past cars is that it was produced by machines that replaced human labor. Which of the four aspects of McDonaldization does this situation best exemplify? (A) Efficiency (B) Calculability (C) Predictability (D) Control

(D) Control Control refers to the idea that workers are replaced by machines. It also refers to workers wearing standardized uniforms and workers that can easily be replaced by another low-wage worker.

Bandura's Bobo Doll experiment introduced concepts of reciprocal determinism and locus of control, crucial to the social cognitive perspective. Reciprocal determinism is idea that out thoughts, feelings, behaviour and environment all interact with each other to determine our actions in a given situation. Which of the following is not an element of Reciprocal Determinism as seen by Social-Cognitive Theory? (A) Actions (B) Thoughts (C) Environment (D) Disposition

(D) Disposition Actions/Behavior, Thoughts/Cognition, and Environment are all elements of Reciprical Determinism.

What is the Peter Principle? Which characteristic of a Bureaucracy may cause the Peter Principle? (A) Division of Labor (B) Hierarchy of Organizations (C) Written Rules and Regulations (D) Employment based on Technical Qualifications

(D) Employment based on Technical Qualifications The Peter Principle is the idea that when individuals are promoted based on technical qualifications, they are promoted over and over until they reach a position that they cannot perform well, at which point they won't get promoted due to their incompetence, yet they will stay there because they have nowhere else to go.

Which of the following is not a category of innate behavior caused by a stimulus? (A) Reflexes (B) Orientation behaviors (C) Fixed Action Patterns (D) Reproductive need

(D) Reproductive need The three categories of innate behavior are Reflexes, Orientation Behaviors (taxis and kinesis), and Fixed Action Patterns

A G-Protein falls under which of the following classes? (A) Tyrosine Derivative (B) Polypeptide Hormone (C) Primary Messenger System (D) Secondary Messenger System

(D) Secondary Messenger System G-Proteins and G-protein coupled receptors are classic examples of the Secondary Messenger system.

What refers to a burst of rapid brain activity during sleep, that some researchers believe help inhibit certain perceptions so that one can maintain a tranquil state during sleep? Ex: Associated with the ability to sleep through loud noises. (A) K-Complexes (B) non-REM sleep (C) REM sleep (D) Sleep spindles

(D) Sleep spindles Sleep spindles refer to a burst of rapid brain activity during sleep, that some researchers believe help inhibit certain perceptions so that one can maintain a tranquil state during sleep

Compare fluid intelligence and crystallized intelligence, and which increases/decreases/remains the same with age.

*Fluid intelligence (IQ)* is the ability to perceive relationships and solve new problems, use logic in new situations and identify patterns. Involves being able to think abstractly and solve problems without prior practice, learning or experience (E.g. solving a puzzle). *Crystallized intelligence (IQ)* involves knowledge that comes with experience and prior learning - intelligence based on facts and rooting experience (e.g. vocabulary). Fluid intelligence declines into late adulthood; crystallized IQ increases with age.

Describe interpersonal attraction and what factors affect attraction

*Interpersonal attraction* is the phenomenon of individuals liking each other - several factors including similarity, self-disclosure, reciprocity, proximity, face-symmetry, dimorphism play roles in determining attraction. In general, the more symmetrical a face is, the more attractive it is. High cheek bones are a huge point for attraction. Furthermore, so is the *golden ratio* (1.618:1) and high cheek bones.

Describe social epidemiology

*Social epidemiology* is a branch of epidemiology that focuses particularly on the effects of social-structural factors on states of health - it assumes that the distribution of advantages and disadvantages in a society reflects the distribution of health and disease.

Describe the World Systems theory of globalization and how it categorizes areas of the globe

*World systems theory* of globalization - focuses on importance of the world as a unit, rather than looking at individual countries, and divides it into 3 regions: core, peripheral and semi-peripheral countries. _Core countries_ include areas like the US, Western Europe, Australia - have strong central govts, economically diversified, industrialized, and independent of outside control. Strong working class force, focuses on production of higher scope material goods rather than raw materials. _Periphery countries_ include areas like Latin America and Africa, which have a relatively weak central govt. Tend to depend on one economic activity (e.g. extracting raw materials). High numbers of poor people and small, controlling upper class creating huge social inequality. Highly influenced by core countries and trans-national corporations. _Semi-periphery countries_ such as India and Brazil make up the middle ground between the two extremes. Not dominant in intl trade but have a diversified and decently developed economy

Some of the key hormones released by the hypothalamus into the pituitary are GnRH, GHRH, TRH and CRF. What do each of these do as they go bind to receptors in the anterior pituitary?

- Gonadotropin-releasing hormone (GnRH) --> stimulates release of FSH and LH - Growth-hormone releasing hormone (GHRH) - stimulates release of growth hormone (GH) - Thyroid-releasing hormone (TRH) - release of thyroid-stimulating hormone (TSH) - Corticotropin-releasing factor (CRF) - release of adrenocorticotropic hormone (ACTH)

Classify each of the following as primary or secondary sexual characteristics, and if they occur in males, females or both: -Testes development -Underarm and pubic hair -General body hair - Hips development

-Testes development - primary sex characteristic in males -Underarm and pubic hair - secondary sex characteristics in both -General body hair - secondary sex characteristics in males - Hips development - secondary sex characteristics in females

In a stressful event, the body goes into "fight or flight mode, where the body increases heart rate and decreases digestion. Hans Selye developed a sequence of physiological responses called the general adaptation syndrome - describe it's three distinct stages (alarm, resistance, exhaustion)

1) Alarm - initial stage where body activates sympathetic NS - hypothalamus stimulates pituitary to release ACTH* which stimulates the adrenal glands to produce cortisol to maintain steady supply of blood + adrenal medulla to release epinephrine and norepineprhine* 2) Resistance - continuous release of hormones keeps the sympathetic NS stimulated for the body to remain engaged to combat stressor 3) Exhaustion - when the body can no longer maintain elevated response with sympathetic NS activity - individuals become more susceptible to illnesses and medical conditions, and death in extreme cases *ACTH = adrenocorticotropic hormone; epinephrine/nor is the same as adrenaline/noradrenaline

Studies have shown three factors influence our ability to multitask. What are they?

1) Task similarity - tasks that are similar would be difficult to multitask to 2) Task difficulty - difficulty of tasks definitely influences (easy to sing to radio and drive than text and drive) 3) Practice - activities well practiced do not need as much attention as they become "automatic"

Think about a grocery store, describe how the _five characteristics of a Bureaucracy_ might be seen there: 1. Division of Labor 2. Hierarchy of Organizations 3. Written Rules and Regulations 4. Impersonality 5. Employment based on Technical Qualifications

1. Division of Labor - The cashier checks people out while the bagger bags groceries, and the stock room workers stock the shelves. No one does each other's job. Each is trained to do one specific task. 2. Hierarchy of Organizations - The CEO of the grocery store is above the regional managers who is above the store managers who is above the hourly-wage workers. 3. Written Rules and Regulations - The cashier must follow exact procedures when checking out customers. 4. Impersonality - A cashier cannot give out special discounts even to their friends. 5. Employment based on Technical Qualifications - Employment at a grocery store is based on your resume, GPA, and ACT score.

Which of the following are the two substages of the Post-conventional Stage? -Self-Interest (Individualism and Exchange) -Conformity and Interpersonal Accord -Social Contract -Obedience vs Punishment -Universal (Ethical) Principles -Authority and Social Order

5. Social Contract 6. Universal (Ethical) Principles

Schizophrenia is a psychotic disorder there individuals suffer from one or more of the following conditions: delusions, hallucinations, disorganized thought or behaviour, catatonia, and negative symptoms. For a person to be diagnosed, they must show symptoms for at least ____________ and this time period must include at least ____________ of "active symptoms" (delusions, hallucinations, or disorganized speech)

6 months, 1 month Schizophrenia is a psychotic disorder there individuals suffer from one or more of the following conditions: delusions, hallucinations, disorganized thought or behaviour, catatonia, and negative symptoms. For a person to be diagnosed, they must show symptoms for at least 6 months and this time period must include at least 1 month of "active symptoms" (delusions, hallucinations, or disorganized speech)

Differentiate between fads, mass hysteria and riots.

A *fad* is a behaviour that becomes very popular quickly and also loses popularity quickly - reach influence of large numbers of people in their time. E.g. cinnamon challenge on Youtube, "low-carb diet for weight loss" *Mass hysteria* refers to a shared, intense delusional concern about threats to society shared by many people - largely influenced by groupthink; delusion augmented by distrust, rumors and propaganda. E.g. Salem trials where people were killed for fears of witchcraft; Anthrax attacks *Riots* are characterized by large groups of people suddenly engaging in deviant behaviour (violence, theft, vandalism etc) - usually fueled by deindividuation.

What is alcohol myopia?

A short-sighted view of the world caused by excess alcohol consumption, resulting in the inability to recognize consequences of actions due to reduced logical reasoning.

How does the cochlea differentiate between sound frequencies?

A sound wave of 100Hz (low frequency) will cause vibrations which move fluid in the cochlea to travel farther towards the apex until it reaches a point where hair cells sensitive to that particular frequency respond.

What are the 3 types of nerve fibers? Distinguish between them

A-beta nerve fibers - largest diameters and plenty of myelin A-gamma nerve fibers - medium, some myelin C-fibers - smallest diameters, no myelin Combo of size and myelin changes how quickly each fiber can send an action potential

thyroid hormones = tyrosine derived + iodine atoms. What kind of hormone category are they likely to fall under?

AA-derivative hormones. Derived from an AA with a few additional modifications.

Compare the Activation-Synthesis Dream Hypothesis to the Freud Dream Theory.

Activation-Synthesis: In REM sleep, the brainstem is active and the cerebral cortex will later make sense of it. Dreams are a result of random firing in the brain. Under this hypothesis, dreams have no prominent meaning. Freud: The manifest content ("plot") has a hidden meaning (the latent content) to be interpreted to resolve hidden problems.

Describe the disorder of Agnosia

Agnosia is a brain disorder caused usually by damage to the brain, such as stroke, multiple sclerosis, etc. It is the loss of ability to recognize objects, people or sounds (usually only 1 of the 3)

There are several types of anxiety disorders. Describe Agoraphobia

Agoraphobia refers to an anxiety disorder characterized by a fear of being in places or situations individuals find hard to escape

Match the following states of consciousness with their key characteristics (1) Alert (2) Daydreaming (3) Drowsy (4) Sleep (A) Seemingly purposeless flow of thoughts come to mind (B) Essentially unconscious, little awareness of world (C) Almost asleep, but semi-aware of the world (D) Awake and able to think

Alert - D Daydreaming - A Drowsy - C Sleep - B

Describe some of the major symptoms of stress induced depression (or depression in general): - Anhedonia - "Learned Helplessness"

Anhedonia - inability to feel pleasure "learned helplessness" - feeling of lack of control over life

If there were no cochlear fluid, would audition be possible? How would the hair cells be affected?

Any vibrations from the stapes to the oval window would move air, whose movement would have a significantly smaller effect on hair cells; thus, they wouldn't reach threshold, making audition impossible.

Compare the differences in symptoms between patients with Broca's aphasia, Wernicke's aphasia, and conductive aphasia.

Aphasia is an impairment of language, due to injury. Broca's aphasia - damage to the Broca would result in impaired or slurred speech. Wernicke's aphasia - difficulty or inability to understand words + producing nonsense words Conductive (or global) aphasia - damage to the arcuate fasciculus resulting in loss of connection between the Broca's and Wernicke's areas - they are both in tact and hence so is speech production and language comprehension but the patient is unable to repeat something said to him. But with enough time patients will correctly get their point across via synaptic plasticity.

There is a structure in the accessory olfactory epithelium called the vomeronasal system, which has specific cells that respond to pheromones. What are these cells? (Hint: there are two types)

Basal cells and apical cells

A week after his house was broken into in the night, James has a panic attack while trying to fall asleep in his bed his first night back home. Could he be diagnosed with a panic disorder?

Based on this information, no. James had an isolated panic attack that was somewhat justified by the situation, not a history of panic attacks for less warranted reasons.

In addition to the cochlea, the brain uses __________ to distinguish between high and low frequencies.

Basilar tuning. The basilar membrane inside the cochlea contains many hair cells

Jerry is extremely poor and worked his whole life as a custodian. He does, however, feel like he was able to choose his own destiny throughout his life. How do the terms Power, Social Class, and Prestige apply to Jerry?

Because Jerry feels like he is in control and he has power to do things, he has a high amount of Power from his perspective. Jerry is poor, though, which means he is of a low social class [at least in a society where social class is based on income] Jerry was a custodian, which is not a very prestigious job.

Explain the difference between Belief Perseverance and Confirmation Bias.

Belief Perseverance is maintaining old, incorrect beliefs by ignoring or rationalizing against the dis-confirming facts presented to you. Confirmation Bias is when you intentionally seek out only facts to support your current opinion/decision.

Differentiate between bipolar I and bipolar II disorders

Bipolar I - manic episodes with or without depressive episodes Bipolar II - hypomania with at least one major depressive episode

Describe the amplification ability of secondary messengers from peptide hormones and catecholamines

Both peptide hormones and catecholamines cannot cross the cell membrane. The binding of the hormone to the G-protein coupled receptor initiates a series of reactions inside the cell (signalling cascade) which can lead to bulk production, or amplification, of the secondary messenger (e.g. cAMP).

For two siblings, with genetic predispositions to depression, one child ends up good looking and receives more attention and affection, and the other ends up ugly and thereby is more isolated from society. What would account for the second child becoming depressed, nature or nurture?

Both. Nature in the sense of genetic predisposition as well as unfortunate aesthetic genes, nurture in the sense that the people in the surrounding environment did not give the child the same sort of attention and privileges.

Similarity has been found to be a huge determinant and factor in attraction. Does this only entail physical or mental attraction or both?

Both. We are more likely to be attracted to someone who shares either or both features - we would be more likely to form friendships and/or relationships with others who have similar attitudes, intelligence, education, height, age, religion, appearance and socioeconomic status.

When you experience pleasure, the common neurotransmitter that the brain releases is ____________, which is produced in the _______________ of the midbrain. (A) Serotonin, Pons (B) Dopamine, Limbic System (C) Dopamine, Ventral Tegmental Area (D) Serotonin, Ventral Tegmental Area

C) Dopamine, Ventral Tegmental Area (MESOLIMBIC REWARD SYSTEM)

Later psychoanalytical theories have given more emphasis to interpersonal, sociological and cultural influences. Carl Jung preferred to think of libido as psychic energy in general. How did Jung define the ego and elements of the theory?

Carl Jung identified the ego as the conscious mind. He divided the unconscious mind into the *personal unconscious* (=Fruedian unsconscious) and the *collective unconscious*, which is a powerful system shared among all humans, considered to be a residue of experiences of early ancestors.

Your wife asks you to go to the grocery store and get a variety of items. She tells you what to get, and you have to remember them. Would rote rehearsal or chunking be a better method of remembering?

Chunking. Chunking (or clustering) refers to a memory trick of taking individual elements of a large list and grouping them into categories with related meaning. Thus, you could group fruits, carbs, proteins and baking goods.

Describe the Zimbardo (Stanford) Prison Study.

College kids taken into a basement. Some were randomly made guards and the others made prisoners. The guards and prisoners started acting in accordance with their role so much so that the study had to be stopped. Zimbardo's Girlfriend had to convince Zimbardo to stop the study.

Describe complex behaviours

Complex behaviours are a category somewhat between innate and learned behaviours - sort of like a spectrum. Think insects: an insect has an innate ability to fly, but through learning it becomes more efficient at flying (adaptive improvement).

What type of light do cones detect?

Cones detect colour and detail. There are three types, named for their wavelengths - short cones (S, blue light), medium cones (M, green light) and long cones (L, red light)

What is the phenomenon of confabulation?

Confabulation - process of creating vivid but fabricated memories. It is thought of to be a ways for the brain to fill gaps of missing memories "False memories" are also a form of confabulation - they didn't happen but could be expressed with strong confidence.

Describe the role of Cost Selection in Altruism.

Cost Selection would indicate that people act altruistically in order to be seen as someone with resources (and worthy of respect) in a community.

David lives in a rural community. He has a hard time focusing in school and finds that he is sad sometimes when his friends don't want to play. If the healthcare system in David's small community underwent Medicalization, how do you think David's life might be impacted?

David would likely be given medications to deal with his problems as his problems would now be seen as illnesses. He would likely be given ADD medication for his inability to focus and Depression medication for his sadness episodes.

Describe how Deindividuation may help explain the Bystander Effect.

Deindividuation is the idea that when you are in a group, you feel less identifiable as an individual. For example, perhaps you don't help the person in trouble because you feel less identifiable as someone to blame for not helping.

Cluster C (anxious/fearful) is defined by three types of personality disorder. Describe dependent personality disorder

Dependent personality disorder - characterized by need for reassurance - tend to remain dependent on one or more specific persons.

What are the physiological effects of Depressants? What are some examples?

Depressants are drugs that lower the body's basic functions and neural activity, i.e. decreased CNS activity, reaction time, processing speed, heart rate and blood pressure. E.g. benzos, barbituates, alcohol

What are the four classes of psychoactive drugs?

Depressants, stimulants, opiates, hallucinogens

Describe Dissociative Amnesia

Dissociative amnesia is a dissociative disorder characterized by the inability to recall past experiences - often linked to trauma. Individuals with the disorder may experience a state of dissociative fugue, where they suddenly wander away from home and responsibilities and even assume a new identity.

Differentiate between drive-reduction theory and the theory that motivation is based on needs.

Drive-reduction theory explains that motivation is based on the goal to eliminate uncomfortable stress. Need based theories state that motivation is explained by how we allocate energy and resources to satisfy our needs.

Why might people who act more altruistically toward others be more likely to also have a high level of empathy?

Empathy is one's ability to see things from a another's perspective and feel sorry for someone who is going through a hard time. People who act altruistically likely do so because they feel sorry for someone else and see that person's heartache from that person's perspective. This is known as the *Empathy-altruism Hypothesis*.

The adrenal medulla is responsible for production of sympathetic hormones epinephrine and norepinephrine (catecholamines) - specialized nerve cells in the medulla allow for direct release of these hormones into circulatory system. What do each do?

Epinephrine increases breakdown of glycogen -> glucose (glycogenolysis) and increases basal metabolic rate. Both hormones increase heart rate, dilate bronchi, and alter blood flow to certain internal organs. In sympathetic response- vasodilation of blood vessels to heart, lungs, skeletal muscle and brain to increase blood flow to these organs; vasoconstriction of vessels to decrease blood flow to the gut, kidneys and skin

Differentiate hypo- and hyperglycemia. Also differentiate Type I and II diabetes.

Excess insulin causes hypoglycemia - low blood glucose conc., and underproduction, insufficient secretion or insensitivity to insulin can cause diabetes. Hyperglycemia is characterized by excessive glucose in the body - in the kidneys, it will overwhelm nephrons and be excreted in urine - causes polyuria (excess urination) and polydipsia (excess thirst) Type I (insulin-dependent) diabetes is caused by autoimmune destruction of beta-cells, causing little to no insulin production Type II (non-insulin dependent) diabetes is due to receptor level resistance to insulin - partially inherited partially due to diet and environment.

Types of population pyramids

Expansive population pyramid - characterized by high fertility rates and lower than average life expectancy rates Constrictive population pyramids - countries with more elderly population and shrinking, less of a young cohort Stationary population pyramids - low birth rates and high quality of life - developed nations

True or false: Functionalism and conflict theory stem from a microsociological perspective

FALSE: Functionalism and conflict theory stem from a MACROSOCIOLOGICAL perspective Symbolic interactionism falls under microsociological perspective

True or false: Stereotypes are always used in negative and inappropriate situations.

FALSE: Stereotypes are fundamentally necessary to everyday life in order to make sense of a complex world by categorizing and systemizing information. Only when it is used in prejudice and discrimination is it used negatively.

True or false: with age, fluid intelligence tends to increase or remain stable, while crystallized intelligence decreases dramatically

FALSE: crystallized intelligence remains stable or increases with age, while fluid intelligence tends to decrease

True or false: studies show that exposure to films with violent behaviour or violent video games have no influence on aggression.

FALSE: studies show that exposure to films with violent behaviour or violent video games HAVE influence on aggression.

True or false: the id is responsible for our system of right or wrong, which substitudes for parental rewards and punishments.

FALSE: the *superego* is responsible for our system of right or wrong, which substitudes for parental rewards and punishments.

True or false: variable-interval schedules are the fastest for learning a new behaviour and also the most resistant to extinction.

FALSE: variable-ratio schedules are the fastest for learning a new behaviour and also the most resistant to extinction. Think about gambling being variable ratio schedule and how effective and addicting it is

True or False? The larger the group, the more likely you are to conform.

False. Conformity is strongest for groups of 3 to 5.

True or false: Sodium channels in the rod cells require GMP to be bound in order for them to be open and allow flow of Na+ ions into the cell.

False. They require cGMP to be bound; conversion of cGMP to GMP by phosphodiesterase (PDE) causes a decline in cGMP concentration, closing the sodium channels.

What might happen in adults fixated in the Oedipal (phallic) stage?

Fixated adults who never resolve the Oedipus complex may exhibit sexual dysfunction.

Compare Frued's Theory of Psychosexual Development with Erikson's Theory of Psychosocial Development.

Frued's Theory states that personality is basically fully developed by the age of 5 and is centered around sexual influences. Erikson focuses on the role of culture and society and believes that personality will continue to change and develop throughout one's lifespan. His theory has 8 stages, and failure at resolving conflict in one does not mean mastery of each is required to move onto the next.

Motor skills can be broken down to two skills - differentiate between gross and fine motor skills

Gross motor skills are movements of large muscle groups and whole body motion (sitting, crawling, walking) - develop earlier Fine motor skills are movements of smaller muscles like fingers and toes + delicate movements and tracking motion

Group interactions can shape an outcome. What is group polarization?

Group polarization is a phenomenon in which group decision making amplifies original opinions of group members. Group ideas have a tendency to polarize towards an extreme, more so than individual ideas or inclinations of members within a group. Some factors: -All views do not have equal influence - viewpoint shared by majority -Arguments made tend to favour popular view, criticism is directed to minority view (confirmation bias)

A long dopamine-4-receptor gene is associated with being a thrill-seeker. Levi has a long Dopamine-4-receptor gene. He also hates sky diving, gambling, and even stays home on Halloween to avoid the thrill. He is very calm and not interested in such activities. Do Levi's actions confirm or disconfirm the Biologic Theory?

Having a longer Dopamine-4-receptor gene is associated with being a thrill-seeker, but just because you have a genetic disposition for something doesn't mean you will certainly act that way. For this reason, Levi's actions neither confirm nor disconfirm the Biologic Theory.

Describe how healthcare and medicine function as a social institution

Healthcare and medicine aim at maintaining/improving health status of the individual, family, community and society. Key goals include: increased access to health care, reduced costs, increased education, disease prevention, primary-care physician association for patients, life course approach to health (history of patients health), and decreased paternalism (doctor knows best attitude) Sociologists investigate medical ethics - key tenets are *beneficence* (responsibility of physician to act in patients best interest), *nonmaleficence* (avoid potential harm of patients), *respect patient autonomy* (respecting patients healthcare decisions unless psychiatric illnesses involved) and *justice* (treat similar patients with similar care + distribute resources fairly).

Hypnosis is an induced state, what does this mean exactly?

Hypnosis begins with hypnotic induction - the hypnotist seeks to relax the subject and increase their level of concentration.

Describe hypomania

Hypomania typically does not significantly impair functioning nor are there psychotic features - though individual may be more energetic and optimistic

What are the main structures involved in the limbic system?

Hypothalamus, Amygdala, Thalamus, Hippocampus

Generally define "intelligence", and what IQ stands for.

IQ = Intelligence Quotient Generally, intelligence gives the abilities to learn from experience, solve problems and adapt to new situations.

Differentiate between manifest and latent functions

If an action is intended to help some part of a system, it is a *manifest function*. Unintended positive consequences of manifest functions are *latent functions*. E.g. manifest functions of annual doctor meetings is to educate a group of physicians sharing research findings and setting goals. Latent function would be stronger interpersonal relations between those physicians and sense of group identity.

What was the problem with Broadbents theory?

If everything else were filtered out when selectively attending to one thing, how do you suddenly change focus when your name is called out in a group? (Cocktail party effect)

Describe the Bobo Doll Experiment. What was the major conclusion of this experiment?

Kids observed a man beating up a Bobo Doll. After the children were tasked with putting together a frustrating puzzle, they were given the opportunity to play with a room full of toys. Several children copied the man's actions and proceeded to beat up the Bobo Doll. The conclusion was that children can learn through observation.

Describe the role of Kin Selection in Altruism.

Kin Selection would indicate that we tend to favor and act more altruistically toward those we are related to.

First Impressions are "Long, Strong, and Easy to Build Upon." What do each of these mean?

Long - First impressions last a long time. Strong - First Impressions are hard to change or erase. Easy to Build Upon - People tend to look for info that supports their first impression.

William Masters and Virginia Johnson noticed certain factors increase sexual arousal - what did they find?

Masters and Johnson noted that physiologically, sexual arousal and motivation was linked to secretion of estrogens, progesterone and androgens, with a strong correlation between hormone conc and sexual desire. Another biological factor was smell - certain odors were shown to increase sexual desire and activity Cultural factors played a role in influencing pleasure and the interpretation of pleasure, as does cognition (sexual videos study). Culture and society also influenced what was deemed appropriate.

Describe the term and what kind of cue it is: motion parallax

Motion parallax is relative motion - it is a monocular cue. When looking at something far while moving, it appears to move slow, whereas something nearby or adjacent seems to be moving fast.

The three types of TrpV1 fibers, A-B, A-D, and C vary in size due to different levels of myelination. How does their relative myelination affect their function?

Myelin is very important for conduction as it increases the number of ions in the axons to create an action potential. Ions in the axons cannot escape through myelinated areas, and so the more myelinated the fiber, the faster the conduction speed. Furthermore, the more myelinated, the wider the axon = less resistance = faster conduction

There are several types of anxiety disorders. Describe Post-Traumatic Stress Disorder (PTSD), and the sort of symptoms it has (intrusion, avoidance, negative-cognitive, arousal).

PTSD is a stress anxiety disorder that occurs after experiencing or witnessing a traumatic event. Unlike other anxiety disorders, it usually has a trigger to onset the disorder. PTSD has intrusion, avoidance and negative cognitive and arousal symptoms: Intrusion symptoms include recurrent reliving the event through flashbacks, nightmares and prolonged distress. Avoidance symptoms include deliberate attempts to avoid memories, people, places, objects etc. associated with the trauma. Negative cognitive symptoms include inability to recall the key features of the incident, negative mood and world views, feeling distanced. Arousal symptoms include increased startle response, irritability, anxiety, destructive or reckless behaviour, insomnia.

Mary is nervous about marrying a really strong guy with a wide face. How might you counsel her in this decision? Is her thinking reasonable?

People with higher testosterone levels tend to have wider faces and stronger muscular features. Higher testosterone is also associated with higher levels of aggression. Maybe you could tell her that there may be an association, but that doesn't necessarily mean causation. You might tell her that it is good for her to be cautious, but she might need to be a little more open minded.

Personality disorders are patterns of behaviour that are inflexible and maladaptive, causing distress or impaired functioning in cognition, emotions, interpersonal functioning or impulse control. What does it mean to say personality disorders are ego-syntonic?

Personality disorders are ego-syntonic, meaning that the individual perceives their behaviour as the correct, normal way and is harmonious with their goals. It is in contrast to ego-dystonic

What equation can be used to determine the Population Growth Rate?

Population Growth Rate = (Birth Rate + Immigration Rate) - (Mortality Rate + Emmigration Rate)

What is the main factor that causes a society to change from one in which individuals are independent and self-sufficient to one in which individuals work together and divide labor?

Population growth within a small space leads to issues that must be reconciled by working together.

What are problems in the brain area related to schizophrenia relating to the prefrontal cortex and hippocampus?

Prefrontal cortex - dysfunction in the prefrontal cortex Hippocampus - schizophrenics often have smaller hippocampus's

What are examples of primary and secondary sex characteristics?

Primary - genetilia - testis, ovaries Secondary - voice changes, body hair, breast and hip development, pubic and underarm hair

One of the senses not often talked about in school is the sense of proprioception. Describe it.

Proprioception is defined as our ability to sense where our body is in space - our sense of position and balance.

Parts of the eye: pupil

Pupil is the opening of the eye which allows light in - it is controlled by the iris via sympathetic division of autonomic nervous system - dilates when it's dark out (sympathetic) so more light can enter light; constricts when its bright out (parasympathetic) so less light enters.

Compare the Self-serving Bias, Fundamental Attribution Error, and Actor-observer Bias.

Self-serving Bias - where you attribute your success to yourself but your failures to the external environment. (always serving your self basically). It is a mechanism to protect your own self-esteem. *Self enhancement* is the need to maintain self-worth, which can be accomplished through the self serving bias. Fundamental attribution error - tendency to attribute people's negative behavior to them personally rather than considering other circumstances/environment. Actor-observer Bias - tendency to attribute your faults to outside factors but other's faults to their personality/personally.

What is sensory adaptation?

Sensory adaptation is a down regulation of sensory receptors over time to a change in stimulus

How do Social Institutions relate to Society? Describe how the following Institutions' roles may have had an impact on your day yesterday. -Legal system -Education system -Healthcare system

Social Institutions are established sets of norms and subsystems that support a society by meeting its needs. If society was a bike, Social Institutions are like the tires, pedals, bike frame, etc. The Legal System may have protected your neighborhood from danger. The Education System may have allowed you to go to class. The Healthcare System may have supported you if you were sick.

Describe how religion might be viewed as a Social Fact.

Social facts originate in an institution or culture and have an influence on individuals. We only notice it is there when we push against it, and it remains there despite people being born and dying. In this context, religion is clearly a social fact because of how it affects behaviors and attitudes of individuals.

What is the benefit of basilar tuning?

Specific frequencies send specific signals allowing the brain to distinguish between different sounds of high or low frequencies. Otherwise, if all hair cells responded to all frequencies, one large signal would be sent to brain without ability to distinguish.

True or false: in rural areas, larger families were more ideal for labour purposes, while in urban smaller families were more ideal for consumption purposes

TRUE

Describe the James-Lange theory of emotion. (hint: think functionalist, William James = founder of functionalism)

The *James-Lange theory of emotion* states that a stimulus first results from physiological arousal, which leads to a secondary response in which the emotion is labelled. Peripheral organs receive signals and respond, and that response is labelled as an emotion by the brain. E.g. "I must be angry because my heart rate is elevated, skin is hot, blood pressure is high"

Describe the Lazarus theory of emotion. How is it different from the Schachter-Singer theory?

The *Lazarus theory of emotion* proposed that the experience of emotion depends on how the experience is cognitively appraised (where you label the situation). It is different from the Schachter-Singer theory because once an event happens, the situation is labelled by the brain, and depending on that label comes the emotional response. So if the situation is labelled as bad or frightful, the emotion experience thereafter is negative and a physiological response occurs (simultaneously).

What is the general intelligence (G factor) theory?

The *general intelligence (G factor) theory* proposes there is one underlying intelligence accounting for our mental abilities and capabilities, and ability in one area = higher ability in another

What is the purpose of the otolithic organs (utrical and saccule)?

The *otolithic organs (utrical and saccule)* help determine the acceleration and head positioning. They have calcium carbonate crystals attached to hair cells. These crystals pull on the hair cells, depending on our head positioning, which results in an action potential, allowing us to detect acceleration and position.

*Social perception* (social cognition) provides tools to make judgements and impressions - describe the three components of social perception (perceiver, target, situation)

The *perceiver* is influenced by experiences, motives and emotional state. Past experiences affect attitudes which set expectations of future events and experiences. Motives influence what info is deemed important vs. negligible, and emotional state flavours interpretation of the event. The *target* refers to the person about which perception is being made. The *situation* is the given social context which is very important in providing information and developing perception of the perceiver.

One model of social perception focuses on selection of cues to form interpretations that are consistent over time. Describe the primacy and recency effects of *impression bias*.

The *primacy effect* of impression bias refers to how first impressions are more powerful than subsequent impressions. The *recency effect* suggests the most recent information about something/someone is the most important in forming our impressions.

How does the *Yerkes-Dodson law of social facilitation* describe the phenomenon?

The Yerkes-Dodson law of social facilitation states that in the presence of others, arousal is raised significantly in a setting of perceived evaluation, which enhances the ability to perform simple tasks and hinders that to perform complex tasks.

Describe the behavioural theory of personality.

The behavioural theory states that personality is the result of the interaction between an individual and their environment.

Describe Jung's *collective unconscious* and what elements it has

The building blocks of the collective unconscious are images of common experiences, referred to as *archetypes* - examples of archetypes: The *persona* - a personality mask we wear in public that is adaptive to social interactions The *anima* (feminine) and *animus* (masculine) - referring to sex-inappropriate qualities - feminine qualities in males, masculine qualities in females The *shadow* - appearance of socially reprehensible and unpleasant thoughts, feelings, emotions in our consciousness.

When at a rock concert, your friend whispers something in your ear and you have no clue on Earth what he said. However, when he whispered the same thing the next day in the library, you had no issues understanding what he said. What concept explains this?

The concept of JND (just noticeable difference). By Weber's Law (ΔI/I=K), the threshold for the JND would be very different compared to the surroundings of the two environments. Relationship between intensity and incremental threshold is linear.

Describe Frued's element of the superego

The ego and superego both have conscious and unconscious elements. The superego also aims to achieve desires, but unlike the id (which desires basic needs), the superego has refined and focused desires on the ideal self - personality perfectionist that judges our actions and responds with pride or guilt. Two elements: the *conscience* - collection of punishable improper actions from childhood; the *ego-ideal* - proper actions rewarded from childhood. These eventually transform into a system of right or wrong.

The self-concept has two aspects: the existential and categorical selfs. Describe the two

The existential self - most basic part of self-concept - a sense of being unique, separate and distinct entities from others + understanding and awareness that the self is constant The categorical self - recognition that we have properties similar to other entities/people - categorizing by age, gender, skills, size. Unlike the existential self, the categorical self can change.

The 1947 study by Kenneth and Mamie Clark explore ethnic self-concepts among white and black children with dolls - describe the experiment and its findings

The experiment showed these children a black doll and a white doll and asked how the child preferred. Most of both white and black kids preferred the white doll, which highlighted the negative effects of racism and minority group status. However, subsequent research showed that black children hold a more positive view of their own ethnicity, which may represent societal changes at large.

Give an example of a Fixed Action Pattern, Migration, and Circadian Rhythm as it might apply to a fish.

The fish may swim in circles when it is trying to attract a mate. This would be an example of a Fixed Action Pattern. The fish may swim upstream to lay its eggs at a certain time of the year. This would be an example of migration. The fish may consistently sleep at night as opposed to the daytime, which would be an example of Circadian Rhythm.

Describe the hierarchal model of semantic memories and spreading activation.

The hierarchal model (which is the old model) was a simplistic hierarchal network of things into general categories.

What is the function of the hippocampus in the limbic system?

The hippocampus works to form new memories - converts short-term memories into long-term memories

There are a few theories that attempt to explain how attitudes influence behaviour. Describe the Learning theory

The learning theory of attitudes states that our attitudes are developed from different forms of learning - can be influenced by direct contact, direct instruction from others, subjective norms, and/or classical, operant conditioning or observational learning.

Talk about the parathyroid gland, which consists of 4 pea-sized structures on the posterior surface of the thyroid, and its hormone.

The parathyroid gland produces PTH (parathyroid hormone) - serves as antagonist hormone to calcitonin. The overall effect of PTH is a significant increase in blood calcium levels with little effect on phosphorus (phosphorus homeostasis).

How does the phototransduction cascade affect rod cells, the downstream bipolar and ganglion cells, and the optic nerve?

The phototransduction cascade deactivates rod cells that typically inhibit downstream cells. In response, a bipolar cell is activated, which activates a retinal ganglion cell that sends signals to the optic nerve, which finally leads to processing in the Brain. It is crucial to realize that rod cells in their normal state INHIBIT downstream signalling, and thus retinal bipolar and ganglion cells are deactivated until light hits the rod cells to deactivate them, allowing downstream signalling.

Before schizophrenia is diagnosed, a patient often goes through a phase characterized by poor adjustment called the Prodromal phase - describe this phase

The prodromal phase is exemplified by clear evidence of deterioration, social withdrawal and isolation, impairment of role functioning, peculiar behaviour, unusual experiences, and inappropriate affect.

Kohlberg organized moral reasoning based on participants' responses to hypothetical moral dilemmas into 6 stages ranging from concrete to the abstract. Describe the conventional phase in terms of the [[Heinz dilemma (stealing an expensive drug to save his wife)]]

The second phase is *conventional morality* and is based on understanding + accepting social rules - develops in early adolescence, when individuals begin to see themselves in terms of relationships to others. Stage 3 (*conformity*) emphasizes the orientation in which we seek approval from others (/should not steal because it's wrong/) Stage 4 (*law and order*) maintains the social order in the highest regard (/if everyone who can't afford stole, the products might not be able to be made by the businesses/)

If a salt receptor was accidentally expressed and activated in a sweet cell, would the sensed taste in the brain be salty or sweet? Why?

The sensed taste would be sweet. If the salty receptor in a sweet cell was accidentally expressed, it would trigger a cascade effect which would still fire an action potential up an axon leading to the designated area in the brain. By the Labelled Lines model, the axons do not mix, and thus the synapse in the brain would still be in the area that detects sweet tastes - thus, the taste would be sweet. Type of cell determines the taste sensed, not the receptor

What is the sleep cycle? How does it go typically?

The sleep cycle refers to a single progression through the stages of sleep - the makeup of sleep cycle changes through the night, as early on SWS (slow wave sleep) predominates as the brain falls into deep sleep and then more wakeful states. Later at night, REM sleep predominates. The cycle goes from N1 -> N2 -> N3 -> N2 -> REM -> back to N1 and repeat.

What does the social interactionist theory of language development state?

The social interactionist theory accounts for interplay between biological and social processes in language development. States children's have a desire to communicate with others, which drives their language development as brain circuits are reinforced through these interactions.

Kohlberg organized moral reasoning based on participants' responses to hypothetical moral dilemmas into 6 stages ranging from concrete to the abstract. Describe the postconventional phase in terms of the [[Heinz dilemma (stealing an expensive drug to save his wife)]]

The third phase is the *postconventional morality* where reasoning is more abstract and by Kohlberg's explanation, not attainable by everyone. Phase is based on social rules that may conflict with laws. Stage 5 (*social contract*) views morals as conventions for the greater good of society, with reasoning focused on individual rights (/everyone has the right to live; businesses have the right to profit/) Stage 6 (*universal human ethics*) is the final stage which reasons that decisions should be made in consideration of abstract principles (/wrong to hold life for ransom/)

Depressants and stimulants seem to have opposite effects. Why don't combining stimulants and depressants cancel each other out? (Hint: think mechanistically)

They do not work on the same Neurotransmitters or neurochemical pathways, so effects on both ends of the spectrum are felt.

Describe Piaget's sensorimotor stage

This stage comes where the child is an *infant ~0-2 years old* - two different types of circular reactions referring to sensorimotor functions: *Primary circular reactions* are repitition of body movements that occus by chance, e.g. thumb sucking; *secondary circular reactions* are those when manipulation is focused on something outside the body, like toys. Through this stage the idea of *object permanence* develops, which is the understanding that objects continue to exist when outside of view

Broadbent's early selection theory

This theory's idea was that all the information from your environment goes into your sensory register which stores all sensory input you get (e.g. words, sirens, etc.) - then this input gets transferred to the selective filter which identifies what its supposed to be attending to through basic physical cues (voice, pitch, speed, accent etc.) and everything else is filtered out. Perceptual processes assign meaning to the words/noise/etc. Then, cognitive processes can occur.

How we sense our five senses boils down to one similar mechanism pattern

Transduction: SENSORY CELLS translating chemical, electromagnetic and mechanical stimuli into ACTION POTENTIALS that our NERVOUS SYSTEM can make sense of

True or False? Unlike innate behaviors, learned behaviors are adaptable, meaning that these behaviors can be altered to better suit the environment.

True

True or false: extreme sleep deprivation can lead to psychosis

True

True or false: there is a fifth EEG wave that corresponds to REM sleep

True

True or False? A downstream hormone could inhibit multiple upstream endocrine glands, and this redundancy shows the importance of feedback inhibition.

True. A downstream hormone could inhibit multiple upstream endocrine glands, and this redundancy shows the importance of feedback control.

True or false? An example of accommodation would be introducing a foreigner to darts as a competitive sport, and the foreigner altering their Schema related to sports to include a less physically involved sport.

True. Accommodation is modifying an existing schema to incorporate new information.

True or false: aggression is influenced by biological components, social/psychological/situational factors, as well as exposure to violent behaviour.

True. Aggression is influenced by all three: (1) biological factors; (2) psychological and situational factors; (3) exposure to violent behaviour.

True or False? Globalization refers to the expansion of global interconnectedness.

True. Globalization refers to the expansion of global interconnectedness.

True or false: in most cases, individuals with dissociative identity disorders have suffered of childhood abuse, either physical or sexual.

True. Most cases - individuals with DID have suffered of severe physical or sexual abuse.

True or false: PTH increases bone resorption to release Ca and Ph by increasing osteoclast activity and decreasing osteoblast activity.

True. Osteoblasts use up calcium to make bones; bone resorption is the breakdown of bone matrix to release Ca and Ph, which is courtesy the activity of osteoclasts.

True or false: Oxytocin is different from other hormones in that it has a positive feedback loop (instead of negative)

True. Oxytocin has a positive feedback effect - released to promote uterus contractions as well as milk production by mammary glands - definitive endpoint exists, usually delivery.

True or false: Insomnia can lead to dependence on sleep medication

True. People who take medication will develop a tolerance to it, causing the need for more and thereby dependence on the medication.

True or false: disruption of SWS and REM sleep can lead to diminished memory

True. Sleep deprivation also causes drop in cognitive performance and negatively affects mood, problem solving and motor skills.

True or false: gene regulation states that ~95% of our genes aren't even protein encoding genes, but rather regulators for activation

True: gene regulation states that ~95% of our genes aren't even protein encoding genes, but rather regulators for activation Sort of like "if we experience..(sugar consumption)...then we activate and encode..(insulin)..."

Frued's psychoanalytical theory has heavy basis on unconscious desires and libido - describe these two.

Unconscious desires influence our behaviours, along with childhood experiences. Our personalities have memories, beliefs, urges and drives we aren't aware of, that make up this unconscious. Libido refers to natural energy source that fuels the mechanisms of the mind - it is fixated at certain psychosexual stages of development.

Sensory adaptaton by your sight senses when it's very dark out - what kind of regulation is this?

Up regulation. When it's dark out, pupils dilate, and rods and cones start synthesizing light sensitive molecules.

How does priming affect selective attention?

We attend to information we have been primed for, even if not consciously thought about. For example, after eating, you would be primed to recognize food words in a word scramble. This would also explain the Cocktail Party Effect, as we'd have been primed to respond to our names.

Differentiate between weak and strong social constructionism and what they say about brute/institutional facts.

Weak social constructionism - social constructs are dependent on brute facts (basic and fundamental - don't rely on other facts) and institutional facts (which are created by social conventions and do rely on other facts). Strong social constructionism - whole of reality is based on language and social habits - all knowledge is a social construct and there are no brute facts. Everything is socially constructed.

What is the purpose of Weber's Law?

Weber noticed that the ratio of the increment threshold to the background intensity is constant. Weber's law quantified the JND for tactile and auditory sensations.

How is a pressurized sound wave formed when someone claps their hands?

When one claps their hands, a number of air molecules that were in a more open space are suddenly compressed into confined space between the palms. With less space to move, this compression forces them to be pressurized - to relieve this pressure, air molecules try to escape, which results in areas of high and low pressure = creating pressurized sound waves

When you drop a baby, it will throw its arms up in the air instinctively. What is this an example of?

When you drop a baby, it will throw its arms up in the air instinctively. This is an example of a reflex. Specifically, the Moro reflex.

What is selective attention?

When you exercise your attention to focus on only one thing - can be thought of as a flashlight beam on some area of interest in your environment, with things around it being "dimmer"

Describe attention

When you pay attention to something, you focus on it exclusively relative to the environment around

On a very foggy day, a driver can have trouble seeing the traffic light. How could this be an example of Signal Detection Theory?

With thick fog, the driver will guess when the light is green (a decision made with uncertainty).

For months, Alterrique appeared to be depressed. However, this episode ends and Alterrique has extreme self-esteem and optimism, but also shows risky behaviors. Could Alterrique have been misdiagnosed and suffer from Bipolar Disorder?

Yes, Alterrique's behaviors after the depressive episode are characteristic of mania. Displaying both manic and depressive episodes is characteristic of bipolar disorder.

Signal Detection Theory helps us make decisions in situations of: (A) Certainty (B) Uncertainty (C) Audition

(B) Uncertainty Signal Detection Theory helps us with "decision making with uncertainty". It answers the question of "at what point is a signal strong enough to notice?"

Oedipus Rex kills his father and marries his mother. People are outraged at Oedipus because he committed incest. Which type of Norm does this example exemplify? (A) Taboo (B) More (C) Folkway (D) Law

(A) Taboo Taboos are based upon the most highly-cherished values in a society such as chastity as is the case with this example.

Fidget Spinners are all the rage among middle schoolers at the moment. They will most likely face a steep decline in popularity. Which type of Collective Behavior is best exhibited by this example? (A) Mass Hysteria (B) Fad (C) Riot (D) Rebellion

(B) Fad Fads are widespread, popular actions that often rise and decrease in popularity extremely quickly.

Which of the following is not a diagnostic tool for diagnosing schizophrenia? (A) Checking for elevated levels of Dopamine (B) Hearing voices (C) Brain scans (D) Patient History

(B) Hearing voices Hearing voices is a common symptom of schizophrenia, not a diagnostic tool.

What is the relationship between the frequency and wavelength of a sound wave? (A) Proportional (B) Inverse (C) Linear (D) Exponential

(B) Inverse

___________________ is the process by which an individuals or groups behaviour and culture begin to resemble that of another group. This can also mean that groups with different cultures begin to merge into one. (A) Culture shock (B) Culture merge (C) Assimilation (D) Multiculturalism

(C) Assimilation *Assimilation* is the process by which an individuals or groups behaviour and culture begin to resemble that of another group. This can also mean that groups with different cultures begin to merge into one.

Which of the following is NOT an example of a Cluster B, known as "dramatic," "emotional," and "erratic" mental disorder? (A) Antisocial (B) Borderline (C) Bipolar (D) Narcisstic

(C) Bipolar

TrpV1 receptors are sensitive to: (A) Temperature (B) Pain (C) Both

(C) Both

Upon which of the following does Personality depend? (A) Inner Experience (B) Behavior (C) Both A and B (D) None of the Above

(C) Both A and B Personality depends on both Inner experience and Behavior.

Which of the following does "Psychosis" include? (A) Hallucinations (B) Delusions (C) Both A and B (D) Neither

(C) Both A and B Psychosis includes both Hallucinations and Delusions.

A girl scout comes to your door and sells you girl scout cookies. The next week, she stops by again, and asks you to donate $1,000 so that she can attend girl scout camp. You agree. Which theory would best explain your willingness to do this? (A) Role-Playing (B) Door-in-the-face (C) Foot-in-the-door (D) Situational Approach

(C) Foot-in-the-Door You likely agreed because you already agreed to support her in a small way before. This enhanced your willingness to support her in a bigger way now. This is perfectly in line with the Foot-in-the-door theory.

Which theorist did not rely on Factor Analysis to develop their theory? (A) Big Five Theorists (B) Hans Eysenck (C) Gordon Allport (D) Raymond Cattel

(C) Gordon Allport Factor analysis is a complex statistical method to narrow down a wider set into a smaller, more representative and significant subset Gordon Allport did not rely on Factor Analysis to develop his theory.

Which of the following global changes occur in the brains of adolescents? I. Increase in myelination II. Increase in synaptic vesicle signaling III. Synaptic pruning (A) II only (B) III only (C) I and III (D) II and III (E) I, II and III

(C) I and III Global brain changes that occur in adolescence are an increase in myelination and synaptic pruning. Increase in myelination makes communications between neurons faster, and synaptic pruning is better for more focused neural communication and use of energy. It could explain why adults think faster than children.

Which stage of sleep is characterized as slow wave sleep and delta waves become present on an EEG? It is often very difficult to wake people up from this stage of sleep. (A) N1 (B) N2 (C) N3 (D) REM

(C) N3 N3 is a stage of sleep that is characterized by slow wave sleep and delta waves become present on an EEG.

Which subcategory of Schizophrenia symptoms involves blunted emotions and loss of interest in activities? (A) None of the following (B) Cognitive (C) Negative (D) Positive

(C) Negative The Negative subcategory of Schizophrenia symptoms includes blunted emotions and loss of interest in activities.

Which of the following is NOT one of the most common focuses of obsessions in OCD? (A) Dirt and Toxins (B) Symmetry (C) Tapping in multiples of a number (D) Accidents and extreme scenarios

(C) Tapping in multiples of a number Option (C) is a compulsion, because it is an action performed. Common Obsessions in OCD are Dirt and Toxins, Symmetry, and Worrying over Accidents and unlikely scenarios.

According to many psychologists, which of the following categories of stressors is the most significant form of stress due to its recurring nature? (A) Significant life change (B) Ambient (C) Catastrophe (D) Daily Hassle

(D) Daily Hassle

If a trait is known to be almost exclusively dependent on environment, which of the following scenarios will have the LEAST variance? (A) A child and their biological parents (B) A child and their adopted parents (C) Monozygotic twins raised separately (D) Dizygotic twins raised together.

(D) Dizygotic twins raised together. Twins that are raised together share as close to identical environments as possible, regardless of being dizygotic or monozygotic. Twins raised separately will have very different environments, and the parent-child generational gap produces even greater environment disparity.

What cues does the body perceive and receive from the different types of somatosensations in order to process information? I. Intensity II. Timing III. Location (A) I only (B) I and II (C) II and III (D) I, II, and III

(D) I, II, and III

Which of the following are abnormalities seen in the nervous system of schizophrenics? I. Diminished tissue size leading to larger ventricles in brain II. Cortical layers of cerebral cortex are disorganized and have less tissue III. The mesocorticolimbic pathway that carries dopamine from VTA to other cortex and limbic system acts irregularly. (A) I only (B) III only (C) I and III (D) I, II, and III

(D) I, II, and III Abnormalities often seen in Schizophrenic patients are larger than normal ventricles in the brain, cortical layers are disorganized, and the mesocorticolimbic pathway that carries dopamine from VTA to other cortex and limbic system acts irregularly.

The olfactory epithelium sends information to the _______________. The __________________ is a specialized part of the olfactory epithelium that sends projections to the accessory olfactory bulb which is then process by the brain. (A) Olfactory bulb, Basal cell (B) Accessory olfactory epithelium, olfactory bulb (C) Olfactory bulb, Vomeronasal system (D) Olfactory bulb, Accessory olfactory epithelium

(D) Olfactory bulb, accessory olfactory epithelium

Of the four main classes of drugs, which class is known to decrease Central Nervous System function, decrease Heart Rate and Blood Pressure, cause relaxation and also act as an analgesic, reducing the perception of pain? (A) Depressants (B) Stimulants (C) Hallucinogens (D) Opiates/Opioids

(D) Opiates/Opioids

Impression Management, introduced by Erving Goffman's Dramaturgical theory, describes how we try to manage the way that: (A) We act (B) Others act (C) We see others (D) Others see us

(D) Others see us Impression Management describes how we try to manage the way that others see us.

Jackson works full time for a non-profit and is paid a reasonable salary. Which type of Organization does Jackson belong to? (A) Coercive Organization (B) Normative Organization (C) Bureaucratic Organization (D) Utilitarian Organization

(D) Utilitarian Organization Because members are paid/reimbursed for their membership, this non-profit would be considered a Utilitarian Organization.

What are criticisms of the world systems theory of globalization?

It is looked at as a fluid model but criticized for being overly focused on economy and forgetting about culture and class struggles.

How are proximity and the mere exposure effect factors in determining physical attraction?

*Proximity* is just being close to someone or something. Studies have shown we are more likely to form friendships and relations with people in the same dorm/class/within a range of proximity as us. Makes sense to be more attracted to someone hot that's next to you than across the globe! *Mere exposure effect* (aka *familiarity effect*) states people prefer stimuli they have been exposed to more frequently; (e.g. acquiring a taste for Diet coke after multiple tastes). In terms of attractiveness, you are more likely to be attracted to a person who you see or interact with more often than another of the same level of relative attractiveness.

Describe the rational choice and exchange theories.

*Rational choice theory* states people take rational decisions based on a mental pros and cons analysis, choosing an option with the best benefit-to-harm ratio when evaluating the potential benefits and costs. Rewards can include money, accolades, honor, prestige, social approval; punishments can include embarrassment, humiliation, sanctions, stigma. *Exchange theory* expands on rational choice theory and focuses on interactions in groups rather than simply independent individual - states people behave with incentive to maximize own rewards and minimize punishments.

Compare role strain, role conflict and role exit

*Role strain* refers to the tensions between multiple requirements and expectations of the same role and status (e.g. being a student and getting everything done). *Role conflict* refers to the tension between roles of two or more statuses, e.g. a doctor and a husband/parent *Role exit* is dropping of one identity for another.

Differentiate between a sect and a cult

A Sect is a breakaway or a revival of an established church. For example, Lutherans may be considered a sect as they broke away from Catholicism to form their own unique sect. A Cult is a radical group (compared to a more established religious group) that is often led by a charismatic leader. These groups often only last for a short period of time. For instance, Heaven's Gate had very unique beliefs and they were led by a charismatic leader named Marshall Applewhite

What is a stressor? What are the four MAJOR categories of stressors?

A stressor is a biological element, external condition of event that leads to a stress response. The four major categories are: Significant life change Catastrophic events Daily hassles Ambient stressors (global and environmental)

What are the body's responses to anger, fear, stress, sadness and happiness in respect to: - Heart rate - Skin temperature - Blood pressure - Heart rate variability

Anger - increased heart rate, skin temperature, decreased heart rate variability, greatest degree of increased dialostic blood pressure Fear - increased heart rate and blood pressure, decreased skin temperature and heart rate variability Stress - increased blood pressure Sadness - decreased blood pressure (same for relaxation) Happiness - decreased heart rate, increased heart rate variability, decreased blood pressure

Calcitonin is produced by C-cells (aka parafollicular cells) - discuss what it does In contrast, talk about the parathyroid gland and it's hormone.

Calcitonin acts to decreased plasma calcium in three ways: 1) Increased Ca excretion from kidneys 2) Decreased Ca absorption from the gut 3) Increased Ca storage in the bone The parathyroid gland produces PTH (parathyroid hormone) - serves as antagonist hormone to calcitonin. The overall effect of PTH is a significant increase in blood calcium levels with little effect on phosphorus (phosphorus homeostasis).

What is daydreaming?

Daydreaming occurs naturally, it is a state of being awake but not entirely aware of the environment and less focus.

Compare the terms delusion and hallucination.

Delusions are fixed false beliefs not explained by a cultural background. Hallucinations are sensory perceptions without stimuli.

In what ways is comparing Depression and a common cold appropriate? In which ways is it inappropriate?

Depression is by far the #1 reason for reaching out to mental health services, so its prevalence in the mental health community is similar to a cold's prevalence. However, this comparison can trivialize depression, since common colds are often recovered from without outside help.

How might the following external factors influence the absolute threshold of how likely an individual is to notice that their spouse is speaking to them while they are studying for the MCAT? - Expectations of stimuli - Experience - Motivation - Alertness

Expectations of stimuli - If you expect your spouse to speak to you while you are studying, you will be more likely to notice when it happens. Experience - If your spouse speaking to you is a common experience, you may be more likely to recognize it. Motivation - If you are motivated to impress your spouse, you may be more likely to pay attention. Alertness - If you are less tired while studying, you will be more likely to perceive that your spouse is speaking to you.

Remaining in stress and overdrive for too long puts your body in a system of overdrive. What are some of the physical effects of stress on metabolism?

Extra or psychosocial cortisol release can lead to excess glucose in the blood stream - can have profound effects on our metabolism and cause diabetes.

Motivation can be described as the purpose/driving force behind action. Can be categorized based on what drives people to act. Differentiate between extrinsic and intrinsic motivation.

Extrinsic motivation - external forces driving motivation - e.g rewards and punishments Intrinsic motivation - internal motivation from within oneself - e.g. interest by a task, or pure enjoyment (me at the gym)

True or False? Individuals tend to act less aggressively in big groups as opposed to when they are all alone.

False. Individuals tend to act MORE aggressively in big groups as opposed to when they are all alone. This might be explained by Deindividuation in which we don't see ourselves as personally identifiable in a large crowd.

True or False? To have an effect, hormones must be very concentrated in the blood (almost as concentrated as White Blood Cells).

False. To have an effect, hormones can be at very low concentrations in the blood (picograms/mL can be enough!)

True or False? In the Asch experiment, when a supporter (one who answers correctly when the others didn't) was added, conformity of the participant increased because they didn't want to look stupid like the supporter.

False. Without a supporter, 37 percent of individuals answered in line with the group 100 percent of the time, but with a supporter, conformity dropped to just 5 percent. The more unanimous a group, the more likely you will be to conform.

True or False? Men live longer than Women on average.

False. Women live longer than men on average.

The Theory of Planned Behavior states that our behavior is determined by how we think about an action before we do it, specifically our implications and intentions. Our intentions are influenced by 3 things - attitudes, subjective norms, and perceived behavior control. Describe how your intention to study for the MCAT might be influenced by these three things.

If you have a negative attitude toward studying ("I hate studying"), others think your studying is a waste of time (subjective norms), and you feel like you can't control how much time you have for studying, you would be less likely to study.

Why is answer choice B wrong in the following question: Two siblings could be born with the same combination of genes that can lead to depression, but the environment only activates those genes for one of the siblings. Which of the following scenarios is this most like? (A) Ben and Chrissy both have a dream to win a reality TV show, but only Ben achieves this dream. (B) Devon and Mike grew up having very different looks, and because Devon was more handsome, he received more positive attention. (C) Ashley and Alan both have a predisposition for panicking under stress, and Alan has a panic attack after he gets in a car accident. (D) Lauren and Desi grew up with very different lifestyles, so that only Lauren enjoys working outdoors.

In this answer choice (B), one sibling ends up receiving more attention and affection from the surroundings and so may end up growing up happier and more optimistic, but this does not suggest any predisposition to depression or any sort of mental disorder, thus assuming the other child would become depressed would not be entirely accurate. If a predisposition was mentioned, then (B) would have more of an option.

Explain why the rats who got shocked when administered sweet water only did not develop an aversion to sweet water?

It is not natural to associate a shock with food poisoning whereas it is natural to associate an upset stomach with food poisoning. This illustrates that we are evolutionarily predisposed to learn certain things.

In terms of Freud's theory of Psychosexual Development, define the following terms: -Libido -Adult Fixation

Libido refers to a natural energy source that fuels the minds' processes. Fixation is when this libido is stuck on one specific focus, instead of successfully advancing through the stages. Vices in the adult life could theoretically be traced to this fixation.

How is light converted into electric signals and sent to the brain?

Light hits the photoreceptors (cones and rods) and causes a conformational change in membrane proteins, which leads to electric signals that go to the brain via the optic nerve

Parts of the eye: cornea

Light passes through the cornea first, which gathers and focuses incoming light. Entering light is bent with an index of refraction = 1.4 (while air = 1.0)

Differentiate between macrosociology and microsociology

Macrosociology is a large scale perspective, looking at big phenomenon that affect large chunks of people, such as societies, civilizations etc. - analyzing large collective for broad social trends Microsociology - focuses on small group interactions and individuals

As humans, males and females tend to be attracted to each other's Sexual Dimorphism. What does this mean?

Males tend to be attracted to feminine females and females tend to be attracted to masculine males.

What is meditation?

Meditation is an induced state of consciousness that is highly dependent on the practitioner and their beliefs. It usually involves the quieting of the mind to achieve a sense of relaxation. In Western traditions it is used for counseling and psychotherapy to provide relief from anxiety or worrying. Causes physiological changes like decreased heart rate and BP

True or false: people who sleep normally after sleep deprivation often experience REM rebound, which is earlier onset and greater duration of REM sleep

True

The retina has two types of photoreceptors in particular- what are they?

Rods and cones

Shi'as and Sunnis are examples of __________ of Islam, which are denominations within a religion that share some beliefs but not others.

Shi'as and Sunnis are examples of *sects* of Islam, which are denominations within a religion that share some beliefs but not others.

There are several types of anxiety disorders. Describe social anxiety disorder

Social anxiety disorder is anxiety due to social situations - fear of being exposed to social situation or performance that may lead to embarrassment.

True or false: people with social support tend to live longer and have less issues with mental health problems

True

Describe the following theories of social movements: - Mass society theory - Relative deprivation theory - Resource Mobilization theory

The *mass society theory* of social movement - skepticism of social movements and their motive resulted in a dark view of social movements, useful only to those who needed refuge and didn't feel belonging in society. Was strong during times of fascism, nazism, Stalinism. The *relative deprivation theory* focused on social movements motivated by those who felt oppressed and deprived of rights (e.g. civil rights movement). The *resource mobilization theory* focuses on factors that help or hinder a social movement, such as gathering group members and resources, money and materials, access to media, political influence etc. A charismatic figure is almost necessary for a social movement (e.g. Martin Luther King in civil rights movement).

One of Jung's archetypes is the self - describe what he means by the self.

The *self* - intersection between the collective unconscious, personal unconscious and conscious mind. This archetype strives for unity.

A child is taking a hearing test. How might the concept of Absolute Threshold of Sensation apply to this situation?

The Absolute Threshold of Sensation is a generally detectable stimulus. It is defined as the minimum intensity of a stimulus needed to detect that stimulus at least 50% of the time. Thus, for a child taking a hearing test, it would be the point at which the child begins to detect the sound in their ear at least 50 percent of the time.

Stanley Milgram was inspired by his background (Jewish parents who immigrated to the US) and by the Holocaust to conduct his experiment regarding obedience. Describe his experiment.

The Milgram experiment was conducted to observe the obedience of participants who were asked to give an individual for answering questions wrong. Individuals did as they were told and administered high level shocks in order to obey the authority, despite having to give up to 'lethal' shocks. It found that stark authority usually overuled peoples morals and people would go to any extent under authority.

The *Rorschach inkblot test* and *thermatic apperception test* both are used to identify individual undesired feelings via projection as a defense mechanism. Describe the tests.

The Rorschach inkblot test - relies on assumption that the client projects his or her unconscious feelings onto the shape Thermatic Apperception test - series of pictures presented to client who is asked to make up a story for each - will elucidate client's own unconscious thoughts and feelings.

What is depth perception?

The ability to perceive the world in 3 dimensions + distance of an object. Depth perception rises from a variety of depth cues (monocular and binocular cues).

What happens when a benzene molecule (or aromatic compound) enters a nasal passage and hits one of the receptors on a nerve projection?

The benzene ring hits the receptor - those sensitive to benzene will trigger a cascade of events and fire an action potential into a particular location in the olfactory bulb called a glomerulus. From there, they synapse into a mitral (or tufted) cell, which projects the signal to the brain.

The olfactory bulb is a bundle of nerves that sends thousands and thousands of nerve projections through the cribriform plate into the olfactory epithelium. What is at the ends of these projections?

The ends of these nerve projections have tiny receptors that are each sensitive to only one molecule each. This accounts for how we smell distinct odors.

Describe the evolutionary approach of motivation

The evolutionary approach is based on the instinct theory - people are driven to do certain behaviours based on evolutionary programmed instincts. Described motivation in reference to Darwinian evolution. William James was the first to write about instincts.

Describe the role of the hypothalamus and how it regulates hormones and maintains homeostasis.

The hypothalamus is the control center that links the CNS and endocrine system - located in the forebrain and can have organism-wide effects. The hypothalamus controls the pituitary through paracrine hormones (regionally acting) into a hypophyseal portal system, and can receive input from a variety of sources. It regulated the release of hormones and maintains homeostasis via negative feedback.

Alfred Alder also formed a psychoanalytical theory - he was the originator of the *inferiority complex*. How did his theory describe personality?

The inferiority complex refers to individual's sense of incompleteness, imperfection and inferiority (physically & socially). Alder's theory stated striving for superiority enhances and drives personality.

Sensory adaptation can protect your ears (via the tympanic reflex) from damage during a long fireworks display. How does this work?

The inner ear muscle contracts after an increased, sustained stimulus, decreasing ossicle vibration and downstream sensitivity.

What are *self-disclosure* and *reciprocal liking* and how are they components of attraction?

The opportunity of *self-disclosure*, sharing one's fears, thoughts, and goals with another person and being met in a non-judgemental empathy, deepens attraction and friendships or relationships. These must be met with reciprocal behaviour, however. Reciprocity influences aspects of attraction - *reciprocal liking* is the phenomenon whereby people like others better when they believe the other person likes them.

The autonomic nervous system is split into sympathetic and parasympathetic components. What do each do?

The parasympathetic nervous system controls homeostasis and the body at rest, and is responsible for the "rest and digest" function of the body. The sympathetic nervous system controls the body's response to threat, and "fight or flight" mode.

How does the social identity theory describe/categorize the self concepts?

The social identity theory has two components - the personal self, referring to our individual and unique self, and the social self, referring to our community and groups we belong to.

What is the essence of the biological perspective of personality theories?

There is variation amongst biological theorists but in essence, the biological perspective holds that personality can be explained by the result of genetic expression in the brain and behaviour. The biological theory suggests that important components of personality are [[inherited]]

What does it mean to say the Population Growth Rate is 7?

There were 7 new individuals in that community per 1,000 people for that year.

True or false: one of our biases is when we judge the behaviour of others, where we are likely to attribute their behaviour more to internal factors of their character rather than external, situational factors.

True: one of our biases is when we judge the behaviour of others, where we are likely to attribute their behaviour more to internal factors of their character rather than external, situational factors. This is known as *Fundamental attribution error*

A _________ signal detection strategy will correctly reject any time there is no signal, however may miss a signal when present. A ________ signal detection strategy will always correctly choose a hit, but will also detect false alarms. Which answer choice correctly fills in the blank? (A) Liberal, Conservative (B) Conservative, Liberal (C) Liberal, Non-liberal (D) Conservative, Non-liberal

(B) Conservative, Liberal

What structure of the eye is transparent and serves to protect the eye while bending incoming light? (A) Pupil (B) Cornea (C) Iris (D) Lens

(B) Cornea The Cornea is a transparent structure in the front of the eye that serves to protect the eye while also bending some incoming light.

While driving, Adam missed his street because that sign was blocked by a closer stop sign. Which monocular cue is this most like? Explain. (A) Relative size (B) Interposition (C) Relative height (D) Shading and contour

(B) Interposition infer that an object that is directly in front of another

When you first jump into a very cold pool, you will noticeably feel the cold. After a while, however, you stop feeling cold. What type of neuron timing does this represent? (A) Non-adaptive (B) Slow-adapting (C) Fast-adapting (D) Light-adapting

(B) Slow adapting The cold is sensed for a considerable amount of time before stopping, meaning that slow-adapting neuron timing is involved here.

After light crosses the lens it passes through the ______________ which is filled with a jelly-like substance known as the ____________, that helps to suspend the lens in place while also providing some structure to the eye itself. (A) Anterior Chamber, Aqueous Humor (B) Anterior Chamber, Vitreous Humor (C) Posterior Chamber, Vitreous Humor (D) Posterior Chamber, Aqueous Humor

(C) Posterior Chamber, Vitreous Humor

Parts of the eye: conjunctiva

thin layer of epithelial cells covering the cornea - helps moisturize cornea and protect from dust and debris

You see two trees ahead of you. One is higher than the other and is perceived to be further in the distance. Which monocular cue is responsible for this? Explain. (A) Interposition (B) Relative size (C) Relative height (D) Shading and contour

(C) Relative height tells us that objects that are higher in our visual field are perceived to be further away

There are approximately ____ million rod cells in the human eye. There are approximately ____ million cone cells in the human eye.

120, 6

Match the type of reasoning with the type of processing: (A) Inductive Reasoning (B) Deductive Reasoning (1)Bottom-Up Processing (2)Top-Down Processing

(A) Inductive Reasoning -> (1)Bottom-Up Processing (B) Deductive Reasoning -> (2)Top-Down Processing Inductive reasoning moves from a specific instance to a generalized conclusion, just like Bottom-Up Processing Deductive reasoning moves from generalized principles to a specific conclusion, similar to Top-Down Processing

The anterior chamber of the eye is filled with a water/salt substance, that provides nutrients to the cells of the cornea and iris. What is this water/salt substance known as? (A) Vitreous Humor (B) Lens (C) Aqueous Humor (D) Retina

(C) Aqueous humor

What is bottom up processing?

Process begins with the stimulus, and the stimulus influences our perception. e.g. looking at a cockpit of a plane as a novice - you look at all the parts and gauges and data received drives your perception; when you go from a dark to a bright room and the eyes must adjust to the brightness

Which of the following is NOT a type of Tyrosine Derivative? (A) Cortisol (B) Adrenaline (C) Thyroid Hormones (D) Norepinephrine

(A) Cortisol Cortisol is a classic Steroid hormone. Tyrosine Derivatives include Catecholamines (Norepinephrine and Adrenaline) and Thyroid Hormones.

In America, the Baby Boomer Population is aging quickly and living longer than ever before. What type of population pyramid would you expect to see in this situation? (A) Constrictive Population Pyramid (B) Retroactive Population Pyramid (C) Expansive Population Pyramid (D) Stationary Population Pyramid

(A) Constrictive Population Pyramid

Which of the following typically unites an organization? (A) Common Goal (B) Seeking out adversity (C) Paying membership dues (D) Difficult initiation rituals

(A) Common Goal A common goal is typically the uniting factor of an organization.

________________ taste buds are mushroom shaped structures located primarily in the anterior portion of the tongue. (A) Fungiform (B) Foliate (C) Circumvallate (D) Filiform

(A) Fungiform Fungiform taste buds are mushroom shaped structures located primarily in the anterior portion of the tongue.

A ray of light from the left visual field hits the ____________ side of the left eye and hits the __________ side of the right eye. (A) Nasal, Temporal (B) Temporal, Temporal (C) Nasal, Nasal (D) Temporal, Nasal

(A) Nasal, Temporal

Match each of the following: (A) Stereotyping (B) Prejudice (C) Discrimination (1) Behavioral (2) Affective (3) Cognition

(A) Stereotyping -> (3) Cognition (B) Prejudice -> (2) Affective (C) Discrimination -> (1) Behavioral

Phil hasn't had a girlfriend in a few years, and is starting to feel extremely lonely. How old is Phil? (A) 5 years (B) 21 years (C) 48 years (D) 68 years

(B) 21 years Phil is struggling with Intimacy vs. Isolation, which occurs between the ages of 18 and 40.

Because of their __________ nature, the protein class of hormones have receptors located ____________. (A) Charged, intracellularly (B) Charged, in the cell membrane (C) Non-polar, intracellularly (D) Non-polar, in the cell membrane.

(B) Charged, in the cell membrane

Mark believes that you should never steal because if everyone stole then society would no longer be a stable place to live. Which stage of moral development is Mark currently in? (A) Pre-conventional (B) Conventional (C) Post-conventional - Social Contract (D) Post-conventional - Universal Principles

(B) Conventional Mark is focused on maintaining social order, which is the focus of the second level of the Conventional Stage.

Which class of hormones has a characteristic cyclic portion made of three 6-membered rings and one 5-membered ring, and are synthesized from cholesterol? (A) Polypeptides/proteins (B) Steroids (C) Tryptophan derivatives (D) Tyrosine derivatives

(B) Steroids

Gary loves working on things. In his spare time, he likes to paint and play sports. He feels productive. How old is Gary? (A) 8 months (B) 2 years (C) 7 years (D) 13 years

(C) 7 years Gary is overcoming Industry vs. Inferiority, which occurs between the ages of 6 and 12.

A Human's brain goes through 4 main stages during sleep. These stages occur in __________ minute cycles during a normal night of sleep. (A) 70 (B) 80 (C) 90 (D) 100

(C) 90

Which structure of the brain receives all auditory information from the cochlea and is separated into regions which detect different frequency sounds? (A) Occipital Lobe (B) Cerebral Cortex (C) Primary Auditory Cortex (D) Secondary Auditory Cortex

(C) Primary Auditory Cortex

Which of the following is NOT an example of a tyrosine derivative? (A) Thyroxine [T3] (B) T4 (C) Epinephrine (D) Cortisol

(D) Cortisol

Which structure is a bundle of nerves that lies right above the cribriform plate that sends little nerve projections through the cribriform plate into the olfactory epithelium? (A) Glomerulus (B) Filiform Papillae (C) Mitral Cell (D) Olfactory Bulb

(D) Olfactory bulb The olfactory bulb is a bundle of nerves that is an extension of the brain and lies above the cribriform plate. It sends nerve projections through the cribriform plate into the olfactory epithelium.

In the movie Avatar, individuals can wear a mask, which puts them inside a virtual world. Some people might argue that our lives are much like this in which the things around us may not actually be real. Which theory is most relatable to? (A) Conflict Theory (B) Symbolic Interactionism (C) Functionalism (D) Social Constructionism

(D) Social Constructionism This theory looks at how our reality is socially constructed and may not actually be as "real" as we think it is.

Your psychology teacher tells you that even though you practiced you presentation, you will perform it differently in front of others than you did alone. Your professors remarks are most closely in line with which sociological phenomenon? (A) Social Loafing (B) Social Change (C) Peer Pressure (D) Social Facilitation

(D) Social facilitation *Social facilitation* is the tendency of people to perform better on simpler (familiar) tasks in the presence of others, while the opposite is true for complex (unfamiliar) tasks.

In Europe, the population is neither increasing nor decreasing and birth rates largely match mortality rates. What type of population pyramid would you expect to see in this situation? (A) Constrictive Population Pyramid (B) Retroactive Population Pyramid (C) Expansive Population Pyramid (D) Stationary Population Pyramid

(D) Stationary Population Pyramid

Gary wants to be a doctor, but feels like he can't get into school because he is not part of a minority group. In order to get into medical school, he decides to lie on his medical school application to make himself sound better than he actually is. Which theory would best explain the Deviance in this example. (A) Social Interactionist Perspective (B) Labeling Theory (C) Theory of Differential Association (D) Strain Theory

(D) Strain Theory Gary is trying to reach a goal and resorts to deviance upon realizing that his goal is unattainable in his situation. This is best explained by Strain Theory.

Austin starts a new initiative to improve their urban areas. This is an example of which of the following? (A) Suburbanization (B) Urban Sprawl (C) Urban Decline (D) Urban Renewal

(D) Urban Renewal *Urban Renewal* is a process by which urban areas are made nicer.

Differentiate between cultural transmission (learning) and cultural diffusion

*Cultural transmission* is the manner in which society socializes its members; *Cultural diffusion* is the spread of norms, beliefs, customs throughout a culture.

Differentiate between distal and proximal stimuli

*Distal stimuli* - objects and events in the world around you *Proximal stimuli* - patterns of stimuli from these objects that actually reach your senses (i.e. eyes, ears etc.)

A diagnosis of persistent depressive disorder is given to individuals who suffer from dysthymia, what is dysthymia?

*Dysthymia* is a depressed mood that isn't severe enough to meet the criteria for a major depressive episode, for at least two years or so.

Is gender identity strictly linked to biological sex? Discuss gender identity, androgyny and gender schema

*Gender identity* describes a person's appraisal of him/herself on scales of masculinity or femininity - these concepts are considered two separate dimensions due to individuals being able to score high on both scales (*androgyny*) and score low on both scales (*undifferentiated*). Gender and biological sex are closely related but need not be necessarily tied, as seen in Western cultures. Theories like the *gender schema* theory believe components of gender identity are transmitted through cultural + societal means.

Describe the three perspectives of looking at globalization: - Hyperglobalist - Skeptical

*Hyperglobalist* perspective - sees globalization as a process, new age in history, as nations become interdependent and function more as one global society rather than individual entities. *Skeptical* perspective - critical of globalization - considers process as more of regionalization than globalization, as 3rd world economies are not experiencing the same benefits of globalization that first world economies are. National borders being important as ever is considered a sign of this. *Transformationalist* perspective - national governments are changing, into patterns that are uncertain - NWO design is developing

Differentiate between prejudice and discrimination.

*Prejudice* is an irrational positive or negative attitude towards a person/group/thing/object etc. without prior experience or interaction with the entity. *Discrimination* is a negative behaviour that ACTS UPON prejudice, causing individuals or groups to be treated differently from others.

Jimmy watches a lot of violent television and plays Call of Duty: Black Ops III all the time. He tends to be more aggressive toward his friends and family. How might the idea of Social Scripts explain Jimmy's behavior?

*Social Scripts* could explain Jimmy's behavior in that Jimmy is learning a script of how he should behave from his video games. Social Scripts basically act as guidelines for us, informing us how we should behave in certain situations.

Durkheim described social facts - what are they?

*Social facts* are ways of thinking and acting formed by society formed before any one individual existed, and will persist after they are gone. They cannot be influenced by an individual and have a coercive effect over the individual, only apparent when we resist them. E.g. laws, moral regulations, religious beliefs, suicide

Describe the following types of formal organizations: - Utilitarian - Normative - Coercive

*Utilitarian organizations* - members paid or rewarded for their efforts (e.g. businesses, govt jobs, universities) *Normative organizations* - groups that come together for a common or shared goal - characterized by unity and purpose (e.g. religious groups) *Coercive organizations* - members don't have much of a choice about membership/don't have exact ability to leave - very strict rules (e.g. prisoners, military).

What are some examples of stigma? Think of any!

- People with mental disorders are dangerous or crazy. - People with addictions are flawed. - Women who have abortions are amoral.

Social Identity says that there are three steps that we take to develop a Self Identity. Describe these three steps using yourself as an example: 1. Categorize 2. Identification 3. Comparison

1. Categorize - You associate different things with other things (i.e. jocks, nerds, drama queens, etc). 2. Identification - You adopt the identity of the group that you identify yourself as (i.e. "I am a nerd"). 3. Comparison - You compare your group to other groups (i.e. "Nerds are so much smarter than Jocks").

World Systems Theory divides the world into three types of countries. Give an example of each type and explain why it is a good example: 1. Core Countries 2. Semi-periphery Countries 3. Periphery Countries

1. Core Countries - United States, Canada, Europe, Japan, Australia have a strong central government, are industrialized, and focus on higher-skill, capital-intensive production. 2. Semi-periphery Countries - Mexico, China, South Africa are a mix between Core and Periphery Countries. 3. Periphery Countries - South America, Africa, Russia, Indonesia have a weak central government, are dependent upon Core Countries economically, and focus on producing raw goods or materials.

Little Timmy sees himself as the center of the universe. Describe how the following stages may play a role in helping little Timmy start to be influenced by others: 1. Preparatory Stage 2. Play Stage 3. Game Stage

1. Preparatory Stage - Timmy starts imitating others, allowing him to realize that he is a distinct individual from others. 2. Play Stage - Timmy starts pretending that he is a lawyer like his daddy. He begins to see things better from other's perspectives. 3. Game Stage - Timmy starts to understand that his teachers at school also have home lives and families. He also starts to see himself from the perspective of others.

Carl Rogers believed that the path toward self-actualization starts early in our lives and requires a growth-promoting climate. What are the two conditions that are essential to a growth-promoting climate?

A growth promoting climate is a place where we feel free to be our *genuine* true selves. It is also a place where we feel *acceptance/unconditional positive regard* from others. Rogers believed that when both these conditions are met, we will develop a self-concept.

Attachment is an emotional bond between a child and its caregiver. Psychologist Mary Ainsworth conducted the *Ainsworth-Strange Situation* - describe it and what it found.

A mother, her child, and a stranger are in a room. The mom gets up and leaves at one point, leaving the child alone with the stranger. Later on the mom returns. The child's behavior is observed, leading to the classification of the attachment style between mother and child - secure or insecure attachments. The experiment found that infants need a secure base in form of a consistent caregiver for the first 6 months - 2 years of life to explore the world and develop appropriately.

How would you describe Dissociative Identity Disorder (DID)?

A personality disorder where two or more distinct personalities or identities exist in the same individual. Both identities have influences on individual's thoughts or behaviours. The identities are distinct from one another in terms of: -Mannerisms -Emotional responses -"Physical" changes (e.g. changes in dominant hand!) -Denial of existence of other personality (usually by the original personality) The disorders legitimacy has been debated due to how rare it is, and how it is even rarer outside the US. Many of the sufferers seem to have had physical or sexual abuse of some sort in childhood.

What is the key difference between primary and secondary sexual characteristics?

A primary sexual characteristic must be present for reproduction to occur, whereas a secondary sexual characteristic could be lacking and reproduction can still occur.

There are several types of anxiety disorders. Describe specific phobias

A specific phobia is a focused anxiety - irrational fear of something that results in a compelling desire to avoid it. Typically associated with fear of animals, insects, blood or enclosed spaces. Claustrophobia - irrational fear of closed spaces; arachnophobia - fear of spiders. Social phobias are more impactful on people's lives - phobias of certain social environments leading to avoidance and isolation for many cases.

Sound waves are discriminated and sorted by its frequency inside the fluid-filled cochlea. Would a lower or higher frequency sound wave travel farther into the cochlea?

A wave of lower frequency would travel farther into the cochlea

What refers to the fact that after getting used to the effects of a drug, one needs more of a drug to achieve the same high? A) Tolerance B) Withdrawal C) Intoxication D) Relapse

A) Tolerance

What is a possible explanation for why learning a language is much easier in childhood than adulthood?

After puberty, the corpus callosum stops developing, which limits new connections made between hemispheres, making it more challenging to learn a new language.

Alcohol is often associated with nightlife, dancing, bars, and a fun time. Why is alcohol then considered a depressant?

Alcohol lowers a person's inhibitions and thus decreases one's cognitive control, a common symptom among depressants. Alcohol inhibits logical thinking and common symptoms for other depressants are observed for alcohol as well; ex: lack of coordination, slurring of speech, etc.

Pancreas - from endocrine standpoint, small clusters of hormone producing cells are grouped into the islets of Langerhans - Islets contain three cell types each secreting a different hormone - what are they and what do they secrete?

Alpha cells - glucagon Beta cells - insulin Delta cells - somatostatin

Compare Alzheimer's Disease with Korsakoff Syndrome: - Cause of each? - Symptoms? - Treatment? - Permanent or Treatable?

Alzheimers: Unknown cause, problems based more on attention, semantic and abstract thoughts. Progressive and terminal. Untreatable. Korsakoff's Syndrome: Caused by lack of B1/thiamine, so lack of glucose metabolism, will have poor balance and abnormal eye movement along with memory/confusion issues. Can be treated and improve.

How is cholesterol (and after that steroid hormones) biosynthesized from isoprene groups and pyrophosphate (a good leaving group)?

An isoprene unit has a C-backbone of 2-methyl butyl (image). A triterpene is made up of 6 isoprene groups, bound to form 4 rings of Carbon - 3 cyclohexanes and one cyclopentyl ring Pyrophosphate is a weak base, making it a good leaving group. Electrons from one pi bond in isoprene will replace a pyrophosphate group of another isoprene to join - once there is a 30 carbon backbone (a squalene), ring closing reactions occur to close a 4-ringed chain, forming the cholesterol structure. Thereafter, cholesterol can be modified to form a steroid hormone.

How would you describe attitudes (ABC model) and how they influence us?

Attitudes can be described as learned tendencies with three components: affective, behavioural, and cognitive components (ABC model). Affective component refers to how we feel emotions, which shapes our attitude Behavioural component refers to how we behave towards a subject, influencing our attitude Cognitive component refers to thoughts and beliefs or knowledge we form about something which influences our components.

Differentiate automatic processing from controlled (effortful) processing

Automatic processing is when information is gained without effort; controlled processing is active memorization of information. With practice, controlled processing can become automatic (think when you get good at a foreign language)

Describe the types of decision making heuristics: - Availability heuristics - Representativeness heuristics

Availability heuristic are when we make decisions or try to decide how likely something is - use examples that are readily available and easily come to mind - sometimes correct but not always. Representativeness heuristics involve categorizing items on the basis of whether they fit stereotypical, prototypical or representative images of the category

Cluster C (anxious/fearful) is defined by which three types of personality disorders?

Avoidance, dependent, and obsessive-compulsive (OCPD) personality disorders.

What were Skinner and Pavlov's views on the behaviourist perspective?

BF Skinner championed the behaviourist perspective. His view focused on operant conditioning, and described personality as the result of behaviours reinforced over time. Pavlov - focused on classical conditioning. Both believed that the environment shapes behaviour and personality, and that it develops over a life span.

Jim meets a new girl at work and Jim thinks she is very attractive. He is later made her manager and is asked to evaluate her in several key indicators. How might the Halo Effect play into Jim's evaluation?

Because his overall impression of her is good, he will likely over-exaggerate how good she is in each individual area of work.

Electroencephalography (EEG) is used to record average electrical patterns of the brain - there are four characteristic patterns, or waves, associated with different states of waking or sleeping. What are beta and alpha waves?

Beta and alpha waves characterize brain wave activity when we are awake; Beta waves have a high frequency (12-30Hz) and occur when we are alert and attending a mental task - neurons are randomly firing Alpha waves occur when we are awake but relaxed with our eyes closed, and are slower than beta waves (8-13Hz) - more synchronized

How are circadian rhythms regulated?

Biochemical signals underlie the concept of circadian rhythms. Less light induces melatonin release for sleepiness, while more induces cortisol release for wakefulness. Sleepiness is attributed to increased levels of melatonin, a seratonin-derived hormone) released by the pineal gland. The retina is directly connected to the hypothalamus, which controls the pineal gland - decreasing light can trigger release of melatonin. In contrast, more light = cortisol release, which contributes to wakefulness. Increased light causes release of corticotropin releasing factor (CRF) from the hypothalamus - causes release of adrenocorticotropic hormone (ATCH) from anterior pituitary, which stimulates cortisol release.

Compare Birth Rate and Fertility Rate.

Birth Rate is the number of new babies per 1,000 people per year. Fertility Rate on the other hand is the number of children a woman would be expecting to give birth to throughout her entire life based on averages (i.e. 2.1 average children per woman).

Cluster B (dramatic/emotional) is defined by four types of personality disorder. Describe borderline personality disorder

Borderline personality disorder - 2x more common in women - characterized by unstable relations and emotions as well as unstable self-image. Can be very impulsive, high rates of suicide and self-mutilation.

Compare bottom-up and top-down processing.

Bottom-up: Stimulus influences our perception. Data driven and no preconceived knowledge is used, thus integrates stimuli slower. Top-down: Using background knowledge to influence perception. Theory-driven, it is quicker, but more prone to mistakes.

When an Ion channel opens due to receptor binding, do cations or anions flow into the cell, causing depolarization?

Cations. When a molecule binds to a receptor on an ion channel, the ion channel opens and +ions begin to flow in, causing depolarization, and an action potential.

Noam Chomsky is known for his study on transformational grammar, and advocated for the existence of some innate capacity for infants to learn language. Which theory of language is this referring to?

Chomsky's theory refers to the *nativist theory (biological/innatist) of language development* - states that infants have an innate ability to learn language. This innate ability was explained by a *language acquisition device (LAD)* - theoretical pathway in the brain that allows infants to process + absorb language rules

Our daily cycle of sleeping and waking up is regulated by internally generated rhythms called __________________

Circadian rhythms. Our circadian rhythms approximate 24-hour cycles and are somewhat affected by external cues such as light.

Compare and contrast Classical and Operant Conditioning. Give an example.

Classical Conditioning is when you learn to associate a stimulus with another stimulus (i.e. Pavlov's dogs associate bell ringing with getting food). Operant Conditioning is when you learn to associate a consequence with a behavior (i.e. My dog associates getting a treat with going potty outside where he is supposed to).

There are ten personality types organized into 3 clusters. What are these clusters?

Cluster A - Odd and eccentric (paranoid, schizotypal, schizoid) Cluster B - Dramatic, emotional and errate (antisocial, borderline, narcissistic, histrionic) Cluster C - Anxious and fearful (avoidant, dependent, obsessive-compulsive)

After living with a depressed roommate for months and not scoring well on the LSAT, Jamarcus starts to develop depression himself. Could co-rumination have been a factor? If so, how?

Co-rumination is definitely a possible factor. Living with a depressed roommate, and after failing, Jamarcus and his roommate both may have sulked, and Jamarcus could have adopted some of his roommate's tendencies and attributions.

Differentiate between the conservative and progressive views of institutions

Conservative view - sees institutions as natural positive byproducts of human nature Progressive view - sees institutions as artificial creations that need to be redesigned to benefit humanity

Describe the term and what kind of cue it is: eye convergence

Convergence - binocular cue - inward movement of eye towards an object when looking at something close to us, relaxing when something is far away

What are binocular cues? Provide examples.

Cues about depth information derived from the use of both eyes. Examples include: Stereopsis (2 slightly diff images on retina), Eye convergence, Retinal disparity (the difference in image location seen by both eyes; ~2.5in) Yielding depth

What are monocular cues? Provide examples.

Cues provide depth information when viewing a scene with one eye. Examples include: Motion parallax (relative motion) Relative size Interposition Relative height Lighting, shading and contours Size constancy Shape constancy Colour constancy

Describe the Dependency theory of globalization and how it relates to the other two theories

Dependency theory was a reaction to modernization theory and uses the idea of core and periphery countries to look at inequalities between them. Periphery (3rd world) countries are described as poor and supply resources to the core, first world countries. It describes these poorer countries as stuck in subordination and dependence to the core countries, and unable to break from that cycle.

Describe discrimination based on sex/gender and sexual orientation

Differences in gender do not necessarily imply inequity but occues in many countries. *Gender inequality* is intentional/unintentional empowerment of one gender over another; *gender segregation* is separation of individuals on the basis of perceived gender (division in labour, bathrooms, sports teams, etc). These are based again on perceived genders, associated with sex and sexual identity and orientation. Restrictions might be placed on homosexuals that don't apply for heterosexuals, such as the right to marry. This has changed now, but others may still exist. Gender identity does not have to match biological sex.

Why does the sympathetic nervous system decreased our digestive rate and inhibit peristalsis (movement of food through our digestive tract)?

Digestion requires energy - in fight-or-flight mode, your body requires all the energy in that state in order to flee or survive the given situational threat. Thus, it decreases digestion, and inhibiting peristalsis is a part of doing so.

Sensory adaptaton by your sight senses when it's extremely bright out - what kind of regulation is this?

Down regulation. When it's too bright, lots of light enters your eye and pupils go from dilated -> constricted to allow less light to enter the back of the eye, and the rods and cones are desensitized to the light.

Describe the drive-reduction theory of motivation

Drives are internal states of tension that induce particular activity. The drive-reduction theory states drives help humans survive by creating an uncomfortable state ensuring motivation to eliminate this state or relieve internal tension. Primary drives (need for food, water, warmth) motivate us to sustain bodily temp in homeostasis. Homeostasis is controlled by negative feedback.

Describe how the Marshmallow Experiment applies to the idea of *Ego Depletion*.

Ego Depletion views self-control as a limited quantity that can get used up. If a kid holds off on eating the marshmallow for 10 minutes and then finally gives in, it would be because his self-control "ran out."

You want to develop better eating habits, how can you use your newfound knowledge of Ego Depletion to help you develop a good diet plan?

Ego depletion is the idea that there are limits to our self control, which eventually "runs out". Understand your limitations. Allow yourself to indulge in a healthy way. If you try to restrain yourself too much, you may end up overeating big time or giving up on your diet altogether. Think of it like, you need cheat days and/or refeed days when trying to get shredded because otherwise you'll crash due to ego depletion and off-set yourself.

What are the three elements of secondary appraisal?

Evaluating harm, threat and challenge. The organism evaluates harm (damage caused by the event), the threat (potential for future damage caused by event) and the challenge (potential to overcome and maybe benefit from the event). Individuals who perceive the ability to cope experience less stress.

Remaining in stress and overdrive for too long puts your body in a system of overdrive. What are some of the physical effects of stress on the immune system?

Excess stress can lead to a suppressed immune system via too much inflammation - arthritis is an example. However, if the body gets conditioned to chronic stress, then the immune system can get suppressed, making us more susceptible to illnesses and other things like reduced healing rate.

How might Exchange Theory relate to Altruism?

Exchange Theory would say that truly Altruistic Acts do not happen since they do not logically result in the greatest gain for the individual deciding to act altruistically.

What are some ways people manage stress?

Exercise and meditation Religious beliefs/faith Cognitive flexibility to change perspective

Compare Exogenous vs. Endogenous Cues.

Exogenous cues are external to our goals, will capture attention anyways (loud noise, bright colours). Endogenous cues are more internalized and require higher order thought (seeing an arrow and knowing it means to look/go that direction).

Differentiate between Freud, Jung and Alder in terms of their psychoanalytical theories of personality.

Freud's major assumption was that behaviour is motivated by inborn instincts; Jung's principal is that a person's conduct is governed by inborn archetypes; Alder's theory assumes people are primarily motivated by striving for superiority.

Describe the role of glucagon in the body

Glucagon is released during times of fasting by alpha cells of the islets of Langerhans - low glucose concentrations stimulate secretion of glucagon to trigger degradation of protein and fat, conversion of glycogen to glucose, and production of new glucose via gluconeogenesis. When glucose levels are high, glucagon release is inhibited.

Describe how the government and economy function as a social institution

Government & economy - defined as systematic arrangements of political and capital relationships, activities, social structures. Government may heavily influence other institutions - may cause family structure changes, changes in education system, religious tolerance and freedom, certifying healthcare and medicine etc. - influence is bidirectional (someone who doesn't like the changes may vote for a different candidate next time around) Capitalist economies requires division of labour for efficiency and specialization, and face little govt intervention - success of businesses driven by consumerism; Socialist economics treat large industries as collective, shared businesses - compensation provided is base don work contribution of individuals and profit is distributed equally in the work force.

Why are some hallucinogens used to treat patients with PTSD (post traumatic stress disorder)?

Hallucinogens can allow people to access painful memories from the past without strong emotions, therefore allowing people to properly handle such an experience - not possible under normal circumstances.

Cluster B (dramatic/emotional) is defined by four types of personality disorder. Describe histrionic personality disorder

Histrionic personality disorder - characterized by being attention-seeking - usually are very extroverted, wear bright clothes, and can even sometimes be dramatic and seductive.

How does a human's Reconstructive Memory differ from a computer's memory?

Humans have reconstructive memory, which can slightly alter a memory as it is recalled based on one's mental state, mood, desires, etc. A computer's memory, however, will recall information accurately time and time again.

Excess of thyroid hormone (which can result from a tumor or over-stimulation of thyroid) can result in hyperthyroidism - what characterizes this condition?

Hyperthyroidism is characterized by heightened activity level, increased body temp, increased heart and breathing rates, heat intolerance, weight loss.

Kohlberg's moral reasoning theory of personality development focuses on development of moral thinking, and states as out cognitive abilities grow, we are able to think about the world in more complex ways. It describes 3 stages and 6 phases - what are they?

I. Preconventional morality - consists of stage 1 (obedience) and stage 2 (self-interest) II. Conventional morality - consists of stage 3 (conformity) and stage 4 (law and order) III. Postconventional morality - consists of stage 5 (social contract) and stage 6 (universal human ethics)

What are implicit (non-declarative) memories?

Implicit memories are more difficult to explain - they include memories for skills or how to do things (like bike riding). They lie outside of our conscious awareness. They use procedural memory (for procedures) and priming (previous experience influencing current interpretation of event)

When aging, which memories improve, remain stable and decline?

Improve: semantic memory, crystallized IQ, emotional reasoning Stable: Implicit memory, recognition, flashbulb memories Decline: recall, episodic memory, processing speed and divided attention, fluid IQ

Contrast the order in which an event, emotion, physiological response, and appraisal/label is applied in the Schachter-Singer and Lazarus Theories of Emotion.

In the Schachter-Singer theory, first an event occurs, then a physiological response, then the labelling of the event/response, and then an emotion. In the Lazarus Theory of emotion, first an event occurs, then it is appraised, and that appraisal simultaneously determines the emotion and physiological response.

Remaining in stress and overdrive for too long puts your body in a system of overdrive. What are some of the physical effects of stress on the heart?

In increased blood pressure, the blood vessels become respond by becoming more rigid and distend (swell from pressure inside) - increased vessel rigidity requires more force from the heart to move blood - leads to a cycle of elevating blood pressure, eventually hypertension. A damaging effect of this can be vascular disease (disease of blood vessels) - veins and arteries experience episodes of damage and inflammation + plaque build up. The coronary arteries is the worst place to experience this as they pump blood to the heart - coronary artery disease --> heart attack.

Describe innate behaviours

Innate behaviours are inherited, intrinsic behaviours that occur naturally. They are typically inflexible (not modified by experience), stereotypical species wide, and consummate (fully developed on first performance). Encoded by DNA, so subject to genetic change and mutations. Examples include blinking, eating, and many reflexes seen at birth.

Describe the term and what kind of cue it is: interposition

Interposition is a monocular cue - ability to know if something is in front of/behind another

What information does a Life Table tell you?

Ironically, a Life Table will tell you the chances of dying at a certain age.

Air moves through the nasal passage to the olfactory epithelium, which is sensitive to various molecules. What is an accessory olfactory epithelium?

It is a specialized epithelium which sends information to the accessory olfactory bulb

What is going to happen to the Baby Boomers as a percent of the population by the year 2025?

It is going to double! We will have many old people to help take care of. The number of people over the age of 64 is rapidly growing as the Baby Boomers population is growing older.

After 4 months out with an injury, a bodybuilder finds it easier to put on lost size than when he did initially. Why?

Kinesthesia plays a big part in muscle memory.

What is L-DOPA? How is it useful for Parkinsons?

L-DOPA is a precursor that is converted to dopamine once in the brain - it is therefore useful for partially managing Parkinsons by replacing what is lost due to the disease.

What is sleep deprivation?

Lack of sleep from one or multiple nights of poor quality or little sleep, resulting in irritability, mood disturbances, decreased performance and slow reaction time. Sleep deprivation is also linked to anxiety, depression, and some risk of obesity due to increased cortisol.

What might be a Latent Function of Apple as a company? What about a Manifest Function?

Latent Functions may include providing employees with a sense of purpose, health insurance, or even friendships with others. These are the purposes of a company that are hidden under the service and not directly related to their core purpose. Manifest Functions may include making profit, innovation, and contributing to the technology of the world. These are the obvious and stated purposes of a Social Institution.

There are three characteristics that influence how we are persuaded for or against something that are part of the elaboration likelihood model - what are they?

Message characteristics - features of the message itself (was the argument logical? did it have key points? was there good grammar? too long/short? addressed the issue?) Source characteristics - (did the speaker seem knowledgeable or trustworthy? where was the messaged collected from? physical environment?) Target characteristics - characteristics of the listener/that you bring - (good/bad mood? high/low self-esteem? level of alertness? intelligence of listener? hungry or satiated?)

Describe mineralocorticoids and how they work (+ e.gs)

Mineralocorticoids are used in salt and water homeostasis - aldosterone is the common example, which increases sodium reabsorption in kidneys. Aldosterone is under the control of the *renin-angiotensin-aldosterone system* - decreased blood pressure = juxtaglomerular cells of kidneys secrete renin = cleaves inactive plasma protein angiotensinogen -> active form angiotensin I -> angiotensin II (by angiotensin-converting enzyme) -> stimulates adrenal cortex to release aldosterone. Negative feedback mechanisms for homeostasis. renin -> (cleaves) angiotensinogen -> angiotensin I -> (enzyme) angiotensin II -> aldosterone release stimulated

How are night terrors different from dreams?

Night terrors are periods of intense anxiety during SWS - it is most common in children, who when experiencing, may thrash and scream and show signs of sympathetic overdrive (fast heart rate, rapid breathing) - can be hard to wake children as this occurs in slow-wave sleep.

There are several types of anxiety disorders. Describe Obsessive-Compulsive disorder

OCD is characterized by obsessions (persistent, intrusive thoughts and impulses) which increase tension or stress level, leading to compulsions (repetitive tasks). The relationship between the two is key - obsessions raise stress level, compulsions relieve the stress. It is a disorder when it comes to the point of disrupting or affecting someone's life and the behaviour becomes a problem.

Cluster C (anxious/fearful) is defined by three types of personality disorder. Describe obsessive-compulsive personality disorder

OCPD - individual affected is a perfectionist and inflexible, tending to like rules and order - also may show excessive stubborness, lack of sense of humour, inability to throw worn out clothes, etc. DIFFERENT from OCD - OCD is

What is observational learning/modelling?

Observational learning, or modeling, is a type of learning most associated with the work and social learning theory of psychologist Albert Bandura. Observational learning can produce new behaviors, and either increase or decrease the frequency with which a previously learned behavior is demonstrated.

Compare and contrast Obsessive Compulsive Disorder and Obsessive Compulsive Personality Disorder.

Obsessive Compulsive disorder is known for having compulsions on outside tasks or objects. Ego-dystonic (understand that their way is not the "correct" or harmonious way) Obsessive compulsive personality disorder has a focus on their life being completely ordered and controlled, not on some outside object. Characterized by being perfectionists and very inflexible, tend to like rules and order. They may also show extreme stubbornness, lack of sense of humour, even inability to throw out old clothes. Ego-syntonic (believe their way is the "correct" way and harmonious with their goals).

Compare Oedipus Syndrome and Electra Syndrome

Oedipus Syndrome occurs when a young boy is sexually attracted to his mom. Electra Syndrome occurs when a young girl is sexually attracted to her dad.

Why is that when one has a cold and their nose is stuffy, their sense of taste is diminished?

Olfaction (smell) and gustation (taste) senses are connected. When someone eats, molecules travel to the back of the throat and nose, so when eating one's sense of taste is being used in conjunction with one's sense of smell. When someone has a cold, their sense of smell is knocked out, therefore diminishing their sense of taste.

PTH is water soluble and does not require a protein carrier to travel in the blood. What does this tell you about PTH? How would work?

PTH is a peptide hormone, since it is water soluble and doesn't require a protein carrier. In the cell, PTH would not be lipid-soluble and so would not be able to pass through a plasma membrane. Thus, it would bind to a receptor within the membrane and signal a second messenger.

There are several types of anxiety disorders. Describe panic disorders

Panic disorders consist of repeated panic attacks - symptoms of a panic attack include fear and apprehension, trembling, sweating, hyperventilation, and a sense of impending doom. Panic attacks tend to be sudden bursts of short-lived but very severe, intense episodes.

How might Expectation Theory apply to a child that failed his most recent math test?

Perhaps this child failed the math test because his teacher did not believe he would succeed on this test; thus, the child was simply living up to the teacher's expectations. Another possibility is that the child's previous teacher did not push the children very hard and gave them easy A's, resulting in their unpreparedness for this next grade level.

Symptoms of schizophrenia include positive and negative symptoms. What are examples of each?

Positive symptoms refer to presence of symptoms such as delusions, hallucinations, disorganized thought, speech or behaviour. They have a psychotic dimension (delusions/hallucinations) and a disorganized dimension (disorganized activity). Negative symptoms refer to the absence of normal or desired behaviour, such as disturbance of affect or avolition (lack of engagement in goal-driven actions) - they may include the flat effect (emotional flattening) and inappropriate effect (discordance with content of speech - e.g. laughing while explaining someone's death).

Describe the cognitive appraisals of stress: primary, secondary appraisal and reappraisal

Primary appraisal - stage 1 - initial evaluation of the environment and associated threat - can be irrelevant, benign or positive, or stressful. Secondary appraisal - stage 2 - begins when threat revealed by primary appraisal - evaluating whether the organism can cope with the stress Some events require ongoing monitoring through constant reappraisal.

Describe REM sleep and why it is also called paradoxical sleep

Rapid eye movement (REM) sleep is interspersed between cycles of Non-REM sleep stages. In this stage, arousal levels reach that of wakefulness but muscles are paralyzed. It is thus called paradoxical sleep because one's heart rate, breathing patterns and EEG mimic wakefulness, yet the individual is in deep sleep.

Differentiate retrograde and anterograde amnesia.

Retrograde amnesia - loss of previously formed memories Anterograde amnesia - inability to form new memories

What is the difference between Sex, Gender, and Sexual Orientation?

Sex refers to biological sex Gender is a social construct corresponding to behavioural, cultural or psychological traits usually associated with sex - how you identify and express yourself Sexual orientation - sexual preferences and desires

Define socialization

Socialization is a lifelong process by which we learn social expectations and how to interact with others. This can include developing, inheriting and spreading norms, customs and beliefs. Individuals gain knowledge, skills, behaviours and habits necessary for inclusion in society.

There are four stages of sleep. What are they characterized by?

Stage 1 - when you are drowsy or doze off - theta waves are experienced by this point (irregular waveforms, low F high V) - also called stage N1 Stage 2 - as you fall more deeply asleep - the EEG shows theta waves along with sleep spindles and K complexes (see image) - stage N2 Stage 3 and 4 are known as slow wave sleep (SWS) - delta waves are seen, though stage 3 is aka stage N3; at this point it is harder to wake someone.

Describe the *self-fulfilling prophecy* in terms of stereotypes

Stereotypes can lead to certain expectations of particular people/groups, which can create conditions leading to conformation of those expectations. This process is known as the *self-fulfilling prophecy*, which essentially states that an initial cognitive stereotype becomes only more true as it is acted upon.

Describe steroid hormones and how they function.

Steroid hormones are derived from cholesterol (nonpolar molecules) and easily cross the cell membrane - they function by binding to intracellular/nuclear receptors to undergo confor-changes and induce increased or decreased transcription of certain genes. Dimerization is a common form of conformational change

What are subliminal stimuli?

Stimuli below the absolute threshold (less than 50% detected/not detected by the individual). Homer Simpson example:

True or false: the *object relations theory* is a psychoanalytical personality theory that refers to objects as representations of parents/caregivers based on subjective experiences during early infancy - these objects persist into adulthood and influence interactions and social bonds.

TRUE: the *object relations theory* is a psychoanalytical personality theory that refers to objects as representations of parents/caregivers based on subjective experiences during early infancy - these objects persist into adulthood and influence interactions and social bonds.

Differentiate between Taxis and Kinesis

Taxis is innate purposeful movement in response to stimuli. E.g. bugs moving towards a spotlight in a dark room Kinesis is innate random movement in response to stimuli. E.g. rats scurrying around and away in response to spotlight in a dark room.

Why is Temperament often associated with the Biologic Theory?

Temperament related to mood, activity level, etc. - it is an innate disposition (inherited) and the Biologic Theory is focused on the innate causes of our behavior.

Describe the Cannon-Bard theory of emotion. (hint: think separate and simultaneous)

The *Cannon-Bard theory of emotion* states that the cognitive and physiological components of emotion occur simultaneously and result in the behavioural component (action). This theory separates the two - disagrees with James-Lange - believes that one could experience physio- response without emotion. An event occurs and causes a response AND an emotion simultaneously E.g. heart races when you are scared but also when you run; "I am afraid because I see a snake and my heart is racing....run!"

The *self-discrepancy theory* consists of the actual self, ideal self and ought self - describe these and how they relate to self esteem

The *actual self* is the way we see ourselves; our self-concept. The *ideal self* is who we want to be The *ought self* is our representation of the way others think we should be. The closer these three selves are to one another, the higher our self-esteem.

What does the cognitive process dream theory state?

The *cognitive process dream theory* states dreams are merely the sleeping counterpart of stream-of-consciousness

Right now, you are preparing for the MCAT. Is the MCAT a Well-Defined or Ill-Defined problem, and why?

The MCAT is a well-defined problem, because it has a clear beginning and end. Especially with the structure and organization of the MCAT Self Prep eCourse! ;) An Ill-defined problem lacks that clarity.

There are a few theories that attempt to explain how attitudes influence behaviour. Describe the attitude-behaviour process model

The attitude-behaviour process model says that an event triggers an attitude, which influences our perception of an object or stimulus. Additional outside knowledge towards the situation or object, combined with our attitude, leads to our behaviour.

Describe the foot-in-door phenomenon on how behaviour influences attitude.

The foot in door phenomenon states we are much more likely to do a large, attitude changing task if we agree to perform a small one first. This is one way people get 'brainwashed' into doing things. "foot in door" or "dipping feet into water". E.g. trying our SARMS before going to full-fledged steroids; gateway drugs

In terms of storing memory, how is the Modified Semantic Networks (current theory) different than a simple hierarchical semantic network model?

The modified theory allows for a "hierarchy" based on personal experience and common knowledge, not necessarily just based on traits. This allows for individual differences in what connections exist in individuals, and differences in relative processing time.

The hypothalamus also releases PIF (prolactin-inhibiting factor) into the anterior pituitary - what is different about this hormonal release?

The other hormones release all stimulated the release of their respective hormones. However, PIF (which is actually dopamine) inhibits the release of Prolactin secretion (stimulates mammary glands in females to produce milk)

What is the first part of sound waves hit when entering the body for transduction?

The pinna, portion of the outer ear

Describe the first step of the phototransduction cascade. Light hits the rod cells, what happens? (think rhodopsin)

The rod cells have membrane bound structures called optic discs which contain proteins, one of which is rhodopsin. When light hits the rod cell, and strikes rhodopsin, it causes a conformational change in a molecule within - 11-cis retinal becomes all trans retinal. Subsequently, rhodopsin also changes shape.

How does Deutsch and Deutsch's Late Selection Theory differ from Broadbent's Early Selection Theory?

The selective filter that discriminates information to be processed in Deutsch's is after the perceptual processing, but is before this in Broadbent's.

There are a few theories that attempt to explain how attitudes influence behaviour. Describe the theory of planned behaviour

The theory of planned behaviour states we consider our implications of our actions before we decide how to behave. The best predictor of our behaviour are our intentions in a given situation.

Why doesn't fluid flow back around the cochlea and hit the eliptical window? Why only the circular window?

There is a membrane that runs along the length of the cochlea known as the organ of corti, which prevents this from happening. It consists of two different membranes, the basilar membrane and tectorial membrane.

What happens when you look at a map with directions and street names? Where in the working memory is this dealt with?

This requires *coordination of the Visuo-Spatial Sketchpad and the Phonological Loop* - *a role done by what is called the Central Executive*. It causes them to create an integrated representation stored in the Episodic Buffer (which is a connection to Long term memory)

Is the saying, "what goes up must come down" a Brute Fact or an Institutional Fact?

This saying is an Institutional Fact because it is based upon other facts such as gravity, which is based on mass, which is based on atomic structure, and so on. A Brute Fact is a fact that cannot be explained by any other facts. It can stand on its own.

True or false: Studies showed that a Type A personality, associated with anger and hostility, was linked to elevated blood pressure and heart levels associated with chronic stress

True

Type theories like the MBTI create a taxonomy of personality types - one well known theory describes type theories as Type A or Type B - what were these types?

Type A- competitive and compulsive behaviour; type B- laid-back and relaxed. Type A were found to be more prone to heart disease, though no evidence of higher mortality rates exist.

Baskin Robbins is known for having 31 flavors of ice cream. Jenny goes there for the first time and feels uncomfortable. How might Tyranny of Choice describe her feelings?

Tyranny of Choice is the idea that it is possible to have too many options/choices and that having too many can cause distress as is the case with Jenny. If her options were simply vanilla or chocolate, perhaps she'd feel less stressed by this choice.

Why are Tyrosine derivatives treated as a separate class of hormones from the polypeptide class?

Tyrosine derivatives are made of exactly one amino acid that is manipulated, not multiple. The cyclic structure also gives tyrosine derivatives some non-polar characteristics, so they can act similarly to both Polypeptide and Steroid hormones in certain contexts.

Both nociception and thermoception (pain and temperature sensation, respectively) use TrpV1 receptors. How do these receptors convert cellular damage into neural signals?

When a cell is damaged, it releases chemical signals that diffuse to the nearest TrpV1 receptor and activate it to fire an action potential.

Why do humans rely very little on pheromones? Which structure in the human olfactory system is not present that is present in other animals that rely on pheromones?

We lack the accessory olfactory bulb. We do have vomeronasal system but not the accessory olfactory bulb.

Parts of the eye: ciliary body

ciliary muscles help change the shape of the lens and make it contract by pulling on suspensory ligaments under parasympathetic conditions

Driving through the Appalachians, the nearby traffic signs appear to be moving faster than the further mountain peaks. Which monocular motion cue is causing this? Explain. (A) Motion Parallax (B) Interposition (C) Relative height (D) Shading and contour

(A) Motion Parallax

What type of constancy refers to fact that even though an image may look bigger than another image because it is closer, we still process the images as the same size? (A) Size Constancy (B) Shape Constancy (C) Relative Constancy (D) Color Constancy

(A) Size constancy

Which stage of sleep is characterized by rapid eye movement, however most of one's muscles are paralyzed? The most dreaming occurs in this stage of sleep and it is the most important stage of sleep for memory consolidation. (A) N1 (B) N2 (C) N3 (D) REM

(D) REM

There are a few processes that can modify the response to a conditioned stimulus after acquisition has occured. Differentiate between generalization and discrimination.

*Generalization* - broadening effect by which a stimulus similar enough to the CS can also produce the conditioned response. Little Alber experiment - afraid of a white rat it was paired with a loud noise. Subsequently showed his conditioning had generalized and fear response was also seen to a white stuffed rabbit, white beard, etc. *Discrimination* - opposite of generalization -when an organism learns to distinguish between two similar stimuli

How does Internal Migration differ from your typical Migration?

Internal Migration is when you move somewhere new within the same country or region.

For patients with CRSD (Circadian Rhythm Sleep Disorders), daily rhythms like body temperature, alertness and hormone secretion are impaired. What is the circadian rhythm responsible for in a healthy individual?

The circadian rhythm is often called a "biological clock", because it causes sleepiness to gradually increase during the day, regulating sleep and wakefulness.

Describe the cognitive approach of motivation

The cognitive approach of motivation focuses on our rationale and decision-making ability

How are the ego and superego related?

The ego mediates the desires of the superego, just as it does those of the id. The three are in conflict, and the ego's recourse for relieving anxiety caused by clash of the id and superego is via *defense mechanisms*

As many as __% of those with major depressive disorder die by suicide.

As many as 15% of those with major depressive disorder die by suicide.

Because the participants in the Zimbardo study knew this was an experiment, they acted the way that they thought Zimbardo wanted them to. This refers to which experimental error? (A) Demand Characteristics (B) Sampling Error (C) Ecological Validity (D) Double-blind Bias

(A) Demand Characteristics Demand Characteristics refers to participants trying to act in the way that they are expected to act in a study.

Correctly order Erikson's 8 stages of psychosocial development and give approximate age ranges for each: (1) Autonomy vs. Doubt (2) Industry vs Inferiority (3) Trust vs Mistrust (4) Initiative vs Guilt (5) Integrity vs Despair (6) Identity vs Role Confusion (7) Intimacy vs Isolation (8) Generativity vs Stagnation

(0 - 1) Trust vs. Mistrust (1 - 2) Autonomy vs. Doubt (3 - 5) Initiative vs. Guilt (6 - 12) Industry vs. Inferiority (12 - 18) Identity vs. Role Confusion (18 - 40) Intimacy vs. Isolation (40 - 65) Generativity vs. Stagnation (65+) Integrity vs. Despair

Dopamine is involved in many neural activities. Match the structure to its activity caused by dopamine: (1) Amygdala (2) Prefrontal Cortex (3) Hippocampus (4) Nucleus Accumbens (A) Memory formation (B) Emotion (C) Attention and planning (D) Controls Motor Function

(1) Amygdala - (B) Emotion (2) Prefrontal cortex - (C) Attention + planning (3) Hippocampus - (A) Memory formation (4) Nucleus Accumbens - (D) Controls Motor Function

The fastest growing *age cohort* in the US is: (A) 85+ (B) 65+ (C) 18-24 (D) 35-65

(A) 85+ >40% of adult patients in hospital beds are 65 or older - profound effects of the age cohorts.

In Kluver-Bucy Syndrome, patients are known to express hyperorality, hypersexuality and disinhibited behavior. Which part of the brain, normally associated with fear and anxiety, is usually affected by Kluver-Bucy Syndrome? (A) Amygdala (B) Thalamus (C) Hypothalamus (D) Hippocampus

(A) Amygdala The amygdala is known for controlling anger/violence and fear/anxiety. Kluver-Bucy Syndrome is a bilateral destruction of the amygdala, causing the aforementioned symptoms.

In the case of Phineas Gage, a metal rod penetrated Gage's skull. Afterwards, he showed less inhibition, along with increased crudity, temper, and an inability to plan. Which two areas were most likely injured? (A) Amygdala and Pre-Frontal Cortex (B) Amygdala and Hypothalamus (C) Cerebellum and Hypothalamus (D) Cerebellum and Pre-Frontal Cortex

(A) Amygdala and Pre-Frontal Cortex The metal rod injured mainly the pre-frontal cortex (planning and inhibition) and the amygdala (anger and emotion, thus attributed to crudity and temper).

Which of the following examples is NOT an example of Complex Behavior? (A) An insect is born knowing how to eat and continues to eat with age. (B) An insect is born knowing how to fly but improves its flying through practice. (C) An child learns how to run but gets progressively better. (D) An infant is born able to cry,. but develops other ways to express themselves.

(A) An insect is born knowing how to eat and continues to eat with age. Complex Behaviors have to exhibit parts that are both Innate and Learned Behavior, and choice (A) does not exhibit traits of Learned Behavior.

Which match of the following DSM5 (a system of categorizing mental illnesses) illness categories with its description is incorrect? (A) Anxiety Disorders - distress caused by only phobias (B) Neurodevelopmental - distress is caused by improper development of the nervous system (C) Neurocognitive - nervous system developed then lost function afterwards, causing distress (D) Bipolar disorders - distress caused by both depressive moods and mania

(A) Anxiety disorders - distress caused by only phobias Anxiety disorders can be general too, not only caused by specific stimuli like phobias.

Which of the following "anxious" and "fearful" Personality Disorders is characterized by feelings of inhibition and being inadequate, fear of rejection, or even extreme shyness? (A) Avoidant (B) Obsessive Compulsive Personality (C) Dependant (D) None of the above

(A) Avoidant Avoidant Personality Disorder is characterized by feelings of inhibition and being inadequate, extreme shyness and fear of rejection despite maybe even ever so desiring social acceptance, so they avoid social situations.

Match each Generational Cohort with the years of birth for that Cohort: (A) Baby Boomers (B) Silent Generation (C) GI Generation (D) Generation Z (E) Generation X (F) Millennials (1) 1901 to 1924 (2) 1925 to 1942 (3) 1943 to 1964 (4) 1965 to 1973 (5) 1974 to 2000 (6) 2001 to Present

(A) Baby Boomers -> (3) 1943 to 1964 (B) Silent Generation -> (2) 1925 to 1942 (C) GI Generation -> (1) 1901 to 1924 (D) Generation Z -> (6) 2001 to Present (E) Generation X -> (4) 1965 to 1973 (F) Millennials -> (5) 1974 to 2000

Even if Ebbinghaus could not recall his nonsense words he learned, they were not completely gone from his long-term memory. Which of the following ways can show information is still stored? (A) By relearning the material faster than novel material is learned (B) By recalling the sequences after context clues are given (C) By learning a new set of letters and having proactive interference from previous sets. (D) By teaching the material to someone else without realizing it.

(A) By relearning the material faster than novel material is learned. Ebbinghaus found that the rate of forgetting is rapid/sharp at first but levels off - called the "curve of forgetting"

Jimmy joined the army in order to serve his country. He and his fellow soldiers love protecting their country's freedoms. The soldiers are not allowed to quit the military but instead must be formally discharged. Which type of Organization does Jimmy belong to? (A) Coercive Organization (B) Normative Organization (C) Bureaucratic Organization (D) Utilitarian Organization

(A) Coercive Organization Coercive Organizations are those that you are either forced to join or those that you can't leave without permission, as is the case with the army.

When large numbers of individuals rapidly and randomly behave in ways that don't reflect societal norms, this phenomenon is known as: (A) Collective behaviour (B) Group behaviour (C) Groupthink (D) Group polarization

(A) Collective behaviour Collective behaviour is a phenomenon when large numbers of people rapidly and randomly behave in ways that don't reflect norms in society. Three types are fads, mass hysteria, and riots.

What is referred to as the awareness of our self and the surrounding environment? (A) Consciousness (B) Daydreaming (C) Drowsiness (D) Sleep

(A) Consciousness

In Zimbardo's Stanford Prison Experiment, the guards lost their identity as college students as they became guards. This is an example of what? (A) Deindividuation (B) Cognitive Dissonance (C) Fundamental Attribution Error (D) Confirmation Bias

(A) Deindividuation Deindividuation is when you feel less identifiable and so you engage in worse behavior. This is what happened with the guards.

An Amoeba can sense and move toward food. What is this an example of?

An Amoeba can sense and move toward food. This an example of Taxis as it is purposeful movement.

Of the four main classes of psychoactive drugs, which class is known for lowering the body's Central Nervous System's activity, decreasing the processing speed of the brain, and lowering one's heart rate and blood pressure? (A) Depressants (B) Stimulants (C) Hallucinogens (D) Opiates/Opioids

(A) Depressants Benzos and Barbituates also increase GABA activity, causing a sense of relaxation. Alcohol also increases GABA activity and dopamine levels.

Trait theory is focused on the _________________ of behavior whereas other theories are focused on the ______________ of behavior. (A) Describing, Origins (B) Origins, Describing (C) Causes, Effects (D) Effects, Causes

(A) Describing, Origins Trait theory is focused on the describing of behavior whereas other theories are focused on the origins of behavior.

What is Trained Incapacity? Which characteristic of a Bureaucracy may cause Trained Incapacity? (A) Division of Labor (B) Hierarchy of Organizations (C) Written Rules and Regulations (D) Employment based on Technical Qualifications

(A) Division of Labor Trained Incapacity is the idea that workers who only have a specific task to complete do not see the big picture, which may undermine the overall goal of the organization. This is caused by Division of Labor.

Which of the following would fall under the "Other/Unspecified" subcategory for Neurodevelopmental Disorders, meaning it meets some but not all of the criteria? (A) Down's Syndrome Disorder (B) Autism Spectrum disorders (C) ADHD (Attention Defecit Hyperactivity Disorder) (D) What formerly was called "mental retardation"

(A) Down's Syndrome Disorder Autism Spectrum Disorders, ADHD and what was formerly called "Mental Retardation" fully meet the criteria for Neurodevelopmental Disorders. Down's Syndrome meets some of the criteria, but not all, for this category, so it falls under the "other/unspecified" subcategory.

If a doctor has diagnosed Schizophrenia in a patient, which medications are they most likely taking to treat it? (A) Drugs targeting dopamine levels and receptors (B) Drugs targeting epigenetic markers (C) Drugs targeting synapse pruning (D) None of the above.

(A) Drugs targeting dopamine levels and receptors Most of the medications used for treating Schizophrenia target dopamine levels and receptors.

In the information processing model, which of the following terms best describes hearing a patient scream or noticing a cricket chirp, which is easily forgotten? (A) Echoic Memory (B) Sensory Memory (C) Working Memory (D) Iconic Memory

(A) Echoic Memory Echoic memory is memory for what you hear and is kept only in sensory memory.

Google gives all of its workers segways so that workers can get from one meeting to another in less time. Which of the four aspects of McDonaldization does this situation best exemplify? (A) Efficiency (B) Calculability (C) Predictability (D) Control

(A) Efficiency Efficiency is any aspect of an organization that is geared toward minimizing time loss.

Match the following theories with their respective theorist: (A) Emile Durkheim (B) George Herbert Mead (C) Karl Marx (1) Functionalism (2) Conflict Theory (3) Symbolic Interactionism

(A) Emile Durkheim -> (1) Functionalism (B) George Herbert Mead -> (3) Symbolic Interactionism (C) Karl Marx -> (2) Conflict Theory

Which theory of dreaming explains that dreaming has developed to occur as a threat simulation to prepare for the real world, or as a method of problem solving? (A) Evolutionary Biology theory (B) Sigmund Freud's Dream theory (C) Memory Consolidation Theory (D) Neural Pathway Theory

(A) Evolutionary Biology theory

When trying to first solve a Rubik's Cube, someone could be incorrectly focused on trying to solve all horizontal rows first. What is this an example of? (A) Fixation (B) Representativeness (C) Means-End Analysis (D) Insight

(A) Fixation Because the solver is stuck on a wrong approach, this is an example of fixation.

Freud stated that in each of his theory's 5 stages, children are faced with a conflict between societal demands and the desire to reduce libidinal tension associated with different erogenous zones of the body. ____________ occurs when a child is overindulged/overly frustrated during a stage of development. (A) Fixation (B) Libido (C) Instinctual drift (D) Neurosis

(A) Fixation Freud stated that in each of his theory's 5 stages, children are faced with a conflict between societal demands and the desire to reduce libidinal tension associated with different erogenous zones of the body. *Fixation* occurs when a child is overindulged/overly frustrated during a stage of development. In response to anxiety caused by fixation - children form personality patterns based on that stage, which persists into adulthood as a functional mental disorder called *neurosis*

You give your dog a treat after every time it goes potty outside. This example is most illustrative of which partial reinforcement schedule? (A) Fixed-ratio (B) Variable-ratio (C) Fixed-interval (D) Variable-interval

(A) Fixed-ratio *Fixed ratio (FR) schedules* reinforce a behaviour after a specific number of performances of that behaviour. Continuous reinforcement is an example of FR schedule. Giving your dog a treat after each successful attempt is an example of a fixed-ratio schedule. The ratio does not have to be 1:1, as in this scenario, to be a fixed-ratio.

Quick and abstract reasoning is _______ Intelligence, whereas accumulated knowledge and verbal skills is ______ Intelligence. (A) Fluid, Crystallized (B) Fluid, Fluid (C) Crystallized, Fluid (D) Crystallized, Crystallized

(A) Fluid, Crystallized

Which of the following situations best exemplifies Outsourcing? (A) Ford hires employees in Mexico to help build its cars and in turn closes down a United States-based factory. (B) Nike conducts a survey of Chinese citizens in order to discern which shoe products appeal to them the most. (C) Microsoft starts selling its products abroad in Sudan. (D) Apple pays another company for the use of Trademarked information, which becomes the property of Apple.

(A) Ford hires employees in Mexico to help build its cars and in turn closes down a United States-based factory. Outsourcing refers to the practice of producing inexpensive products by building factories and hiring workers abroad as is the case in this example.

Match each of the following psychologists with their theory of development: (A) Freud (B) Vygotsky (C) Erikson (D) Kohlberg (1) Psychosexual Development (2) Moral Development (3) Sociocultural Development (4) Psychosocial Development

(A) Freud --> (1) Psychosexual Development (B) Vygotsky -> (3) Sociocultural Development (C) Erikson -> (4) Psychosocial Development (D) Kohlberg -> (2) Moral Development

Jack started hunting to provide food 20 years ago, but continues to do so today despite plenty of food and lack of necessity, purely out of enjoyment. This can be best explained by ______________, a major part of Allport's trait theory which describes how a behaviour continues despite satisfaction of the drive that originally caused that behaviour. (A) Functional autonomy (B) Cardinal traits (C) Cognitive dissonance (D) Functional determinism

(A) Functional autonomy Jack started hunting to provide food 20 years ago, but continues to do so today despite plenty of food and lack of necessity, purely out of enjoyment. This can be best explained by *functional autonomy*, a major part of Allport's trait theory which describes how a behaviour continues despite satisfaction of the drive that originally caused that behaviour.

The leadership team at Apple meets to discuss the new Apple Watch. Some members come to the meeting thinking that perhaps pink would be an awesome color for the new wrist band. By the end of the meeting, every team member strongly agrees that pink is the way to go. Which term best describes the leadership team's behavior? (A) Group Polarization (B) Conformity (C) Groupthink (D) Confirmation Bias

(A) Group Polarization The team members left the meeting with a stronger conviction than before in the direction of the average group member's original opinion. This is in line with Group Polarization.

You are working on your homework and although you originally noticed the clock ticking in the background, you've stopped noticing it for the past hour. This example is most closely aligned with which Learned Behavior? (A) Habituation (B) Classical Conditioning (C) Operant Conditioning (D) Insight Learning

(A) Habituation *Habituation* is the process of diminishing an innate response to a frequently repeated stimulus. You no longer noticing the clock is a form of habituation.

_____________ frequency sounds activate the base of the cochlea, while _____________ frequency sounds activate the apex of the cochlea. (A) High, Low (B) High, High (C) Low, High (D) Low, Low

(A) High, Low

Which of the Cluster B (dramatic, emotional and erratic) Personality disorders is characterized by attention seeking and displaying a lot of outward emotions? (A) Histrionic (B) Borderline (C) Antisocial (D) Narcissistic

(A) Histrionic Histrionic Personality Disorder is characterized by attention seeking and displaying a lot of outward emotions - very extroverted, can be seductive.

Order the 3 types of nerve fibers from from fastest to slowest: I. A-beta fibers (A-B) II. C fibers III. A-delta fibers (A-D) (A) I > II > III (B) III > I > II (C) II > I > III (D) I > III > II

(A) I > II > III A beta fibers have the highest conduction velocity due to largest diameter, decreasing resistance; A-gamma follow, and C-fibers have the slowest conduction

In free recall, the serial position effect says subjects are more likely to remember the beginning and end of a list than the middle. Which of these effects is not a cause of the serial position effect? I. Context Clues II. Primacy Effect III. Recency Effect (A) I Only (B) I and II Only (C) I, II and III (D) II and III Only

(A) I Only NOT a cause

Who of the following can be considered Gestalt Psychologists? I. Solomon Asch II. Carl Jung III. Stanley Milgram (A) I Only (B) I and II Only (C) I and III Only (D) II and III Only

(A) I Only Solomon Asch was a Gestalt Psychologist.

In the analogy of the iceberg, which parts of our mind are completely below the surface of the water (only part of our unconscious)? I. Id II. Ego III. Superego (A) I Only (B) I and II Only (C) II and III Only (D) I, II, and III

(A) I Only The Id is described as the most unconscious and earliest developed part of our mind which has a pleasure principle under which it operates.

Which of the following combination of activities can a child in the preoperational stage perform? I. Symbolic/Pretend Play II. Reversible actions III. Mental math (A) I only (B) I and II only (C) II and III only (D) I, II and III

(A) I only A child in the preoperational stage can perform symbolic/pretend play. Both reversible actions and mental math are traits of operational thought, which are not developed until later.

Which of the following has been noticed in studies of patients with depression? I. Decreased Frontal Lobe activity II. Increased Temporal lobe activity III. Decreased Occipital lobe activity (A) I only (B) III only (C) I and II only (D) I and III only

(A) I only Studies have noted abnormally decreased frontal lobe activity and increased limbic system activity in depressive symptoms or disorders. These are consistent with other findings relating to emotions and stress.

Put the following 5 steps of the phototransduction cascade in order: I. The retinal goes from 11-cis to all trans, changes conformation of retinal and rhodopsin. II. Sodium channels close without cGMP bound, rod hyperpolarizes, rod inactivated. III. Light hits retinal in the rhodopsin (rod opsin). IV. Transducin (a GPCR) will break from rhodopsin, and the alpha subunit binds to Phosphodiesterase (PDE). V. PDE converts cGMP->GMP. (A) III > I > IV > V > II (B) III > V > IV > I > II (C) II > V > IV > I > III (D) II > I > IV > V > III

(A) III > I > IV > V > II The 5 steps of the phototransduction cascade in order: Light hits retinal in the rhodopsin (rod opsin). > The retinal goes from 11-cis to all trans, changes conformation of retinal and rhodopsin. > Transducin will break from rhodopsin, and the alpha subunit binds to Phosphodiesterase (PDE). > PDE converts cGMP->GMP. > Sodium channels close without cGMP bound, rod hyperpolarizes and thus is inactivated.

Jack always wears black to look like his favorite Green Day band members. He is demonstrated which of the following? (A) Identification (B) Compliance (C) Conformity (D) Internalization

(A) Identification Jack is doing what he is doing because he respects and looks up to Green Day. This is what is seen in Identification.

The "Invisible Gorilla" is a famous experiment where watchers are asked to track a ball in a film, and more than half will miss a gorilla walking across the screen! This experiment best demonstrates which principle? (A) Inattentional blindness (B) Change blindness (C) Intentional blindness (D) Momentary blindness

(A) Inattentional blindness Check out the video here!: https://www.google.com/url?sa=t&rct=j&q=&esrc=s&source=video&cd=1&cad=rja&uact=8&ved=0ahUKEwj-1Jzcy5PWAhWqq1QKHXaOBlsQtwIIKDAA&url=https%3A%2F%2Fwww.youtube.com%2Fwatch%3Fv%3DvJG698U2Mvo&usg=AFQjCNFlaWi_8quTj_ffq-FAOUodKRZMhg

What is the definition of conformity?

Conformity is the tendency of people to bring their behaviour in line with group norms.

When you fail a test and blame it on yourself, this is an example of exhibiting ______________________, whereas blaming the teacher or system would be exhibiting __________________. (A) Internal locus of control, external locus of control (B) External locus of control, internal locus of control (C) Internal locus of control, learned helplessness (D) External locus of control, reciprocal determinism

(A) Internal locus of control, external locus of control

Which of the following individuals best exemplifies the Authoritarian Personality Type? (A) Jack is obedient to his superiors. (B) Jill sees her supervisors as jerks. (C) Mark never listens to his dad, but he will readily listen to his friends. (D) Mary refuses to do anything anyone says.

(A) Jack is obedient to his superiors. The Authoritarian Personality Type is characterized by being obedient to one's superiors yet being oppressive to those beneath them. They are also rigid and inflexible in their thoughts and ways. It is thought that these people had a harsh and very disciplined childhood. Use prejudice as a coping mechanism.

Which of the following is the least likely way that someone could develop Dissociative Identity Disorder (DID)? (A) Janelle's mom suffered from DID, and Janelle slowly became paranoid she would develop DID until the point where she did. (B) After spending grueling months studying for the LSAT and finally getting an offer to practice, the lawyer is thrust into a high-profile case right away as a "trial" before he is offered a contract. (C) As a child, Charlie was frequently abused by his stepdad. (D) Mike feels mistreated by all those around him.

(A) Janelle's mom suffered from DID, and Janelle slowly became paranoid she would develop DID until the point where she did. DID is known to be caused by extreme stress, with a significant portion of those being child abuse-related. Being paranoid about familial health is least likely to cause that level of stress.

What refers to a type of brain activity that suppresses cortical arousal, while also helping sleep based memory formation? This type of activity can occur naturally or by gently touching someone while they are sleeping. (A) K-Complexes (B) non-REM sleep (C) REM sleep (D) Sleep spindles

(A) K-Complexes K-Complexes refer to a type of brain activity that suppresses cortical arousal, while also helping sleep based memory formation.

Which of the following brain disorders is caused by a thiamine deficiency in the brain? (A) Korsakoff's syndrome (B) Parkinson's disease (C) Alzheimer's disease (D) Agnosia

(A) Korsakoff's syndrome Korsakoff's syndrome is the result of thiamine deficiency in the brain. It is marked by retrograde and anterograde amnesia, and even confabulation (fabiracted, distorted or misinterpreted memories).

When you are hungry, your ______________ will signal to you that you are hungry. Once you've eaten enough and are full, your ____________ will signal to you to stop eating. (A) Lateral Hypothalamus, Ventromedial Hypothalamus (B) Ventromedial Hypothalamus , Lateral Hypothalamus (C) Ventromedial Hypothalamus , Ventromedial Hypothalamus (D) Lateral Hypothalamus, Lateral Hypothalamus

(A) Lateral Hypothalamus, Ventromedial Hypothalamus When you are hungry, your Lateral Hypothalamus will signal to you that you are hungry. Once you've eaten enough and are full, your Ventromedial Hypothalamus will signal to you to stop eating.

Jimmy sees an old lady walking down the street and says to him mom, "Look at that old fart." His mom reprimands him and says, "Jimmy! You are aging too. We all are. It is something that happens throughout our entire lives. You and that nice old lady have more in common than you think! She was once a young child like yourself." Jimmy's Mom's remarks are most similar to which theory?

(A) Life Course Theory Life Course Theory looks at aging as a process that takes place over a long period of time. It looks at events as resulting from one's past history and examines their entire life instead of just the current situation as is the case with Jimmy's Mom.

While taking illicit drugs, Mary-Kate experienced a state of euphoria. Which of the following nervous system structures was most likely affected? (A) Limbic System (B) Frontal Cortex (C) Lateral Hypothalamus (D) Ventromedial Hypothalamus

(A) Limbic System The limbic system is known for controlling emotion, cognition, motivation and pleasure, and is a common target for illicit drugs.

Which of the following terms refers to the form of memory that holds almost limitless amounts of information, including implicit and explicit memories? (A) Long-term Memory (B) Sensory Memory (C) Working Memory (D) Iconic Memory

(A) Long-term Memory Long-term Memory can hold almost limitless amounts of information, and includes implicit (or non-declarative) and explicit (or declarative) memories.

Elizabeth has always dreamed about being a doctor, but she is not studying hard for the MCAT because she needs to work her hourly job at Walmart in order to pay the bills and afford groceries. Which theory of motivation is best able to explain Elizabeth's situation? (A) Maslow's Hierarchy (B) Evolutionary Approach (C) Cognitive Approach (D) Optimal Arousal Theory

(A) Maslow's Hierarchy According to Maslow's Hierarchy of Needs, we fulfill basic needs before higher-level needs just as Elizabeth is doing in this situation.

During the Salem Witch Trials, the majority of the population believed that witches should be hunted and killed. Which type of Collective Behavior is best exhibited by this example? (A) Mass Hysteria (B) Fad (C) Riot (D) Rebellion

(A) Mass Hysteria Mass Hysteria occurs when there is a widespread delusion amongst a large group of people as was the case with the Salem Witch Trials.

Hannah writes an essay on Social Movements. In her essay she paints a dark picture of Social Movements, seeing them as dangerous and only useful as a refuge for those who felt they didn't belong elsewhere. Hannah's Essay lends support to which theory of Social Movements? (A) Mass Society Theory (B) Resource Mobilization Theory (C) Relative Deprivation Theory (D) Rational Choice Theory

(A) Mass Society Theory Mass Society Theory has a similar view to the one taken by Hannah in her essay, seeing them as dark, dangerous, and irrational.

Which type of neurons are hypothesized to be responsible for our ability to imitate? (A) Mirror Neurons (B) Efferent Neurons (C) Afferent Neurons (D) Interneurons

(A) Mirror Neurons Mirror Neurons are hypothesized to be responsible for our ability to imitate because they cause us to have similar brain activity to when we watch someone do something to when we actually do that same thing ourselves.

Which sleep stage is between sleep and wakefulness? This stage of sleep produces Theta Waves, and one may experience hypnagonic hallucinations (imagining things that are not there), or hypnic jerks (feeling of falling). (A) N1 (B) N2 (C) N3 (D) REM

(A) N1 N1 is the sleep stage between sleep and wakefulness. This stage of sleep produces Theta Waves, and one may experience hypnagonic hallucinations (imagining things that are not there), or hypnic jerks (feeling of falling).

Another three-year-old has not acquired any language abilities yet, and the specialist keeps referring to a Language Acquisition Device in the child's brain. Which theory of Language Development would this specialist most agree with? (A) Nativist approach (B) Learning reinforcement (C) Interactionist approach (D) Universal approach

(A) Nativist Approach The Nativist Approach claims infants have a Language Acquisition Device (LAD) in their brains to allow them to learn and understand any language, and that there is a critical/sensitive period where it is easiest to learn a language. That LAD only operates during that sensitive period.

Unlike in past years, today, women's cricket has evolved and become much more prevalent. We have the Womens World Cup, Women's t20 WC, and many more forms of the game that includes women in a sport previously only played by men. What describes this situation best? (A) Norms evolving over time to promote women's cricket (B) Sanctions being lifted on women's cricket (C) A previously known taboo is no longer so (D) Laws prohibiting womens cricket were lifted

(A) Norms evolving over time to promote women's cricket

Zach is extremely obese and loves to eat, at which stage did Zach become fixated? (A) Oral (B) Anal (C) Phallic (D) Genital

(A) Oral Fixation at the Oral stage is characterized by things related to the mouth such as smoking, biting one's nails, or overeating.

Which of the "Odd and Eccentric" Personality Disorders has the defining characteristic of distrust and suspicion of others? (A) Paranoid (B) Schizotypal (C) Schizoid (D) Both B and C

(A) Paranoid

Jack sees himself as a pretty agreeable person. He rarely gets upset or starts fights with others. He gets a new boss at work one day, and feels like his boss is pushing him around and treating him like a slave. Instead of confronting his boss, Jack underperforms on key projects in order to frustrate his boss. Which defense mechanism is Jack most likely implementing? (A) Passive Aggression (B) Displacement (C) Projection (D) Altruism

(A) Passive Aggression Jack is getting back at his boss without direct confrontation as is seen in Passive Agression. It is often carried out by failing to do something. It is an indirect or passive way of expressing one's feelings.

Which of the following hormone classes vary the most in size, are made in the Rough ER, and are later packaged in the Golgi before vesicular release? (A) Polypeptides/proteins (B) Steroids (C) Tryptophan derivatives (D) Tyrosine derivatives

(A) Polypeptides/proteins These polypeptides/proteins can vary from a few amino acids to very large proteins. Proteins are made in the Rough ER, and these hormones are packaged in the Golgi before vesicular release.

Every time you play your drumset really loud, your neighbor comes and knocks on the door to tell you to stop playing so loud. Which of the following Operant Conditioning terms is this an example of? (A) Positive Punishment (B) Negative Punishment (C) Positive Reinforcement (D) Negative Reinforcement

(A) Positive Punishment Your neighbor telling you to stop is an example of Positive Punishment because your neighbor is an unpleasant stimulus being added to the situation.

Jimmy wants some candy. When his teacher is handing it out, he asks the teacher if he can have a piece to bring home to his sister who has the flu. He eats the candy on his way home. Which stage of moral development is Jimmy currently in? (A) Pre-conventional (B) Conventional (C) Post-conventional - Social Contract (D) Post-conventional - Universal Principles

(A) Pre-conventional Jimmy is demonstrating Self-interest which is the second level of the Pre-conventional Stage.

Sarah says that you should never ever ever hurt someone else because you might get put in time out. Which stage of moral development is Sarah currently in? (A) Pre-conventional (B) Conventional (C) Post-conventional - Social Contract (D) Post-conventional - Universal Principles

(A) Pre-conventional Sarah sees doing the right thing as right because it allows you to avoid punishment. This is in line with the first level of the Pre-conventional Stage.

In an experiment, researchers found that they were unable to train racoons to put coins into a piggy bank via operant conditioning because they would take coins back out and rub them together. This seemed to be due conflicting with their natural behaviour, or _____________________. This difficulty in overcoming instinctual behaviour is known as _________________. (A) Preparedness, instinctual drift (B) Latent learning, Problem solving (C) Preparedness, Problem solving (D) Preparedness, instinctive drive

(A) Preparedness, instinctual drift In an experiment, researchers found that they were unable to train racoons to put coins into a piggy bank via operant conditioning because they would take coins back out and rub them together. This seemed to be due conflicting with their natural behaviour, or *preparedness (predisposition)*. This difficulty in overcoming instinctual behaviour is known as *instinctual drift*.

Prejudice can form in response to dissimilarities amongst groups, races, ethnicities, even environments. They may be kept internally or shared with the larger community. _______________ is a common way large organizations/political groups attempt to create prejudices in others, for example, view of Americans in North Korea and vice versa. (A) Propaganda (B) Racial discrimination (C) Institutional discrimination (D) Ethnocentrism

(A) Propaganda *Propaganda* is a common way large organizations/political groups attempt to create prejudices in others, for example, view of Americans in North Korea and vice versa.

Which of the following is not a category of symptoms often expressed in schizophrenics? (A) Psychotic (B) Cognitive (C) Negative (D) Positive

(A) Psychotic Schizophrenia is a subset of psychotic disorders, not the other way around. Symptoms expressed in Schizophrenia fall under the subcategories of Cognitive, Negative and Positive

Christy sees her father die before her very eyes. Whenever she talks about the problem with a therapist, she starts sucking her thumb as a way to comfort herself. Which defense mechanism is Christy most likely implementing? (A) Regression (B) Suppression (C) Repression (D) Denial

(A) Regression Christy is resorting to behavior that is characteristic of an earlier stage of development as is seen in Regression.

Which of the following is not a coping mechanism for Cognitive Dissonance? (A) Removing the Cognitions (B) Modifying the Cognitions (C) Trivializing the Cognitions (D) Denying the relation between the Cognitions

(A) Removing the Cognitions The main strategies of reconciling cognitive dissonance is modifying the cognitions, trivializing the cognitions, or adding more cognitions to confound the first two, and denying the relation between the cognitions.

Jack is a medical student. He is trying his best to act like a doctor. Over time, he starts to truly feel like a doctor. Which theory best explains Jack's change? (A) Role-Playing (B) Door-in-the-Face (C) Foot-in-the-Door (D) Situational Approach

(A) Role-Playing Jack is role-playing being a doctor until he starts feeling like one.

If you are looking for supplies at a bookstore, and you are trying to learn your list by simply repeating the items over and over again, which encoding strategy are you using? (A) Rote Rehearsal (B) Chunking (C) Pegword Mnemonic Device (D) Method of Loci

(A) Rote Rehearsal It is also the weakest method.

Which example best illustrates the difference between Front Stage and Back Stage Self? (A) Sarah loves to sing in the shower, but not in front of others. (B) Jim is better at sports he has practiced than those he hasn't in front of others. (C) Gary feels anxious about meeting new people but he is very comfortable around friends. (D) Elizabeth doesn't like people to be around her in the mornings because she isn't ready for the day.

(A) Sarah loves to sing in the shower, but not in front of others. Front Stage Self is how we act when others are around as Sarah does not sing with others around. Back Stage Self is how we act when others are not around as Sarah does when she sings in the shower. This ideology is known as *impression management*, introduced by *Erving Goffman's dramaturgical theory* - describes how we try to manage the way others see us..

Lexy has strong violent urges to hurt other people. Instead of hurting others, though, she lets out her feelings through art. Which defense mechanism is Lexy most likely implementing? (A) Subilimation (B) Humor (C) Rationalization (D) Intellectualization

(A) Subilimation Lexy is channeling her negative feelings into a more positive activity, which is what happens in Sublimation.

Software companies bought up land in Silicon Valley, and as they did so many people began to move away from the city to this new area, creating organized neighborhoods. This is an example of which of the following? (A) Suburbanization (B) Urban Sprawl (C) Urban Decline (D) Urban Renewal

(A) Suburbanization Suburbanization refers to the movement away from a city into a nearby smaller neighborhood as is the case here.

Which of the following statements about classification of mental disorders is incorrect? (A) The ICD and DSM rarely change, and only change if new research demands that disorders must be reclassified. (B) There are two main systems for classifying mental disorders: the ICD and DSM. (C) The American Psychiatric Association and World Health Organization group mental disorders differently (D) The DSM-5 has more top-level categories, meaning it breaks down mental disorders into more categories.

(A) The ICD and DSM rarely change, and only change if new research demands that disorders must be reclassified. The ICD and DSM are updated regularly, for a variety of reasons!

Margot Robbie is one of the most attractive actresses in Hollywood today. People often see her as perfect and exaggerate any action or small thing she does. Similar is seen with Emilia Clarke from Game of Thrones, who is also considered very attractive. This is an example of ____________________. (A) The halo effect (B) Attribution theory (C) Implicit personality theory (D) Reliance on central traits

(A) The halo effect Margot Robbie is one of the most attractive actresses in Hollywood today. People often see her as perfect and exaggerate any action or small thing she does. Similar is seen with Emilia Clarke from Game of Thrones, who is also considered very attractive. This is an example of *the halo effect*.

Which of the following is not a cause of controversy regarding Dissociative Identity Disorder? (A) The media have taken an undocumented disorder, DID, and propagated it to the point where even the DSM has to address it. (B) Therapists may support the line of thinking that can lead to DID, causing some occurrences accidentally. (C) As rare as DID is in the USA, it is rare almost to the point of not existing elsewhere, making DID appear like a social construct. (D) People play different roles in different situations, so this could just be an extreme case brought on by stress.

(A) The media have taken an undocumented disorder, DID, and propagated it to the point where even the DSM has to address it. There are real, documented cases of Dissociative Identity Disorder. Options (B), (C), and (D) are all real controversies regarding DID.

In deciding whether or not she should go on a trip, Eliza decides not to go because she feels that the weather is out of her control and may ruin her vacation. Which attitude theory best explains Eliza's thought process? (A) Theory of Planned Behavior (B) Elaboration Likelihood Model (C) Attitude-to-behavior Process Model (D) Prototype Willingness Model (PWM)

(A) Theory of Planned Behavior The Theory of Planned Behavior is focused on people making decisions based on the implications and intentions of their actions. Those intentions are affected by 3 factors, including perceived behavior control, and in this case Eliza feels she does not have control over her vacation's weather.

If a three-year-old has a withdrawn temperament, which of the following is most likely true for the same individual in adulthood? (A) They are more likely to stay at home on a Friday night to read a book. (B) They are going out with a small group of friends but need constant attention from friends. (C) They are going to end up at a party and have fun, no matter what happens there. (D) There is almost no consistency in a child's temperament and their actions as an adult.

(A) They are more likely to stay at home on a Friday night to read a book. A child's temperament persists in most cases through adulthood, so this individual would likely do an introverted, withdrawn action.

Use the following information to calculate the Population Growth Rate: Birth Rate = 5.2 Fertility Rate = 3.1 Mortality Rate = 9.8 Emmigration Rate = 4.1 Immigration Rate = 6.3 (A) -4.5 (B) -2.4 (C) 2.4 (D) 4.5

(B) -2.4 Population Growth Rate = (Birth Rate + Immigration Rate) - (Mortality Rate + Emmigration Rate) Population Growth Rate = (5.2 + 6.3) - (9.8 + 4.1) Population Growth Rate = (11.5) - (13.9) Population Growth Rate = -2.4

In a given community, there are people of only 7 different ages. Calculate the Dependency Ratio given the population of each age: 5 year olds: 132,869 11 year olds: 342,987 15 year olds: 1,496,932 25 year olds: 9,864,234 54 year olds: 2,435,312 67 year olds: 4,897,013 78 year olds: 734,896 (A) .23 (B) .44 (C) .61 (D) .74

(B) .44 Dependency Ratio = (population that is 14 and under + 65 and older)/(population that is 15 to 64 years of age) Dependency Ratio = (132,869 + 342,987 + 4,897,013 + 734,896)/(1,496,932 + 9,864,234 + 2,435,312) Dependency Ratio = approx. 6,000,000 / 14,000,000 (actual: 6,107,765 / 13,796,478) Dependency Ratio = approx. a little less than .5 (actual: 0.44)

About ______% of adults meet the criteria of at least one mental illness, and ____% of adults meet the criteria for at least one serious mental illness causing distress or disability. (A) 25, 10 (B) 25, 6 (C) 10, 6 (D) 35, 10

(B) 25, 6 About 25% of adults meet the criteria of at least one mental illness, and 6% of adults meet the criteria for at least one serious mental illness.

If a known trait is greatly impacted by genetic differences, which of the following scenarios will have the most different expression of the traits? (A) A child and their biological parents (B) A child and their adopted parents (C) Monozygotic twins raised separately (D) Dizygotic twins raised together.

(B) A child and their adopted parents If the trait's variance is genetically-backed, then a child and their adopted parents (which do not necessarily share any genetic material) will have the most variance. A child will have 50% of each of their parent's genetic material, and dizygotic twins also share about 50% of their genetic material. Monozygotic twins share all of their genetic material.

Where is adrenaline produced? (A) The Kidney (B) A gland on top of the kidney (C) The Liver (D) A gland on top of the liver

(B) A gland on top of the kidney Adrenaline is produced by the Adrenal Gland, which is located on top of the kidneys.

Thompson was quite rowdy as a young man. Now that he is older and has taken on more responsibility as a superior in his community he has settled down and is contributing more to society. Thompson's situation is most similar to which theory? (A) Life Course Theory (B) Age Stratification Theory (C) Activity Theory (D) Disengagement Theory

(B) Age Stratification Theory The Age Stratification Theory looks at age as a way of hierarchically organizing a society. There is a sense of role and prestige associated with one's age. It can be detrimental though when age is a source of inequality such as with Ageism. In this case, Thompson is experiencing a change in his role due to his age.

If there is a lesion in the left optic nerve before the optic chiasm, how would vision be affected? (A) All information from the right eye would be compromised. (B) All information from the left eye would be compromised. (C) All information from the right visual field would be compromised. (D) All information from the left visual field would be compromised.

(B) All information from the left eye would be compromised. If there were a lesion in the left optic nerve BEFORE the optic chiasm (where visual fields cross), then the information from the left eye would not go through.

On an electroencephalogram (EEG), ___________ waves have a frequency of 8-13 Hz and are present when one is in a daydreaming state, while ___________ waves have a frequency of 0.5-3 Hz and are seen most often when one is in a coma or deep sleep. (A) Beta waves, Delta Waves (B) Alpha waves, Delta Waves (C) Beta waves, Theta Waves (D) Alpha waves, Theta Waves

(B) Alpha, Delta

Sarah is extremely orderly and can't stand her email inbox to be unorganized, at which stage did Sarah become fixated? (A) Oral (B) Anal (C) Phallic (D) Genital

(B) Anal Fixation at the Anal stage is characterized by being overly neat or overly messy.

The Broca's area and Wernicke's area are connected by the ____________________, a bundle of axons that allows proper association between language comprehension and speech production. (A) Synaptic plasticity (B) Arcuate Fasciculus (C) Wernicke's axon (D) Spinal duct

(B) Arcuate Fasciculus (synaptic plasticity is a different concept relating to memory, the other two are entirely made up)

A child shows little or no preference between the caregiver and stranger, and barely any emotions (distress/relief) when the caregiver leaves and returns. This is due to the caregiver having little to no response to a distressed child. What sort of attachment is this describing? (A) Secure attachment (B) Avoidant attachment (C) Ambivalent attachment (D) Disorganized attachment

(B) Avoidant attachment *Avoidant attachment* results when a caregiver has little to no response to a distressed child. These children seem to have no preference between the caregiver and a stranger - little emotions (distress and relief) shown when caregiver leaves and returns.

Which of the following is not one of Carl Rogers' three components of self-concept? (A) Self-Esteem (B) Awareness of self (C) Self-Image (D) Ideal Self

(B) Awareness of self

Which type of depressant is known as tranquilizers, and is used to induce sleep or reduce anxiety, however is often not prescribed because of negative side effects such as reduced memory, judgement, concentration and addiction? (A) Benzodiazepines (B) Barbiturates (C) Alcohol (D) Marijuana

(B) Barbituates Barbituates were historically used as anxiety reducing (anxiolytic) and sleep medications, but have been replaced with benzos due to negative side effects and addiction.

Which type of depressant is the most commonly prescribed suppressant that is used to treat insomnia, anxiety and seizures, acting as an anticonvulsant? (A) Barbiturates (B) Benzodiazepines (C) Alcohol (D) Methadone

(B) Benzodiazephines Benzos include alprazolam, diazepam, and clonazepam. Like barbituates, benzos are highly addictive

On an electroencephalogram (EEG), _________ waves have the highest frequency (12-30 Hz) and are are seen when alert. __________ waves are closer to 4-7 Hz are seen most often in drowsiness, early sleep, and especially in children. (A) Alpha waves, Delta waves (B) Beta waves, Theta waves (C) Delta waves, Alpha waves (D) Theta waves, Beta waves

(B) Beta, Theta

After sleeping through her alarm on her Biology test day, Kourtney's body releases a hormone to redistribute glucose in her body. This same hormone suppresses her immune system, and she develops a cold later that week. Which hormone most likely caused these changes? (A) Epinephrine (B) Cortisol (C) Oxytocin (D) Norepinephrine

(B) Cortisol Cortisol is a glucocorticoid that redistributes glucose and suppresses the immune system as part of the "fight or flight" response.

Hunger is one of the strongest natural motivations, yet people often eat for sheer pleasure. Some cultures have diets traditionally high in fat, and social activities involving food. In other cultures, anorexia has become more prevalent and is related to personality disorders. Which of the following best explains these? (A) Motivation stemming from taste bud reward response (B) Biological and sociocultural factors impact motivation (C) Expectancy-value theory of motivation (D) Maslow's Hierarchy of Needs

(B) Biological and sociocultural factors impact motivation Biological and sociocultural factors can impact motivation. Sociocultural example is the traditions where high-fat diets are prevalent + social activities involving food (example: Pakistani weddings or gatherings). Biological example is anorexia - sufferers are more likely to suffer from personality disorders, indicating biological interactions leading to the condition.

Which of the Cluster B (dramatic, emotional and erratic) Personality disorders is characterized by having very unstable emotions, self-image and relationships, and being very impulsive? (A) Histrionic (B) Borderline (C) Antisocial (D) Narcissistic

(B) Borderline Borderline Personality Disorder is characterized by having unstable emotions, self-image and relationships and being very impulsive. 2x more common in women, high suicide and self-harm rates.

Synaptic plasticity allows neurons to improve their connectivity and strength. How does repeatedly firing a neuron increase its signaling strength? (A) By triggering the growth of new neurons (cell division) (B) By converting the pre-synaptic signals into more effective, stronger synapses (Long term potentiation) (C) By changing which signaling molecules are released by the pre-synaptic neuron, the post-synaptic ion channels can be more activated (D) By sensitizing the synapse to decrease the distance between the axon terminal and dendrite

(B) By converting the pre-synaptic signals into more effective, stronger synapses (Long term potentiation)

This article details the stabbing of two men that tried to stand up to a man that was yelling hateful remarks on a train: http://www.npr.org/sections/thetwo-way/2017/05/27/530351468/2-dead-1-injured-after-stabbing-in-portland-ore The two men that got stabbed for standing up to the man stand in stark contrast to which sociological phenomenon? (A) Social Loafing (B) Bystander Effect (C) Diffusion of Responsibility (D) Deindividuation

(B) Bystander Effect Instead of just standing by and watching like the other people, these two men stood up to the hateful man. This stands in stark contrast to the Bystander Effect in which people do nothing but stand by and watch when they are in a big group/crowd.

In one experiment, a man will ask someone on a busy street for directions. Some interruption will distract the direction giver, and the asker will be replaced by a different person, and most direction givers won't notice. This experiment best demonstrates which principle? (A) Inattentional blindness (B) Change blindness (C) Intentional blindness (D) Momentary blindness

(B) Change blindness Change blindness is failing to notice changes between a previous and current state

If you are trying to memorize which lights in a grid are lit, and the top row is lit, you may remember each of those lights by remembering "top row lit". Which encoding strategy is used here? (A) Rote Rehearsal (B) Chunking (C) Pegword Mnemonic Device (D) Method of Loci

(B) Chunking

Imagine a Great Gatsby-sized party. Through all of those voices, you can focus on the one you are conversing with until you hear your name from across the room, switching your attention. What does this scenario best demonstrate? (A) Primacy Effect (B) Cocktail Party Effect (C) Descriptivist Theory of Names (D) Change Blindness

(B) Cocktail Party Effect This would be an endogenous cue, because it was the meaning of your name that drew your attention!

Psychological and situational factors can influence aggression. Which of the following states that we are more likely to respond to others aggressively when feeling negative emotions, such as pain, fatigue, sickness, frustration? (A) Anxious-ambivalent model (B) Cognitive neoassociation model (C) Self-disclosure model (D) Cognitive dissonance model

(B) Cognitive neoassociation model The *cognitive neoassociation model* states that we are more likely to respond to others aggressively when feeling negative emotions, such as pain, fatigue, sickness, frustration. The others are made up.

Jakob spends a lot of time with gangsters. Soon his parents start to notice that Jakob is dressing differently, doing drugs, and engaging in new illicit behaviors. Which term best describes Jakob's behavior? (A) Group Polarization (B) Conformity (C) Group think (D) Confirmation Bias

(B) Conformity Conformity is known colloquially as "peer pressure" and is the tendency for individuals to bring their behavior in line with group norms. Jakob is conforming to be like his new group of friends.

When making an attribution concerning his professor, Jack notices that Dr. Marks wears a red shirt every Tuesday without fail, representing a _____________ cue. He also notices that Dr. Marks doesn't wear a red shirt on any other day of the week, representing a _____________ cue. Lastly, he realizes that Dr. Marks is the only professor on campus who acts this way, representing a _______________ cue. (A) Consensus, Consistency, Distinctiveness (B) Consistency, Distinctiveness, Consensus (C) Distinctiveness, Consistency, Consensus (D) Consensus, Distinctiveness, Consistency

(B) Consistency, Distinctiveness, Consensus When making an attribution concerning his professor, Jack notices that Dr. Marks wears a red shirt every Tuesday without fail, representing a Consistency cue. He also notices that Dr. Marks doesn't wear a red shirt on any other day of the week, representing a Distinctiveness cue. Lastly, he realizes that Dr. Marks is the only professor on campus who acts this way, representing a Consensus cue.

When Phil's Wife dies at the age of 65, he decides to start dating again and ends up remarrying. He finds this relationship to be satisfying as it fills the need in his life for companionship. This most closely relates to which theory? (A) Life Course Theory (B) Continuity Theory (C) Activity Theory (D) Disengagement Theory

(B) Continuity Theory Continuity Theory suggests that we try to maintain the same foundational structure throughout our lives in order to maintain consistency and to meet our needs. In this case, Phil is meeting his need for companionship.

Which theory takes the concept of cues one step further by taking into account intentions of peoples' behaviours? (A) Fundamental attribution theory (B) Correspondent inference theory (C) James functionalist theory of attribution (D) Attribution intention theory

(B) Correspondent inference theory The *correspondent inference theory* takes the concept of cues one step further by taking into account intentions of peoples' behaviours. The other three are made up.

Which structure, lined with tiny holes, separates the olfactory epithelium from the human brain? (A) Glomerulus (B) Cribriform Plate (C) Amygdala (D) Vomeronasal System

(B) Cribriform Plate The Cribriform Plate is lined with tiny holes and separates the olfactory epithelium from the human brain.

In almost every country in the world, there is a ceremony that is associated with marriage/partnership. Which concept does this example best relate to? (A) Gate Keeping (B) Cultural Universals (C) Tokenism (D) Agent of Socialization

(B) Cultural Universals Cultural Universals are elements, patterns, traits, or institutions that are common among ALL human cultures as in the case here with marriage/partnership ceremonies.

After falling behind on MCAT preparation, Janelle has worked under intense stress the last three weeks. Which of the following is not a symptom she could experience from chronic stress? (A) Suppressed immune system (B) Decreased blood sugar (C) Hypertension (D) Reproductive issues

(B) Decreased blood sugar Chronic stress would likely lead to increased blood sugar

Violent behaviour in crowds and riots is considered *antinormative behaviour* explainable by the social phenomenon of ________________. (A) Bystander effect (B) Deindividuation (C) Social facilitation (D) Social loafing

(B) Deindividuation *Deindividuation* is a social phenomenon that occurs in group settings - it leads to a loss of identity and anonymity when in presence of a large group, causing dramatically different behaviour. In crowds and riots, this is antinormative behaviour (behaviour against the norm).

Sarah is in an abusive relationship with her husband. Instead of confronting him, however, she takes her frustration out on her children by bossing them around. Which defense mechanism is Sarah most likely implementing? (A) Passive Aggression (B) Displacement (C) Projection (D) Altruism

(B) Displacement Sarah is taking out her frustration on her husband on an easier target, her children. This is commonly seen in Displacement.

Formerly called Multiple Personality Disorder, this mental disorder is characterized by a person having multiple distinct personalities that will act in different mannerisms, including different emotional responses and denial of the other personalities. (A) Somatic symptom disorder (B) Dissociative Identity Disorder (C) Schizophrenia (D) Bipolar Disorder

(B) Dissociative Identity Disorder

Which of the following is a criticism of the learning theory of language development? (A) Does not involve a learning acquisition device (B) Does not explain how children can learn new words or form sentences they haven't heard before (C) It explains language acquisition by a conditioning factor and reinforcement (D) It lacks biological backing

(B) Does not explain how children can learn new words or form sentences they haven't heard before

Which of the following is NOT a way that the changing environment affects gene regulation? (A) Steroids like testosterone can enter the nucleus and affect transcription to up-regulate genes. (B) During development, a girl with two X chromosomes will silence the genes from one chromosome (C) Pheromones can excite the brain and turn on genes there. (D) All of the above examples exhibit gene regulation from a changing environment.

(B) During development, a girl with two X chromosomes will silence the genes from one chromosome Answer choice (B) is an example of Genomic Imprinting, and is not caused by the changing environment as the other examples are.

Max explains his desire to eat lasagna at dinner as resulting from his innate need for food. Which theory of motivation is Max relying upon to make this explanation? (A) Maslow's Hierarchy (B) Evolutionary Approach (C) Cognitive Approach (D) Optimal Arousal Theory

(B) Evolutionary Approach "innate need for food" = instinct The Evolutionary Approach focuses on the role instincts play in motivation. Max is explaining his motivation in terms of innate instincts that make him want to eat.

After some time, your mom stops giving you Amazon gift cards all the time, resulting in you no longer getting happy when she calls. This is an example of which of the following Conditioning terms? (A) Spontaneous Recovery (B) Extinction (C) Generalization (D) Discrimination

(B) Extinction Extinction refers to a loss of the Conditioned Response in response to the Conditioned Stimulus. This is what happened when your mom stopped giving you Amazon gift cards.

A conditioned response is not permanent. If the conditioned stimulus is presented without the unconditioned stimulus enough times, the organism can lose the conditioned response to it - this is called _______________. This is not permanent either, as after some time if appropriately presented, a weak conditioned response can be sometimes exhibited to the conditioned stimulus - this process is known as ________________. (A) Spontaneous recovery, extinction (B) Extinction, spontaneous recovery (C) Generalization, extinction (D) Generalization, discrimination

(B) Extinction, spontaneous recovery A conditioned response is not permanent. If the conditioned stimulus is presented without the unconditioned stimulus enough times, the organism can lose the conditioned response to it - this is called *extinction*. This is not permanent either, as after some time if appropriately presented, a weak conditioned response can be sometimes exhibited to the conditioned stimulus - this process is known as *spontaneous recovery*.

The Parvo Pathway is __________ detecting while the Magno Pathway is __________ detecting. (A) Form, Form (B) Form, Motion (C) Motion, Form (D) Motion, Motion

(B) Form, motion There are ~1 mil retinal ganglion cells - 80% are parvo type cells, 20% are magno type cells - *Parvo cells* detect colour, detail and form, and are numerous, small and slow conducting; *magno cells* process info about depth, motion and change, and are large and fast-conducting. Magno cells have larger receptive fields, but both respond to stimuli over limited area of the retina. Parvo = 4 dorsal projections; magno = 2 ventral projections.

An adolescent is presented with a nuanced story of how a lady stole medication she couldn't afford from a pharmacy to live. When asked about it, the adolescent understood it was a complex situation, not black and white. Which stage would this adolescent belong to? (A) Preoperational (B) Formal Operational (C) Concrete Operational (D) Sensorimotor

(B) Formal Operational Since the adolescent was able to examine the situation from multiple angles instead of simply right or wrong, they are clearly in that final stage of Cognitive Development: Formal Operational.

How can we define consciousness?

Consciousness can be described as one's level of awareness of themselves and their environments. Can be influenced by many factors including drugs. There are many stages of consciousness.

After running various intelligence tests on subjects, Charles Spearman found that students who score highly in one area tend to score very well in other categories too. Which intelligence theory does his data support? (A) Multiple Intelligences (B) General Intelligence (C) Triarchic Theory of Intelligence (D) Seven Types Theory of Intelligence

(B) General intelligence The overlap of high scores in many areas supports the existence of a "g factor", or a general intelligence factor.

Little Timmy feels like most of the girls at his school think he is cute. The girls at his school are most likely part of what concept in Timmy's life? (A) Vague Other (B) Generalized Other (C) Significant Other (D) Attractive Other

(B) Generalized Other They likely play into how Timmy thinks other people in general view him, which is most in line with the idea of the Generalized Other. A Significant Other would be someone like Timmy's best friend or his parents. Vague and Attractive Other are not sociological terms.

The _____________ is where various sensory olfactory cells, that are sensitive to the same molecule, synapse. (A) Mitral/Tufted Cell (B) Glomerulus (C) Olfactory Epithelium (D) Cribriform Plate

(B) Glomerulus The Glomerulus is where various sensory olfactory cells, that are sensitive to the same molecule, synapse.

What refers to the process your body takes to maintain body temperature, heartbeat, metabolism, and it can occur when the body is resting or even taking drugs? (A) Hallucination (B) Homeostasis (C) Inhalation (D) Relaxation

(B) Homeostasis

Which psychoanalytical theory postulated that individuals with neurotic personalities are governed by one of ten *neurotic needs*, each of which is designed to make life and interactions bearable? (A) Jung's psychoanalytical theory (B) Horney's psychoanalytical theory (C) Alder's psychoanalytical theory (D) Freud's psychoanalytical theory

(B) Horney's psychoanalytical theory The primary concept of Horney's theory is *basic anxiety*, which stems from inadequate parenting causing vulnerability, anxiety and helplessness; while neglect and rejection cause *basic hostility*

Jeremy's father recently passed away. Jeremy was extremely close to his father, but instead of getting upset or feeling bitter, he talks with his family about the good times he had with his dad, telling jokes and lightening the mood. Which defense mechanism is Jeremy most likely implementing? (A) Subilimation (B) Humor (C) Rationalization (D) Intellectualization

(B) Humor

Which form of an induced state of consciousness usually involves getting a person to relax and focus on their breathing, however becoming more susceptible to suggestion in this state? (A) Meditation (B) Hypnosis (C) Insomnia (D) Narcolepsy

(B) Hypnosis

Parenting Style is shown to make a big difference on a child's attachment to their parents. A healthy attachment to one's parents as a child will tend to influence which of the following down the road? I. Relationship with one's spouse II. Attachment Style of one's children III. Relationship with one's peers (A) I Only (B) I and II Only (C) I and III Only (D) I, II, and III

(B) I and II Only A healthy attachment to one's parents as a child will tend to influence one's relationship with their spouse and their children. Relationship with one's peers is not necessarily heavily influenced by one's attachment style.

People often compare familiar tasks to riding a bike. Which 2 of the following memory terms describe a skill like riding a bike? I. Implicit II. Non-Declarative III. Explicit (A) I Only (B) I and II Only (C) I and III Only (D) II and III Only

(B) I and II Only Implicit and Non-Declarative Memory are synonyms and describe riding a bike. Declarative and Explicit are also synonyms.

Innate behaviours are broken down into simple and complex types. Which of the following are examples of Simple Innate Behaviors? I. Reflexes II. Kinesis III. Migration (A) I Only (B) I and II Only (C) II and III Only (D) I, II, and III

(B) I and II Only Reflexes, Kinesis (random movement in response to stimulus), and Taxis (purposeful moving towards or away from stimuli) are examples of Innate Behaviors. Complex Behaviors include Fixed Action Patterns, Migration, and Circadian Rhythm.

Which of the pathways potentially involved in depression has an incorrect description of its function? I. The Raphe System is responsible for serotonin secretion II. Locus coeruleus responsible for dopamine and serotonin secretion. III. The VTA supplies much of the dopamine of the brain (A) I Only (B) II Only (C) I and II Only (D) II and III Only

(B) II Only The Locus Coeruleus is responsible for norepinephrine secretion. All three of these pathways begin in the brainstem and seem to have abnormalities in depressed patients. Raphe system for secreting serotonin (from Raphe nuclei), Locus coerulus for secreting norepinephrine, and the VTA (mesolimbic reward pathway) for secreting dopamine.

Order the following steps of a Cochlear Implant from first to last: I. Transmitter sends electrical impulse to the Receiver. II. Stimulator sends electrical impulse to the Cochlea which converts the electrical impulse to a neural impulse that is sent to the brain. III. Sound is converted to an electrical impulse by Speech Processor. IV. Receiver sends electrical impulse to the Stimulator. (A) I > III > IV > II (B) III > I > IV > II (C) II > IV > III > I (D) IV > II > III > I

(B) III > I > IV > II

Place the following steps in order from first to last: I. Signal synapses on to the glomerulus. II. Basal/apical cell sends axon through accessory olfactory epithelium to the accessory olfactory bulb. III. Pheromone molecule activates receptor on basal/apical cell. IV. Signal is sent to the amygdala where it is processed. V. Signal is sent to the mitral or tufted cell. (A) I > II > III > IV > V (B) III > II > I > V > IV (C) II > I > III > IV > V (D) III > II > I > IV > V

(B) III > II > I > V > IV (1) Pheromone molecule binds to and activates receptor on basal/apical cell (2) Basal/apical cell sends an axon through the accessory olfactory epithelium --> accessory olfactory bulb (3) The signal then synapses onto a glomerulus (cluster of nerve endings) (4) Signal is sent to a mitral or tufted cell (5) Signal is then sent to the amygdala for processing

Which of the following primary appraisal(s) will trigger the start of a secondary appraisal? I. Irrelevant II. Benign/ Positive III. Stressful (A) I only (B) III only (C) II and III (D) I, II and III

(B) III only Only a first appraisal of "stressful" will trigger a secondary appraisal.

_____________ refers to a social group with which you identify with/have a sense of belonging; ________________ refers to a social group with which you don't identify. (A) Out-group, in-group (B) In-group, out-group (C) Individualistic group; institutional group (D) Relative group; ethnocentric group

(B) In-group, out-group *In-group* refers to a social group with which you identify with/have a sense of belonging; *Out-group* refers to a social group with which you don't identify. Negative feelings about an out group usually stems from favouritism of the in-group.

In the final month before her PhD dissertation, Rebecca has been working long hours and feels her stress levels rising. Which of the following is not a suitable coping or management technique to reduce the effects of stress? (A) Exercise and meditation after work (B) Increasing her daily eating habits to account for her increased workload and energy usage (C) Having "perceived control" over her dissertation (D) Cognitive flexibility which allows her to change her perspective on the situation

(B) Increasing her daily eating habits to account for her increased workload and energy usage Increasing her eating does nothing to combat the real stressor or manage the stress that comes with it. Cognitive flexibility, perceived control, exercise and meditation are all appropriate coping and management techniques.

You have no idea whether or not jumping off a 30-story bridge is dangerous or not, but your friend group assures you that it is not dangerous at all. You believe your friends and jump off the bridge. This is an example of ____________ Influence. If you had known that it was dangerous, but went ahead and jumped as to not look like a wimp, that would be an example of ______________ Influence. (A) Normative, Informative (B) Informative, Normative (C) Public, Private (D) Private, Public

(B) Informative, Normative You have no idea whether or not jumping off a 30-story bridge is dangerous or not, but your friend group assures you that it is not dangerous at all. You believe your friends and jump off the bridge. This is an example of Informative Influence as you are looking to the group for guidance. If you had known that it was dangerous, but went ahead and jumped as to not look like a wimp, that would be an example of Normative Influence as you are letting yourself be influenced by social norms.

Racial and ethnic identity may be pronounced in first generation immigrants, and the same biases against race/ethnic minorities may be compounded by the immigrant status of an individual - this interplay is known as: (A) Immigrant discrimination (B) Intersectionality (C) Ethno-racial immigrant prejudice (D) Immigrant opposition

(B) Intersectionality *Intersectionality* is the interplay between multiple demographic factors, especially when leading to oppression and discrimination

Which route of drug delivery refers to the drug being delivered by needle into the muscle, and can be either a quick or slow delivery of the drug? (A) Inhalation (B) Intramuscular (C) Injection (D) Transdermal

(B) Intramuscular Intramuscular drug delivery refers to the drug being delivered by needle into the muscle, and can be either a quick or slow delivery of the drug

Jim joins a gang at a young age. Although he wasn't originally a very deviant boy, as soon as the people in his neighborhood started associating Jim with the gang he belonged too, Jim started to act the way his neighbors would expect a gang member to act. Which theory would best explain the Deviance in this example.

(B) Labeling Theory Jim is labelled by his neighbors and begins to act accordingly. This would best be explained by Labeling Theory.

Learning that occurs without a reward but that is spontaneously demonstrated once a reward is introduced - this is an example of: (A) Predisposition (B) Latent learning (C) Problem solving (D) Instinctive drive

(B) Latent learning *Latent learning* is learning that occurs w/o rewards but is spontaneously demonstrated once a reward is introduced. Experiment with rats in a maze - rats who were simply carried through the maze and then incentivized with food reward performed just as well if not better than rats operantly conditioned through the maze.

Another two-year-old has learned to say female words like "mama", but has not learned "dada" or any other male words. The specialist insists it is because those masculine sounds/words are not being rewarded enough by the parents. Which theory of Language Development would this specialist most agree with? (A) Nativist approach (B) Learning reinforcement (C) Interactionist approach (D) Universal approach

(B) Learning Reinforcement Learning Reinforcement states that the infant learns to make and put together certain sounds because specific sounds will make the individuals around them will react positively to them, reinforcing those sounds.

_____________ is a hormone, produced in the ___________ gland that helps control a human's internal clock or circadian rhythm. (A) Aldosterone, Pineal (B) Melatonin, Pineal (C) Aldosterone, Adrenal (D) Melatonin, Adrenal

(B) Melatonin, Pineal Melatonin is a hormone, produced in the Pineal gland that helps control a human's internal clock or circadian rhythm.

Joseph writes in his dissertation that any country can become industrialized. He believes that all countries take a similar path to get there. Joseph's Dissertation lends support to which Globalization Theory? (A) World Systems Theory (B) Modernization Theory (C) Dependency Theory (D) Global Positivity Theory

(B) Modernization Theory Modernization Theory states that any country can become modernized and industrialized through a similar process.

Imagine that in Taiwan, lifting up your middle finger is a sign of respect (not actually true), but in America it is very disrespectful and is akin to swearing at someone. Frank is from Taiwan and flips off another driver as a sign of respect. The driver gives him an angry face. Which type of Norm does this example exemplify?

(B) More A More is a norm that is based on the moral values of society. Flipping someone off is seen as morally wrong but is not punishable by law.

Which stage of sleep is characterized by Theta Waves, Sleep Spindles, and K-Complexes? (A) N1 (B) N2 (C) N3 (D) REM

(B) N2

The intense heat of a fire causes people to run away from it. Which of the following is this an example of? (A) Positive Taxis (B) Negative Taxis (C) Positive Kinesis (D) Negative Kinesis

(B) Negtive Taxis Because the response is to move away from the stimulus, this is an example of Negative Taxis. Moving towards would be positive taxis (like a moth to a light) There is no such thing as positive or negative Kinesis, because Kinesis involves changes in speed and turning, not moving to/away from something.

With current classifications, there are three clusters of Personality Disorders. Which of the following is not one of these clusters? (A) Odd and eccentric (B) Nervous and timid (C) Anxious and fearful (D) Dramatic and emotional

(B) Nervous and timid The other three are all types of clusters

Women are shown naked in movies 3 times more often than men are. This most closely illustrates which term of interest to Feminists? (A) Discrimination (B) Objectification (C) Oppression (D) Stereotyping

(B) Objectification Objectification is the treating of someone like an object to be looked rather than respected as is the case with pornography or pornographic media, which in this example illustrates that women are more commonly the victim of objectification.

Which of the following is the correct description of bloodflow during the Fight or Flight Response? (A) Peripheral Vasoconstriction will increase bloodflow and blood pressure to the extremities, allowing them to increase performance and escape the stressor. (B) Peripheral Vasoconstriction will decrease bloodflow to the extremeties, increasing blood pressure and saving blood for the core area and vital organs. (C) Peripheral Vasodilation will increase bloodflow and blood pressure to the extremities, allowing them to increase performance and escape the stressor. (D) Peripheral Vasodilation will decrease bloodflow to the core area and vital organs, and decrease blood pressure.

(B) Peripheral Vasoconstriction will decrease bloodflow to the extremeties, increasing blood pressure and saving blood for the core area and vital organs.

We tend to recall information best when we can put it into the context of our own lives. This phenomenon is known as: (A) Semantic encoding (B) Self-reference effect (C) Mnemonics (D) Automatic processing

(B) Self-reference effect

Think about the vocabulary tests from elementary school. What type of memory was tested? (A) Procedural Memory (B) Semantic Memory (C) Priming (D) Episodic Memory

(B) Semantic Memory

In the information processing model, which of the following terms best describes the initial scent of fresh-cut grass that can easily be forgotten later? (A) Echoic Memory (B) Sensory Memory (C) Working Memory (D) Iconic Memory

(B) Sensory Memory

What refers to a change over time of the responsiveness to a constant stimulus and thus represents a down regulation of a sensory receptor in the body? (A) Amplification (B) Sensory Adaptation (C) Feature Detection Theory (D) Proprioception

(B) Sensory adaptation

After working in a neurobiology research lab for a semester and learning the basics, your PI tells you she has something for you that will "engage your prefrontal cortex" more. Which of the following interpretations is most likely? (A) She wants to coordinate your left and right hemispheres better. (B) She wants to give you a larger project where you will solve problems and make decisions. (C) She wants to elicit a fear response out of you before she awards you a greater role in the lab. (D) She wants you to recall all of the relevant literature you have read during the previous semester.

(B) She wants to give you a larger project where you will solve problems and make decisions The Prefrontal Cortex is in charge of problem solving, decision making and managing behavior in social situations. Congratulations on the new project!

Which sleep disorder refers to people often being unaware they have the disorder? During sleep, one stops breathing multiple times throughout the night and the body ends up not getting enough N3/slow wave sleep. (A) Narcolepsy (B) Sleep Apnea (C) Insomnia (D) Sleep walking

(B) Sleep Apnea Sleep Apnea is a sleep disorder that people are often unaware they have. People with sleep apnea stop breathing multiple times throughout the night resulting in the body not getting enough N3/slow wave sleep.

____________________ refers to the concept of people being anxious or concerned about confirming a negative stereotype about one's social group, which can cause reduced performance and lower one's personal investment in an activity. An example could be women driving or white males in sports such as Olympic sprinting. (A) Stereotype (B) Stereotype threat (C) Self-fulfilling prophecy (D) Prejudice

(B) Stereotype threat *Stereotype threat* refers to the concept of people being anxious or concerned about confirming a negative stereotype about one's social group, which can cause reduced performance and lower one's personal investment in an activity. An example could be women driving or white males in sports such as Olympic sprinting.

Of the four main classes of psychoactive drugs, which class is known for exciting the Central Nervous System, increasing one's Heart Rate, Blood Pressure, awareness, and energy? (A) Depressants (B) Stimulants (C) Hallucinogens (D) Opiates/Opioids

(B) Stimulants Stimulants like amphetamines, cocaine etc. cause increased heart rate, blood pressure, alertness, energy, euphoria. They may cause increases in levels of dopamine, norepinephrine and seratonin and decreased reuptake

Which of the following is not a cognitive reaction used to interpret surroundings? (A) Appraisal (B) Surveying (C) Expectations (D) General Thoughts

(B) Surveying Appraisal, expectations and general thoughts will combine to produce an emotional reaction.

Some taste types have similar cell receptors. Which taste cells rely on G-Protein Coupled Receptors? (A) Sweet, Salty, Sour (B) Sweet, Umami, Bitter (C) Salty, Sour, Umami (D) Bitter, Umami, Sour

(B) Sweet, Umami, Bitter

A ray of light from the right visual field hits the __________ side of the left eye and hits the _________ side of the right eye. (A) Nasal, Temporal (B) Temporal, Nasal (C) Temporal, Temporal (D) Nasal, Nasal

(B) Temporal, Nasal

Ever since Becca became concussed, she has noticed her senses are all impaired except for smell. Which structure did she most likely injure? (A) Amygdala (B) Thalamus (C) Hypothalamus (D) Hippocampus

(B) Thalamus The thalamus acts as a relay station for all senses except smell, sending the information to various parts of the cortex.

Self-concept can be broken down into what two parts? (A) The Existential and Rhetorical Self (B) The Existential and Categorical Self (C) The Rhetorical Self and Categorical Self (D) The Rhetorical Self and Reflective Self

(B) The Existential and Categorical Self Self-concept can be broken down into The Existential and Categorical Self.

True or False. When you are hungry, Leptin is found in high amounts in your blood as it is a hunger-stimulating hormone.

False. When you are full, Leptin is found in high amounts in your blood as it is an appetite-suppressing hormone.

During adolescence, the brain continues to develop in several ways. Which of the following areas of the brain is NOT known to continue to develop during adolescence? (A) The limbic system (B) The somatosensory cortex (C) The corpus callosum (D) The pre-frontal cortex

(B) The somatosensory cortex The limbic system, corpus callosum and pre-frontal cortex all continue developing through (at least parts of) adolescence.

In terms of the Trichromatic theory of color vision, which of the following is the most accurate description? (A) There are three types of rods, each most sensitive to a certain quantity/intensity of light. (B) There are three types of cones, each most sensitive to a specific wavelength of light (Red, green, and blue). (C) There are three types of cones, each most sensitive to a specific quantity/intensity of light. (D) There are three types of rods, each most sensitive to a specific wavelength of light (Red, green, and blue).

(B) There are 3 types of cones each most sensitive to a specific wavelength of light (RGB)

Which of the following is a popular explanation for non-vital neonatal reflexes? (A) They are an artifact of convergent evolution. (B) They are evolutionary holdovers, artifacts that may have been useful to ancestors. (C) They prime muscles for later motor development. (D) There are no reasonable explanations for non-vital neonatal reflexes.

(B) They are evolutionary holdovers, artifacts that may have been useful to ancestors

Just outside of Denver, city dwellers began buying the land surrounding the city randomly, which resulting in a sprawling urban area that was totally disorganized. This is an example of which of the following? (A) Suburbanization (B) Urban Sprawl (C) Urban Decline (D) Urban Renewal

(B) Urban Sprawl Urban Sprawl is what happens when a city expands in a disorganized fashion. This is the case here in Denver.

In some forms of gambling, the user is rewarded at random. This example is most illustrative of which partial reinforcement schedule? (A) Fixed-ratio (B) Variable-ratio (C) Fixed-interval (D) Variable-interval

(B) Variable-ratio *Variable-ratio (VR) schedules* reinforce a behaviour after a varying number of performances, such that the _average number of performances_ to receive the reward is _relatively constant_ (e.g. rewarding a rat with food after 2 button presses, then 8, then 4, then 6) Gambling utilizes variable-ratio conditioning at it rewards after a random number of actions.

In a negative feedback loop, how is the upstream endocrine gland given feedback? (A) Via nerves that are activated by the downstream hormone (B) Via receptors on the upstream endocrine gland for the downstream hormone (C) Via sensing the effects of the downstream hormone (D) There is no known mechanism

(B) Via receptors on the upstream endocrine gland for the downstream hormone In a negative feedback loop, the upstream endocrine gland is given feedback via receptors on the upstream endocrine gland for the downstream hormone.

In patients with insomnia, elevated cortisol levels are often found. This extra cortisol will __________ their metabolism and ___________ their relaxation and sleep. (A) enhance, enhance (B) enhance, suppress (C) suppress, suppress (D) suppress, enhance

(B) enhance, suppress Cortisol is an "awake" or "stress" hormone that increases metabolism and stress, which will suppress relaxation and sleep

Sound waves are represented graphically by peaks and troughs. If a wave has a high wavelength, its peaks must be: (A) close to each other. (B) far away from each other. (C) very tall. (D) very short.

(B) far away from each other

The top of the hair cells, which make up the hair bundle (upper membrane of the organ of corti) are made up of small filaments known as ______________, connected by _________ (A) auditory nerve, kinocilium (B) kinocilium, tip links (C) stapes, tip links (D) cilia, kinocilium

(B) kinocilium, tip links

Hannah feels like she is making a great contribution to the world through what she does each day. "Make a difference," is her mantra. How old is Hannah? (A) 5 years (B) 21 years (C) 48 years (D) 68 years

(C) 48 years Hannah is conquering Generativity vs. Stagnation, which occurs between the ages of 40 and 60.

Classical conditioning is the process of taking advantage of reflexive, unconditioned stimulus to turn a neutral stimulus into a conditioned stimulus. This process is known as: (A) Spontaneous recovery (B) Generalization (C) Acquisition (D) Reinforcement

(C) Acquisition Classical conditioning is the process of taking advantage of reflexive, unconditioned stimulus to turn a neutral stimulus into a conditioned stimulus. This process is known as *acquisition*

_________________ is a dream theory that explains that dreams equates to the brain activity in our brainstem attempting to be interpreted by the brain's frontal cortex. (A) Freud's Theory of Dreaming (B) Evolutionary Biology Dream Theory (C) Activation Synthesis Hypothesis (D) Neural Pathway Theory

(C) Activation Synthesis Hypothesis The Activation Synthesis Hypothesis is a dream theory that explains that dreams equates to the brain activity in our brainstem attempting to be interpreted by the brain's frontal cortex

Bill recently retired and is finding less meaning in his day-to-day life. He misses going to work each day and feeling like a contributing member of society. His wife Jenny suggests that he pick up a new hobby to fill the void. Jenny's advice is most similar to which theory? (A) Life Course Theory (B) Age Stratification Theory (C) Activity Theory (D) Disengagement Theory

(C) Activity Theory Activity Theory suggests that when you stop one activity, you should replace it with another to maintain morale and well-being. This is in line with Jenny's recommendation.

If there is a lesion after the optic chiasm on the left side, how would vision be affected? (A) All information from the right eye would be compromised. (B) All information from the left eye would be compromised. (C) All information from the right visual field would be compromised. (D) All information from the left visual field would be compromised.

(C) All information from the right visual field would be compromised. If there were a lesion in the left optic nerve AFTER the optic chiasm, then information from the right visual field would be compromised.

Which of the following symptom groups are NOT characteristic of depression? (A) Feeling helpless and low self esteem (B) Lack of energy and change in weight (C) Decreased self-awareness and increased interests (D) Low mood and Decreased focus

(C) Decreased self-awareness and increased interests

A caregiver shows inconsistent responses to a distressed child (sometimes appropriate, sometimes neglectful) which disables the child from forming a secure base. The child becomes distressed when the caregiver leaves, but shows mixed response upon their return - the child is always anxious about the caregiver's reliability. What sort of attachment is this describing? (A) Secure attachment (B) Avoidant attachment (C) Ambivalent attachment (D) Disorganized attachment

(C) Ambivalent attachment *Ambivalent attachment* is sometimes known as *anxious-ambivalent attachment*. The child becomes distressed when the caregiver leaves, but shows mixed response upon their return - the child is always anxious about the caregiver's reliability, and unable to form a secure base.

Which of the Cluster B (dramatic, emotional and erratic) Personality disorders is characterized by a lack of regard for others, including no remorse? (A) Histrionic (B) Borderline (C) Antisocial (D) Narcissistic

(C) Antisocial Antisocial Personality Disorder is characterized by a lack of concern for others, and its sufferers often find themselves in legal issues showing no remorse. 3x more common in men, many criminals tend to have this personality type. Maybe it's linked to low inhib high T...

Which of the following is the best example of Demographic Transition? (A) As Africa becomes more industrialized its population increases more rapidly. (B) As Africa becomes more industrialized its population declines more rapidly. (C) As Africa becomes more industrialized its population stabilizes. (D) As Africa becomes more industrialized its population declines more slowly.

(C) As Africa becomes more industrialized its population stabilizes. This is the basic premise of Demographic Transition, namely that as a country industrializes its birth and death rates will decrease, resulting in a more stable population.

Two siblings could be born with the same combination of genes that can lead to depression, but the environment only activates those genes for one of the siblings. Which of the following scenarios is this most like? (A) Ben and Chrissy both have a dream to win a reality TV show, but only Ben achieves this dream. (B) Devon and Mike grew up having very different looks, and because Devon was more handsome, he received more positive attention. (C) Ashley and Alan both have a predisposition for panicking under stress, and Alan has a panic attack after he gets in a car accident. (D) Lauren and Desi grew up with very different lifestyles, so that only Lauren enjoys working outdoors.

(C) Ashley and Alan both have a predisposition for panicking under stress, and Alan has a panic attack after he gets in a car accident. In this scenario, the two people (Ashley and Alan) both suffer from a predisposition to a mental health issue (Panic Attacks), but the environment only activates this in one of the two (Alan in his car accident). Both have same DNA combo, but a direct environmental cause affected only one's predisposition.

Mohammad was on a roller coaster when it broke down. Ever since then, he has had a bad attitude toward riding roller coasters and no longer rides on them. Which attitude theory best explains Mohammad's thought process? (A) Theory of Planned Behavior (B) Elaboration Likelihood Model (C) Attitude-to-behavior Process Model (D) Prototype Willingness Model (PWM)

(C) Attitude-to-behavior Process Model. According to the Attitude-to-behavior Process Model, an event will influence our attitude, which will then influence our behavior.

Schizophrenia is likely to be a cause of: (A) Genetics (B) Environmental factors (C) Both A and C (D) None of the above

(C) Both A and C Both genetics and environment are believed to contribute to schizophrenia

Which of the following affect intelligence? (A) Nature (B) Nurture (C) Both Nature and Nurture (D) Neither

(C) Both Nature and Nurture Based on twin studies, intelligence has been shown to be influenced by nature and nurture.

70% of individuals have language controlled in the left hemisphere of the brain. The _____________ of the brain controls our speech function, and the ______________ controls our understanding and comprehension of language. (A) Broca's Area, hippocampus (B) Wernicke's Area, amygdala (C) Broca's Area, Wernicke's Area (D) Pre-frontal cortex, amygdala

(C) Broca's area, Wernicke's area

If you were to ask Jackie's 5 besties what single trait best defines her, they all unanimously say that she is determined. Determination could be said to be Jackie's: (A) Central Trait (B) Secondary Trait (C) Cardinal Trait (D) Distinct Trait

(C) Cardinal Trait Cardinal Traits are traits that we are most known for and that most closely define us. This is the case with Jackie.

______________ taste buds are flat mound structures that are primarily found in posterior portion of the tongue. (A) Fungiform (B) Foliate (C) Circumvallate (D) Filiform

(C) Circumvallate Circumvallate taste buds flat mound structures that are primarily found in posterior portion of the tongue.

Sarah is motivated to go to work because she knows that it is the responsible thing to do. When asked why she doesn't skip work to party, she replies that it makes rational sense to go to work so that she can pay the bills and not end up living on the streets. Which theory of motivation is Sarah relying upon to make this explanation? (A) Maslow's Hierarchy (B) Evolutionary Approach (C) Cognitive Approach (D) Optimal Arousal Theory

(C) Cognitive Approach Sarah is relying on logical thinking to explain her motivation. The Cognitive Approach focuses on rationality and decision-making abilities.

When processing the environment, feature detection is known as breaking down an object to its component parts. What are these component parts? (A) Color, Form, (B) Motion, Form (C) Color, Form, and Motion (D) Color, Motion

(C) Colour, form, motion

Mark belongs to a community in which everyone shares property and profits. His community most closely resembles which type of government? (A) Democracy (B) Dictatorship (C) Communism (D) Monarchy

(C) Communism

Billy thinks that hospitals are great as they help take care of sick people. He sees big hospitals as the natural outgrowth of small family doctor offices. Which perspective does he have on Social Institutions? (A) Progressive (B) Democratic (C) Conservative (D) Optimistic

(C) Conservative The Conservative view sees Social Institutions as naturally springing up to fill a need in society. The Progressive view sees Social Institutions as man-made organizations that need to be controlled if they are to do their job right. There is no such thing as a Democratic or Optimistic view of Social Institutions.

Which of the following "anxious" and "fearful" Personality Disorders is characterized by being submissive and clingy? (A) Avoidant (B) Obsessive Compulsive Personality (C) Dependant (D) None of the above

(C) Dependant Dependant Personality Disorder is characterized by being submissive and clingy.

The conclusion of Jessie's Dissertation is that South America is trapped in never ending subordination to countries such as the United States. Jessie's Dissertation lends support to which Globalization Theory? (A) World Systems Theory (B) Modernization Theory (C) Dependency Theory (D) Butterfly Effect Theory

(C) Dependency Theory Dependency Theory states that some countries are dependent upon other countries in ways that may be hard to disrupt. It states that these countries cannot follow the same path to industrialization as countries like the US and Canada did.

The main active ingredient in marijuana is THC. A hot-button topic for 21st century medicine is if medical marijuana should be legalized/prescribed. Which classes of drugs does Tetrahydrocannabinol (the active ingredient) act like? (A) Stimulant and depressant (B) Depressant and narcotic (C) Depressant and hallucinogen (D) Stimulant and hallucinogen

(C) Depressant and hallucinogen THC both increases perceptual sensitivity (hallucinogen) and inhibits the CNS (depressant)

Which of the following is not a way that hormone levels are regulated in the body? (A) Negative feedback loops (B) Metabolized in liver (C) Digested in small intestine (D) Filtered out by kidneys

(C) Digested in small intestine Hormones are regulated by negative feedback loops, being metabolized in the liver, and by being filtered out by the kidneys

How are the 5 different tastes (salty, sweet, sour, umami, and bitter) sensed by the tongue? (A) There are areas of the tongue whose taste buds are sensitive to one of the five flavors. (B) There are areas of the tongue whose taste bud types are sensitive to one of the five flavors. (C) Each taste bud is composed of multiple cells, that combine to sense the 5 flavors. (D) Each taste bud is composed of multiple cells, that combine to sense 1 of the 5 flavors.

(C) Each taste bud is composed of multiple cells, that combine to sense the 5 flavors. Every single taste bud can detect each one of the tastes. Because of this, any flavor can be tasted on any part of the tongue!

Some people argue that Solomon Asch's experiment is not generalizable to everyday situations in which there are incentives to conform with a group. This refers to which experimental error? (A) Demand Characteristics (B) Sampling Error (C) Ecological Validity (D) Double-blind Bias

(C) Ecological Validity These people are questioning the Ecological Validity of the study, the degree to which it applies to the real world. Sampling Error refers to the unrepresentativeness of the sample, in this case the sample being mostly college students and not diverse at all. Demand characteristics refers to how participants may change behaviour in order to match with expectations of the experimenter

Which category of behavior is characterized by a sequence of multiple actions that are performed without any interruption? (A) Reflexes (B) Orientation behaviors (C) Fixed Action Patterns (D) None of the Above

(C) Fixed Action Patterns A Fixed Action Pattern is characterized by a coordination sequence of multiple actions that will be performed without any interruption or change once initiated (hence the word "fixed"). E.g. praying mantis

Zach is at dinner with his girlfriend's parents. He eats with his elbows on the table, which his girlfriend's parents frown upon. Which type of Norm does this example exemplify? (A) Taboo (B) More (C) Folkway (D) Law

(C) Folkway Folkways are long-held traditions and are often seen as courtesies as is the case with not eating with your elbows on the table. They are not often punished.

Mark's Professor reads Mark's research article. His professor states that the paper does not account for the individual people that make up the society examined and that it does not take into account wide-scale changes that may occur. Mark most likely used which perspective in writing his paper? (A) Conflict Theory (B) Symbolic Interactionism (C) Functionalism (D) Social Constructionism

(C) Functionalism Mark looked at society as a whole and failed to account for wide-scale changes that might come from conflict. This implies that Mark was relying on the Functionalist perspective.

Hannah believes that there are thousands of different traits out there in the world, but she believes that some traits are more important than others. Which theorist is her thinking most closely in line with? (A) Big Five Theorists (B) Hans Eysenck (C) Gordon Allport (D) Raymond Cattel

(C) Gordon Allport Gordon Allport believes that there are over 4500 different traits, but that some are more important than others (i.e. Cardinal Traits being more important than Central or Secondary Traits).

NASA met to discuss whether they should move forward with the Challenger launch. Those in opposition to the launch keep quiet. When those with a different opinion do speak up, they are censored. The group feels they are invincible and move forward with the launch. The launch fails, killing the astronauts involved. Which term best describes the leadership team's behavior? (A) Group Polarization (B) Conformity (C) Groupthink (D) Confirmation Bias

(C) Groupthink

Of the four main classes of psychoactive drugs, which class is referred to as psychedelics, and known for causing distorted perception in its users such as heightened sensations, mood swings and increased/decreased energy, however the exact effect is different for each individual personality? (A) Depressants (B) Stimulants (C) Hallucinogens (D) Opiates/Opioids

(C) Hallucinogens

Which of the following scenarios would most follow the James-Lange theory of emotion? (A) Ashley is crying because she is already sad from not getting a rose. (B) Seeing a ghost, Scooby simultaneously started to feel fearful and started to scream (C) Holding your puppy can make you happy by increasing heart rate, which changes certain neurotransmitter levels in the brain (D) Jadeveon screamed in anger before the football play, because he thought the other team would score.

(C) Holding your puppy can make you happy by increasing heart rate, which changes certain neurotransmitter levels in the brain. The James-Lange theory of emotion hypothesizes that one's interpretations of physiological changes cause emotions. A physiological change precedes the emotion!

Which route of drug entry is the most direct, and goes right to the vein, entering the circulatory system very quickly? This method can be very dangerous, and is a highly addictive method of drug delivery. (A) Inhalation (B) Intramuscular (C) Injection (D) Transdermal

(C) Injection

Martha visited her primary care physician because she was having issues with being dehydrated and not able to breastfeed her newborn. Which of the following endocrine glands does the doctor think is malfunctioning? (A) Pancreas (B) Parathyroid (C) Hypothalamus (D) Adrenal Gland

(C) Hypothalamus The hypothalamus is in charge of producing ADH, an antidiuretic hormone, and oxytocin, which is necessary for the letdown of milk in breastfeeding.

Clayton was in the woods hunting deer when he drew the attention of a black bear. Which area of the limbic system will regulate his fight-or-flight response to this scare? (A) Amygdala (B) Thalamus (C) Hypothalamus (D) Hippocampus

(C) Hypothalamus The hypothalamus regulates the autonommic nervous system, including the fight-or-flight and the rest-and-digest principles.

Which of the following hormones are stored in the posterior pituitary gland before release? I. ADH II. Oxytocin III. TSH (A) I only (B) III only (C) I and II (D) I and III

(C) I and II ADH and Oxytocin are both produced in the hypothalamus, but stored in the posterior pituitary gland before release.

Often accused of just trying to be prescribed medications, Raven will go to many doctors falsifying signs and symptoms for the attention she can get being by being sick. What could Raven be diagnosed with? I. Munchhausen's II. Factitious Disorder III. Munchhausen's by Proxy (A) I only (B) III only (C) I and II (D) II and III

(C) I and II Munchhausen's and Factitious Disorder are synonymous. It would be Munchhausen's by Proxy if somebody else (like her mother) were presenting Raven with false symptoms.

Referring to the previous example, which of the following cues would lead Jack to assume that Dr. Marks' behavior is resulting from the situation (i.e. That is is a Tuesday)? I. Consistency II. Distinctiveness III. Consensus (A) I Only (B) II Only (C) I and II Only (D) I, II, and III

(C) I and II Only Consistency is high in this case; which would lead Jack to assume that Dr. Marks is wearing the red shirt because it is a Tuesday and not because it is related to Dr. Mark's innate characteristics.

Which of the following structures are affected by Parkinson's Disease? I. Substantia Nigra II. Dopaminergic Neurons III. Myelin (A) I only (B) II only (C) I and II only (D) II and III only

(C) I and II only The biological basis of Parkinson's disese is decreased dopamine production in the substantia nigra, a layer of cells in the brain that functions to produce dopamine to permit proper functioning of basal ganglia. The substantia nigra is noticeably less dark and the dopaminergic neurons are affected in patients with Parkinsons.

You are teaching a classroom full of students about psychology. At the beginning of the lecture, you teach the students about Freud, in the middle about Erikson, and at the end about Piaget. Which theorist(s) will the students remember the best? I. Freud II. Erikson III. Piaget (A) I Only (B) I and II Only (C) I and III Only (D) II and III Only

(C) I and III Only We tend to remember information at the beginning (Primacy Bias) and information presented at the end (Recency Bias).

Rate the following retrieveal tests in order of diffuclty from hardest to easiest to complete. I. Recognition II. Free Recall III. Cued Recall (A) I > II > III (B) II > I > III (C) II > III > I (D) I > III > II

(C) II > III > I Hardest -> easiest. Free recall is hardest, recognition is easiest.

What is the correct order of when the following major motor milestones develop in infants from first to last? I. Crawl II. Lift head III. Roll over IV. Stand with support (A) II > IV > III > I (B) III > IV > II > I (C) II > III > I > IV (D) II > III > IV > I

(C) II > III > I > IV All of these developments occur within the first 12 months of the infant's life. Exact ages can vary from infant to infant. The typical stages (within the first 14-18 months!) are as follows: Lift head -> roll over -> sit w/o support -> stand w/ support -> pull self up -> crawl -> walk w/ support -> stand w/o support -> walk w/o support

Put the following phases of the Sexual Response Cycle in order from first to last: I. Plateau II. Orgasm III. Excitement IV. Refractory Period

(C) III > I > II > IV In order from first to last, the order of the phases of the Sexual Response Cycle are as follows: Excitement > Plateau > Orgasm > Refractory Period.

_________________________________ refers to the category we place others in during impression formation. This theory that states that there are sets of assumptions people make about how different types of people, their traits and their behaviours are related. (A) The halo effect (B) Attribution theory (C) Implicit personality theory (D) Reliance on central traits

(C) Implicit personality theory *Implicit personality theory* refers to the category we place others in during impression formation. This theory that states that there are sets of assumptions people make about how different types of people, their traits and their behaviours are related.

African Americans Adam and Sarah Smith are trying to rent a new apartment. They don't get approved by the landlord. Then a white couple, Jack and Jill Jones, with the same income and credit history as the Smiths, apply for the apartment and get approved. This is an example of which type of Discrimination? (A) Unintentional Discrimination (B) Side-effect Discrimination (C) Individual Discrimination (D) Institutional Discrimination

(C) Individual Discrimination This is an example of Individual Discrimination because a single individual, the landlord is doing the discriminating. If this was a widespread problem within a real estate company, at that point we would refer to it as Institutional Discrimination. This is actually a common problem across apartment complexes. Very sad.

In the Solomon Asch conformity study, some participants observed that cohorts gave an answer that seemed incorrect, but convinced themselves the group must be right and gave the group's answer, believing it is right. Which of the following is this an example of? (A) Normative Social Influence (B) Societal influence (C) Informational Social Influence (D) Perceptual Error

(C) Informational Social Influence Because the participant correctly perceived the right answer but was convinced before answering the group was right and *doubted themselves*, this exemplifies Informational Social Influence. In normative social influence, the participant believes the group is wrong, but provides that same wrong answer *to avoid ridicule*. In perceptual error, the participant actually concludes a wrong answer from the information provided, not due to group influence. Societal Influence is a fictitious answer.

According to the Psychoanalytic Theory, what two things influence our behavior? (A) Our current emotions and unconscious desires (B) Our current emotions and future expectations (C) Our childhood experiences and unconscious desires (D) Our childhood experiences and future expectations

(C) Our childhood experiences and unconscious desires The Psychoanalytic Theory says that our childhood experiences and unconscious desires influence our behaviour. Freud's theory consists of three major entities: the id, ego and superego. This theory is also known as the psychodynamic theory of personality.

Instead of feeling a "fight or flight" response before the championship game, Moriah experiences a response closer to ""Tend and Befriend," where she feels a closer bond to her team and a need to work together to overcome it. Which hormone most likely is causing this response? (A) Epinephrine (B) Cortisol (C) Oxytocin (D) Norepinephrine

(C) Oxytocin

Which of the following endocrine glands is most likely going to be affected after digesting a meal with a large amount of sugar? (A) Thyroid (B) Parathyroid (C) Pancreas (D) Hypothalamus

(C) Pancreas The pancreas secretes both glucagon and insulin to control blood sugar levels.

Jimmy suffers from Sexual Dysfunction, at which stage did Jimmy become fixated? (A) Oral (B) Anal (C) Phallic (D) Genital

(C) Phallic Fixation at the Phallic stage is characterized by sexual dysfunction.

The hypothalamus does not send tropic hormones to the posterior pituitary - instead, neurons in the hypothalamus send their axons down the ________________ directly into the posterior pituitary. (A) Hypophyseal portal system (B) Pituitary duct (C) Pituitary stalk (D) Pituitary receptors

(C) Pituitary stalk Neurons in the hypothalamus send their axons down the pituitary stalk directly into the posterior pituitary.

You are trying to teach your parakeet to say your name. Every time she says something close to your name, you give her a treat. Which of the following Operant Conditioning terms is this an example of? (A) Positive Punishment (B) Negative Punishment (C) Positive Reinforcement (D) Negative Reinforcement

(C) Positive Reinforcement Giving your parakeet a treat is an example of Positive Reinforcement because you are adding a pleasant stimulus.

Lexie believes that it is okay to kill someone if it would result in saving a greater number of lives. She believes that what is legal is not always what is moral. Her mantra is, "The greatest good for the greatest number." Which stage of moral development is Lexy currently in? (A) Pre-conventional (B) Conventional (C) Post-conventional - Social Contract (D) Post-conventional - Universal Principles

(C) Post-conventional - Social Contract This stage is focused on making life better for the majority. It is also focused on the morally right over the legally right.

Zane was drunk driving and ended up killing two young girls and their mother. Instead of feeling guilty about his actions, Zane is convinced that it wasn't his fault because he wasn't actually very drunk (even though he was). Which defense mechanism is Zane most likely implementing? (A) Subilimation (B) Humor (C) Rationalization (D) Intellectualization

(C) Rationalization Zane is convincing himself that it wasn't his fault using false logic, which is characteristic of Rationalization.

After Donald Trump was elected, large groups of people gathered together in various cities to show their disapproval. In some cases, behavior turned violent and buildings and property were damaged. Which type of Collective Behavior is best exhibited by this example? (A) Mass Hysteria (B) Fad (C) Riot (D) Rebellion

(C) Riot Riots entail large groups of people gathered to make a statement and often turn violent.

Which of the following best exemplifies Class Consciousness? (A) Jim is in school learning about social class. He realizes for the first time in his life that he is part of the middle class. (B) Mary is playing with her friend Sarah. Sarah says she won't play with Mary because she is fat, increasing Mary's sensitivity to her weight. (C) Sam works at the only lemonade stand in the town. After talking to his buddy from a neighboring town, he realizes that he and his fellow citizens are being ripped off, so he incites his fellow citizens to demand change. (D) Thomas is applying to college when he realizes that he is unable to afford enrolling in all the needed classes. He feels jealous of wealthier students.

(C) Sam works at the only lemonade stand in the town. After talking to his buddy from a neighboring town, he realizes that he and his fellow citizens are being ripped off, so he incites his fellow citizens to demand change. _Class Consciousness_ occurs when the oppressed class realizes that they are being taken advantage of and join together based on this understanding.

Jim is working out because he likes the respect he gets from his friends when they see his six-pack at the beach. Which level of Maslow's Hierarchy is Jim relying upon? (A) Safety (B) Phsyiological (C) Self-esteem (D) Belongingness/Love

(C) Self-esteem The need for Self-esteem is closely related to our desiring of respect from others, just as is the case with Jim the muscle man.

An expert pianist and a first year music student at Berklee both perform at a concert venue. The expert pianist feels confident and performs better than he expected he would in solitude, while the student, unfamiliar with the instrument, was unable to put on a good show. What phenomenon does this describe? (A) Bystander effect (B) Deindividuation (C) Social facilitation (D) Social loafing

(C) Social Facilitation Social Facilitation is the idea that you perform differently in front of others than you would if you are all alone.

What is the part of the human brain that is referred to as the map of the human body? (A) Sensory Strip (B) Midbrain (C) Somatosensory Homunculus (D) Sensory Cortex

(C) Somatosensory Homonculus

A student shows up drunk to a psychology exam, claiming he is using a retrieval cue because he studied drunk. Which retrieval cue is he using? (A) Priming (B) Context Clues (C) State-dependent (D) Recency

(C) State-dependent

Which of the following properly describes a Split Brain patient? (A) The connection between the occipital and temporal lobes is severed (B) The Arcuate Fasciculus is severed (C) The Corpus Callosum is severed (D) The Central Sulcus is severed.

(C) The Corpus Callosum is severed This produces a Split Brain patient, where the right and left hemispheres do not interact with one another.

Tomi loves debating her ideas and gets excited when an argument starts, but Megyn will back away from arguments and feel anxious. Which emotion theory allows for varying appraisals to cause different physiological reactions and emotions? (A) The James-Lange Theory (B) The Schachter-Singe theory (C) The Lazarus Theory of Emotion (D) The Cannon-Bard theory

(C) The Lazarus Theory of Emotion Only the Lazarus theory has appraisal occurring before a physiological response.

Which of the following theories is incorrectly described? (A) The theory of general intelligence states that a "g factor" could apply to various tests of intelligence. (B) The Theory of Primary Mental Abilities suggests there are seven specific factors that combine to make up all of intelligence. (C) The Theory of Multiple Intelligences is another 7-9 specific factors of intelligence that rely on each other. (D) The Triarchic Theory of Intelligence limits itself to 3 independent intelligences that will directly lead to real-world success.

(C) The Theory of Multiple Intelligences is another 7-9 specific factors of intelligence that rely on each other. They are independent and don't rely on each other

Alder's psychoanalytical theory of personality incorporated two notions: the notion of ___________, which is the force by which individuals shape their uniqueness and personalities; and the notion of ______________, which represents a person's unique way of achieving superiority. (A) Inferiority complex, the creative self (B) The creative self, inferiority complex (C) The creative self, style of life (D) The persona, style of life

(C) The creative self, style of life *Alder's psychoanalytical theory of personality* incorporated two notions: the notion of *the creative self*, which is the force by which individuals shape their uniqueness and personalities; and the notion of *style of life*, which represents a person's unique way of achieving superiority. Alder was the originator of the *inferiority complex*, and his theory stated we strive for superiority which determines personality.

Sarah was a very sweet girl until she started hanging out with the "popular" girls at school. The more time she spent with these mean girls, the more she began to talk, act, and even think like them. Which theory would best explain the Deviance in this example? (A) Social Interactionist Perspective (B) Labeling Theory (C) Theory of Differential Association (D) Strain Theory

(C) Theory of Differential Association Sarah changed over time due to her association with these mean girls. This would best be explained by The Theory of Differential Association.

You see a billboard at the local grocery store advertising for new employees. On the billboard, there is a great diversity of employees modeled, from blacks, hispanics, asians, and whites. Inside the store, however, you notice that there is only one asian and tons of whites working there. Which concept does this example best relate to? (A) Gate Keeping (B) Cultural Universals (C) Tokenism (D) Agent of Socialization

(C) Tokenism This is an example of Tokenism in which is the practice of making only a perfunctory or symbolic effort to do a particular thing, especially by recruiting a small number of people from underrepresented groups in order to give the appearance of sexual or racial equality within a workforce as is the case in this example.

Without basilar tuning, humans would not be able to differentiate between sounds. What is the mapping of different frequency sounds in the brain referred to as? (A) Auditory mapping (B) Basilar mapping (C) Tonotopical mapping (D) Geographical mapping

(C) Tonotopical mapping. It is the mapping of different frequency sounds in the brain, allowing distinct areas to respond and process distinct frequencies.

Why is Trial and Error typically not used when other problem solving methods are feasible? (A) Trial and Error is a methodical approach, but non-methodical approaches are easier. (B) Trial and Error is a resource-intensive approach, and can drain the problem solver (C) Trial and Error is not a methodical approach, and therefore an inefficient approach and does not guarantee a correct solution. (D) None of the above because Trial and Error is the superior problem solving method.

(C) Trial and Error is not a methodical approach, and therefore an inefficient approach and does not guarantee a correct solution.

Robert Sternberg found that people's intelligences could be broadly grouped into a few well-defined, separated groups. Which intelligence theory is based off of this? (A) Multiple Intelligences (B) General Intelligence (C) Triarchic Theory of Intelligence (D) Seven Types Theory of Intelligence

(C) Triarchic Theory of Intelligence Analytical, creative and practical

Overtime, Bill's downtown neighborhood began to attract more crime, buildings became abandoned and the local school fell into disrepair. This is an example of which of the following? (A) Suburbanization (B) Urban Sprawl (C) Urban Decline (D) Urban Renewal

(C) Urban Decline Urban Decline describes the decline of a city into a worse condition, often caused by people moving away from the city. This is what is described as happening in Bill's neighborhood.

Sometimes referred to as short-term memory, this type of memory can hold about 7 (plus or minus 2) pieces of information at a time and manipulate them. (A) Echoic Memory (B) Sensory Memory (C) Working Memory (D) Iconic Memory

(C) Working Memory

Which example best illustrates Bureaucratization? (A) As Sarah's business is growing, she continually needs to rely on many more businesses to help her own succeed. (B) Billy recently started a new company and is tasked with organizing his office. (C) Zach's business has been growing for several years, and over that time, he has had to increase the number of rules and regulations, making the structure of his company more rigid over time. (D) Marty is replaced as the CEO of his company as soon as his company goes public.

(C) Zach's business has been growing for several years, and over that time, he has had to increase the number of rules and regulations, making the structure of his company more rigid over time. Bureaucratization refers to the process by which a company becomes increasingly governed by law and policy as is the case here.

With _______ sleep apnea, the sufferer stops breathing without any clear respiratory tract cause, but rather due to ventilation control system errors. With ________ sleep apnea, there is a clear physical blockage. (A) Obstructive, Obstructive (B) Obstructive, Central (C) Central, Central (D) Central, Obstructive

(D) Central, Obstructive With Central sleep apnea, the sufferer stops breathing without any clear respiratory tract cause, but rather due to ventilation control system errors. Obstructive sleep apnea has a clear physical blockage, like this image.

Which of the following examples best exemplifies The Mere Exposure Effect? (A) Jim is better able to remember the lyrics to country songs compared to other songs due to hearing a lot of country music growing up. (B) Sarah is subliminally exposed to various images of hamburgers and hotdogs, making her more likely to stop at Burger King as opposed to Subway for lunch. (C) Zane sees many oak trees around his neighborhood growing up. As a result, he tends to favor neighborhoods with more oak trees. (D) Jane tends to like guys that she has met in person more than she likes guys that she merely messages online.

(C) Zane sees many oak trees around his neighborhood growing up. As a result, he tends to favor neighborhoods with more oak trees. The Mere Exposure Effect states that we tend to like or favor things that we have had more exposure to. In this case, Zane had a high amount of exposure to oak trees, causing him to favor them more. Answer choice (A) mentions nothing about likes/preferences. Answer choice (B) is more related to priming. Answer choice (D) mentions nothing about Jane being exposed to certain types of guys more than others.

Individuals with Anterograde Amnesia are taken to a lab and shown faces. They come to the lab on a later day, and tell the researcher that they ____ remember the faces, and they ____________ favor the faces that they were exposed to during the first visit. (A) do, do (B) do, don't (C) don't, do (D) don't don't

(C) don't, do Individuals with Anterograde Amnesia cannot remember events moving forward. For this reason, these individuals would forget the faces. The amazing thing is that the Mere Exposure Effect is still exhibited by these individuals, which is why they favor the faces that they were exposed to before.

In Demographic Transition, first the _________ rate will decrease and then the ____________ rate will decrease, stabilizing the population growth. Will this result in an increase or decrease in the overall population? (A) mortality, birth (B) birth, migration (C) mortality, birth (D) migration, birth

(C) mortality, birth In Demographic Transition, first the mortality rate will decrease and then the birth rate will decrease, stabilizing the population growth. This results in an increase of the overall population

You can combine images of faces to create a face that is the average of all the images. Which would be most attractive to most individuals? (A) 2 Face Average (B) 4 Face Average (C) 8 Face Average (D) 16 Face Average

(D) 16 Face Average Facial averageness is seen as more attractive.

Connor feels guilt about the things he has done throughout his life. He is starting to feel like a total failure. How old is Connor? (A) 5 years (B) 21 years (C) 48 years (D) 68 years

(D) 68 years Connor is struggling with Integrity vs. Despair, which occurs during the ages of 60+.

A man is shown a picture of a woman, and he rates her appearance on a scale of 1 to 10 as a 7. He then walks across a narrow, scary bridge and is then asked to reevaluate the image. What rating would he be expected to give? (A) 2 (B) 5 (C) 7 (D) 9

(D) 9 Attractiveness can be mediated by unwarranted physical arousal. Arousal will increase our attraction to others because we misinterpret our sympathetic nervous system activation as resulting from our attraction.

Which of the following best describes how temperament and behavior are genetically encoded? (A) In a single gene (B) A group of genes with no epigenetic changes (C) There is no genetic link (D) A group of genes is epigenetically modified based on environment

(D) A group of genes is epigenetically modified based on environment

Which of the following examples best relates to the concept of a Reference Group? (A) You are look to the lifeguard to know if it is safe to swim in the water. (B) Before getting married, a man asks his girlfriend's dad if he can have her permission to marry his daughter. (C) A man uses a female shopping catalogue to know what gift to buy for his wife. (D) A medical student tries to be a good doctor by thinking about the doctors he has shadowed in the past.

(D) A medical student tries to be a good doctor by thinking about the doctors he has shadowed in the past. Reference Groups are groups that we look up to in order to know what beliefs, attitudes, and behaviors to adopt.

Which of the following endocrine glands is improperly matched to its function? (A) Pituitary Gland - Directs other endocrine glands in their function (B) Thyroid - Monitoring metabolism (C) Parathyroid - controlling calcium levels in the blood (D) Adrenal medulla - producing steroids

(D) Adrenal medulla - producing steroids The Adrenal medulla is responsible for producing catecholamines, while the Adrenal cortex produces steroids.

Dee was a serial killer. He was extremely violent and was sentenced to prison for the remainder of his life. While in prison, he finds joy and forgets his problems as he focuses on serving his fellow inmates, trying to make their stay in prison more enjoyable. Which defense mechanism is Dee most likely implementing? (A) Passive Aggression (B) Displacement (C) Projection (D) Altruism

(D) Altruism Dee is forgetting about his own problems and finding joy in serving others. This is an example of Altruism.

In terms of Ethology (studying Behavior as it relates to adaptation), what would an "Overt Behavior" be? (A) A socially undesirable behavior (B) A behavior performed in secret (C) A behavior done for unclear reasons (D) Any observable behavior

(D) Any observable behavior In terms of Ethology, an Overt Behavior is any observable behavior. This differs from the conversational definition of overt!

A small chihuahua is laying in the shade and sees a coyote approaching. Not realizing that the coyote presents a dangerous situation, the chihuahua does not react. Which term best describes the step in which the chihuahua interpreted the threat level of the coyote? (A) Evaluation (B) Stressor (C) Stress reaction (D) Appraisal

(D) Appraisal Appraisal is the step where the situation is interpreted and Lazarus claims it can be much more impactful than the individual stressor.

As the environment becomes _______ between subjects, the heritability of given traits will _________. (A) more controlled, Increase (B) more controlled, decrease (C) less controlled, decrease (D) Both A and C

(D) Both A and C As the environment becomes more controlled between subjects, the heritability of given traits will increase - the more controlled, the less the environment contributes to differences in specified trait(s). As the environment becomes less controlled between subjects, the heritability of given traits will decrease, because the environment begins to contribute a bigger % to differences in specified trait

Which of the following situations best exemplifies Diffusion as understood by a Sociologist? (A) Japanese school children are pushed to work harder in school in order to keep up with America. (B) African employees begin making more money, leading to industrialization within their country. (C) South American Countries begin to notice more Americans walking around their main industrial centers. (D) Chinese teenagers start listening to Lady Gaga, watch Kobe Bryant play basketball, and enjoy going to McDonalds.

(D) Chinese teenagers start listening to Lady Gaga, watch Kobe Bryant play basketball, and enjoy going to McDonalds. Diffusion is the transfer of culture from one place to another as is exemplified here.

The leadership team at Google decides that self-driving cars are so cool and will change the future forever. Whenever they encounter information to the contrary, they ignore it and whenever they encounter information to the affirmative, they believe it. Which term best describes the leadership team's behavior? (A) Group Polarization (B) Conformity (C) Groupthink (D) Confirmation Bias

(D) Confirmation Bias This group is struggling with Confirmation Bias in which confirming information is believed while info to the contrary is dismissed.

Which of the following is NOT a theory on the psychologial factors/causes of depression? (A) Learned Helplessness (B) Cognitive Distortions (C) Internal, stable and global attributions (D) Decreased coping abilities

(D) Decreased coping abilities Theories of psychological causes/factors of depression include Learned Helplessness, Cognitive Distortions and Internal, stable and global attributions.

On YouTube, people write nasty comments because they have a username that is not related to who they are. This can be explained best by which term? (A) Social Loafing (B) Bystander Effect (C) Diffusion of Responsibility (D) Deindividuation

(D) Deindividuation These YouTubers feel less identifiable and thus engage in worse behavior than if their usernames were their actual names. This is explained by Deindividuation.

Fred's wife is having an affair. Instead of dealing with the problem , he simply ignores it and carries on with life as usual. Which defense mechanism is Fred most likely implementing? (A) Regression (B) Suppression (C) Repression (D) Denial

(D) Denial Fred is pretending that the problem is not true, which is what is done in Denial.

Which of the following scenarios best exemplifies a cohort? (A) Mary and her children. (B) Bill and his neighbors. (C) Jane and her siblings. (D) Dick and his high school graduating class.

(D) Dick and his high school graduating class. A Cohort is a group of people with a similar age. Because they are from the same age group, they've had similar experiences.

When Yoda reached age 932, he retired into the woods, and reflected upon all that he learned throughout his life. He found this to be extremely fulfilling. Yoda's experience is most similar to which theory? (A) Life Course Theory (B) Continuity Theory (C) Activity Theory (D) Disengagement Theory

(D) Disengagement Theory Disengagement Theory suggests that the elderly should detach themselves from society in order to engage in reflection. It views elderly that are still engaged with society as not adjusting to old age very well.

Children with this sort of attachment show no clear pattern of behaviour in response to presence/absence of the caregiver, but rather a mix of different responses which can range from avoidance or resistance to confusion to repetitive behaviours. This attachment is often associated with erratic behaviour and social withdrawal, and may be a red flag for abuse. What sort of attachment is this describing? (A) Secure attachment (B) Avoidant attachment (C) Ambivalent attachment (D) Disorganized attachment

(D) Disorganized attachment Children with this *disorganized attachment* show no clear pattern of behaviour in response to presence/absence of the caregiver, but rather a mix of different responses which can range from avoidance or resistance to confusion to repetitive behaviours. Disorganized attachment is often associated with erratic behaviour and social withdrawal, and may be a red flag for abuse.

Aging is almost always talked about with a decline in cognitive memory, however some cognitive abilites actually improve. Which of the following does NOT improve? (A) Semantic Memory (B) Crystallized Memory (C) Emotional reasoning (D) Dividing Attention

(D) Dividing Attention Semantic Memory, Crystallized IQ (using knowledge and experience for problem solving) and Emotional Reasoning all improve with age.

Researchers are trying to determine if musical tastes are genetic or environmental, which setup would best study the genetic component of the variance? (A) Monozygotic twins raised together (B) Dizygotic twins raised separately (C) Monozygotic Twins raised separately (D) Dizygotic twins raised together

(D) Dizygotic twins raised together Studying dizygotic twins will maximize the genetic variance (50% of genes will be shared), and raising twins together will give the closest approximation of identical environments to be raised in

Aging is almost always talked about with a decline in cognitive memory. Contrary to popular belief, aging does not actually come with significant memory loss, as elderly retain vivid and prospective memories. Of the following, which is the only type of memory to decrease with age? (A) Semantic Memory (B) Recognition (C) Emotional reasoning (D) Episodic Memory

(D) Episodic Memory

The humanistic theory emphasizes that we as humans have __________ and that we can actively develop ourselves to our highest potential and reach ____________________. (A) Individuality, Summation (B) Individuality, Self-actualization (C) Free Will, Summation (D) Free Will, Self-actualization

(D) Free Will, Self-actualization The *humanistic theory (or phenomological)* emphasizes that we as humans have Free Will and that we can actively develop ourselves to our highest potential and reach Self-actualization. Maslow was the first humanist!

Which match of the following DSM5 (a system of categorizing mental illnesses) illness categories with its description is incorrect? (A) Personality disorders involve distress related to long term behavioral features considered characteristic, and is broken into smaller clusters of diseases (B) Somatic symptom disorders involve having symptoms that are derived psychologically, similar to an illness but without anything wrong at tissues (C) Dissociative disorders involve distress from issues of identity or memory (D) Gender dysphoria is never recognized as an illness by the DSM5

(D) Gender dysphoria is never recognized as an illness by the DSM5 Gender dysphoria is a mental disorder if the person feels distress or disability regarding their gender identity. If there is no distress or disability, it is not considered a mental illness.

Put the following steps of an Action Potential in the Olfactory System in order: I. Odor molecule binds to specific G-Protein Coupled Receptor (GPCR) in the olfactory epithelia. II. G-Protein binds to an ion channel which allows odor molecule through the membrane, and triggers and Action Potential. III. G-Protein dissociates from Receptor. IV. Odor molecule activates mitral/tufted cell, which then synapses to the brain. V. Odor molecule travels through the Cribriform Plate to the Glomerulus. (A) I -> II -> III -> IV -> V (B) I -> III -> II -> IV -> V (C) I -> II -> III -> V -> IV (D) I -> III -> II -> V -> IV

(D) I > III > II > V > IV 1) The odor molecule enters the nasal pathway and binds to a specific G-protein Coupled Receptor (GCPR) on an olfactory sensory cell in the olfactory epithelium 2) This binding causes the G-protein to dissociate from the receptor 3) The G-protein subsequently binds to an ion channel which allows positive ions to flow into the cell, causing it to depolarize and fire an action potential 4) Firing the action potential -> the odor molecule goes through the cribriform plate to the glomerulus designated 5) From there, the odor molecule synapses to and activates a mitral/tufted cell and then synapses to the brain.

Which of the following are examples of neonatal reflexes (reflexes a newborn is born with and disappear by 12 months of age)? I Babinski reflex (curling foot with stimulus) II Pupillary reflex (the pupil will shrink in brighter lit places) III Rooting reflex (stroking a cheek makes the head turn that direction) (A) I only (B) II only (C) I and II (D) I and III

(D) I and III The Babinski reflex and Rooting reflex are both examples of neonatal reflexes.

Which of the following are examples of heuristics? I) Availability II) Representativeness III) Means-end analysis (A) I only (B) III only (C) I and III (D) I, II and III

(D) I, II and III Availability (of examples that come to mind), Representativeness (using prototypes), and Means-end Analysis (breaking large problems into smaller ones) are all heuristics, or mental shortcuts used in problem solving. Working backwards is another common heuristic.

Which of the following are potential characteristics of Mania? I. Delusions of grandeur II. Poor Judgment III. Risky behaviors influenced by increased optimism (A) I Only (B) I and III Only (C) II and III Only (D) I, II and III

(D) I, II and III Characteristics of Mania include Delusions of grandeur, Risky Behaviors, Poor Judgment, and dangerously increased Self-esteem and Optimism.

Which of the following are examples of the negative effects of elevated levels of stress? I Frontal cortex and hippocampus atrophy II Anger III Addiction (A) II only (B) III only (C) I and II (D) I, II and III

(D) I, II and III Negative effects of stress include frontal cortex and hippocampus atrophy, anger, addiction, anxiety, and depression.

Which of the following do Stigmas share aspects of? I.Stereotypes II.Prejudices III.Discrimination (A) I only (B) I and III only (C) II and III only (D) I, II and III

(D) I, II and III Stigmas have some aspects of Stereotypes, Prejudices, and Discrimination.

Which or the following can be affected by genetics? I. Drug Use II. Sexual Activity III. Hunger Levels (A) I and II Only (B) I and III Only (C) II and III Only (D) I, II, and III

(D) I, II, and III Drug Use, Sexual Activity, and Hunger Levels are all affected by genetics.

Vygotsky believed that Elementary Mental Functions develop into Higher Mental Functions. Which of the following are Elementary Mental Functions as listed by Vygotsky? I. Attention II. Perception III. Sensation (A) I Only (B) II Only (C) III Only (D) I, II, and III

(D) I, II, and III Vygotsky believed that there are four basic mental functions: (1) Attention (2) Sensation (3) Perception (4) Memory

Put the following terms in order of increasing size: I. Metropolis II. Village III. City IV. Megalopolis

(D) II < III < I < IV In order of increasing size: Village < City < Metropolis < Megalopolis

Herbert Blumer came up with the three tenets of Symbolic Interactionism. Which of the following is/are not some of his tenants? I. Our actions are dependent upon the meaning we attach to things. II. People learn the meanings of things from others. III. The meaning we attach to things are stable over time. (A) I Only (B) II Only (C) I and II Only (D) II and III Only

(D) II and III Only People learn the meanings of things from others (II), and the meaning we attach to things are stable over time (III) are NOT tenets of Symbolic Interactionism. The three tenets of Symbolic Interactionism are as follows: I. Our actions are dependent upon the meaning we attach to things. II. Different people attach different meanings to things. III. The meaning we attach to things CAN change over time.

Which of the following are appropriate uses of the word "mood" according to the DSM-5? I. A short-term emotional state, directly driven by emotions II. A long-term emotional state, related to emotions III. The subjective experience of emotions, similar to affect (A) I only (B) I and II only (C) I and III only (D) II and III only

(D) II and III only Appropriate uses of "mood" include Long-term emotional states (compared to short-term emotions) and the subjective experience of emotions, similar to affect.

Put these steps of the middle ear audition pathway in order: I. Malleus, incus and stapes vibrate in that order. II. Oval window (elliptical window) starts to vibrate and moves cochlear fluid. III. Tympanic membrane vibrates. (A) I > II > III (B) II > I > III (C) III > II > I (D) III > I > II

(D) III > I > II Ear drum vibrates --> three bone structure vibrates --> stapes makes oval window vibrate and move cochlear fluid

In the information processing model, which of the following terms best describes the initial sight of blood or seeing a white coat that is easily forgotten? (A) Echoic Memory (B) Sensory Memory (C) Working Memory (D) Iconic Memory

(D) Iconic Memory Seeing either blood or a white coat would count as iconic memory, because they were seen but kept only in sensory memory.

A researcher is looking at blood samples of clinically depressed patients. Which of the following are they most likely to find? (A) Decreased blood glucose levels (B) Increased cytokines (C) Decreased dopamine (D) Increased cortisol levels

(D) Increased cortisol levels Many findings have seen abnormal blood concentrations of hormones in patients with depressive disorders, including increased cortisol and other stress-related hormones.

What is kinesthesia?

An awareness of the position and the movement of parts of the body by means of sensory organs. Kinesthesia is much more concerned with movement and behaviour, and is a key component of hand eye coordination and muscle memory.

After a long time of trying to solve the Rubik's Cube, the solver takes a break and later has an "aha" moment and solves it rather quickly. What is this an example of? (A) Fixation (B) Representativeness (C) Means-End Analysis (D) Insight

(D) Insight The solver underwent Incubation by taking a break from the problem, which led to Insight when the solution popped into their head.

Jeffrey struggles with Alcoholism. Instead of getting emotional about his problem, he instead tracks his alcoholic intake, reads up on scientific literature, and plans out his day to avoid triggers. He speaks about his problem using scientific terminology and rarely displays emotions in regards to his problem. Which defense mechanism is Jeffrey most likely implementing? (A) Subilimation (B) Humor (C) Rationalization (D) Intellectualization

(D) Intellectualization Jeffrey is implementing the intellectualization defense mechanism by separating the emotion from the ideas of the problem. He is making something emotional into something more academic.

Jeremy doesn't initially agree with the ideology of the KKK. After attending their meetings with his friends for some time, however, he begins to agree with their values. Even after his friends stop attending the meetings, Jeremy keeps attending and is an avid supporter. Jeremy has fallen prey to which term? (A) Identification (B) Compliance (C) Conformity (D) Internalization

(D) Internalization Jeremy has internalized the values of the group over time, which is in line with Internalization.

Mark is a basketball player, and on the basketball team, it is totally normal to slap another player's butt when they make a basket. During gym class, however, Mark slaps a girl on the butt after she makes a basket. He is charged with Sexual Assault and sentenced to 5 days in prison. Which type of Norm does this example exemplify?

(D) Law In this case, Mark broke a law, and the norms of society were formally enforced.

Which of the following examples best exemplifies Urbanization? (A) A small grocery store is torn down to make way for a skyscraper. (B) A new sidewalk is built, allowing for more pedestrian traffic in a downtown area. (C) A new apartment building cannot accept any new applications as they have too many residents already. (D) Mexicans are moving out of small pueblos in order to move to Mexico City, which is now the largest city in the world.

(D) Mexicans are moving out of small pueblos in order to move to Mexico City, which is now the largest city in the world. Urbanization refers to the moving of more people from rural areas to urban areas.

Which of the Cluster B (dramatic, emotional and erratic) Personality disorders is characterized by having huge egos, a large need for praise, and not handling criticism well? (A) Histrionic (B) Borderline (C) Antisocial (D) Narcissistic

(D) Narcissistic. Narcissistic Personality Disorder is characterized by being egocentric, a large need for praise, and not handling criticism well.

You are conditioning your parents to stop asking you about your personal life. You leave your clothes all over your room except on days that your parents don't ask you about your personal life. Which of the following Operant Conditioning terms is this an example of? (A) Positive Punishment (B) Negative Punishment (C) Positive Reinforcement (D) Negative Reinforcement

(D) Negative Reinforcement Not leaving your clothes on the floor is an example of Negative Reinforcement because you are taking away a negative stimulus.

At the end of the Sensorimotor stage (0-2 years) of Piaget's stages of development, toddlers would fully understand and utilize: (A) Pretend play (B) Symbolic representation (C) Operational thought (D) Object Permanence

(D) Object Permanence

John enjoys being a physician. He finds it to be much more mentally stimulating than his last job cleaning toilets and much less scary than his old job cleaning windows on 100-story skyscrapers. He feels like he is motivated to be at work because it is exciting to the right level. Which theory of motivation is John relying upon to make this explanation? (A) Maslow's Hierarchy (B) Evolutionary Approach (C) Cognitive Approach (D) Optimal Arousal Theory

(D) Optimal Arousal Theory "exciting to the right level" - hint! John has found the optimal level of arousal and it is motivating him to perform his job. In accordance with this, the Optimal Arousal Theory would suggest that John is also performing his job well due to a balanced level of arousal.

Dawn's therapist recommmended that she try cognitive behavioral therapy to cope with her agoraphobia. The therapist hopes that she will become conditioned to these situations with increased exposure, and her fight or flight response will decrease. Which of the following experiments is the therapist hoping to emulate? (A) B.F. Skinner shaping birds' actions to spin around by gradually reinforcing similar behavior. (B) A dog trainer using chaining to teach the dog a 3-minute "dance" routine slowly over time (C) Pavlov's dogs being conditioned to salivate when a bell is rung, since it signals food. (D) Pavlov's dogs, over time, becoming habituated with the bell ringing and salivating less than when first conditioned.

(D) Pavlov's dogs, over time, becoming habituated with the bell ringing and salivating less than when first conditioned. The key idea of this form of cognitive behavioral therapy recommended to Dawn was habituation of the response when faced with the unfamiliar situation.

What are molecules known as specialized olfactory cells that act as chemical signals released by a member of a species and sensed by another member of the same species? (A) Nasal Cells (B) Basal Cells (C) Apical Cells (D) Pheromones

(D) Pheromones Pheromones are molecules known as specialized olfactory cells that act as chemical signals released by a member of a species and sensed by another member of the same species.

Fred believes that he should give what he doesn't need to the poor because he truly cares about them. He tries to see his actions from the viewpoint of everyone else. He believes in principles that can be applied differently in different situations. Which stage of moral development is Fred currently in? (A) Pre-conventional (B) Conventional (C) Post-conventional - Social Contract (D) Post-conventional - Universal Principles

(D) Post-conventional - Universal Principles Fred is focused on universal moral principles.

Which of the following is NOT one of the roles of the Gonads in the Endocrine System? (A) Production of sex hormones (B) Development of Secondary Sexual Characteristics (C) Progressing the body through various sexual stages, like puberty and menopause (D) Production of Urine

(D) Production of Urine The Gonads produce sex hormones, which develop secondary sexual characteristics and guide the body through sexual stages like puberty and menopause.

Natalie knows people who drink and they are cool in her mind. She has also always been someone to try something new. Therefore, she started drinking and going to parties at the age of 13. Which attitude theory best explains Natalie's thought process? (A) Theory of Planned Behavior (B) Elaboration Likelihood Model (C) Attitude-to-behavior Process Model (D) Prototype Willingness Model (PWM)

(D) Prototype Willingness Model (PWM) The Prototype Willingness Model takes into account the role models one has and one's willingness to engage in risky behavior, as is the case here with Natalie.

Which of the following is NOT a Sleep-Wake Disorder? (A) Insomnia (B) Somnambulism (sleep walking) (C) Sleep Apnea (D) REM Deficiency

(D) REM Deficiency Insomnia, Somnambulism and Sleep Apnea are all Sleep-Wake Disorders.

Jack believes that there are 16 traits that can be used to characterize everyone. Which theorist is her thinking most closely in line with? (A) Big Five Theorists (B) Hans Eysenck (C) Gordon Allport (D) Raymond Cattel

(D) Raymond Cattel Raymond Cattel believed that there are 16 traits that can be used to characterize us.

Individuals tend to organize perception of others based on traits and personal characteristics of he target most relevant to the perceiver. This is known as __________________________. (A) The halo effect (B) Attribution theory (C) Implicit personality theory (D) Reliance on central traits

(D) Reliance on central traits Individuals tend to organize perception of others based on traits and personal characteristics of he target most relevant to the perceiver. This is known as *reliance on central traits*.

Which sleep disorder primarily occurs during N3/slow wave sleep and is mostly a genetic disorder, that decreases with time? (A) Narcolepsy (B) Sleep Apnea (C) Insomnia (D) Sleep walking

(D) Sleep walking Sleep walking is a sleep disorder that primarily occurs during N3/slow wave sleep and is mostly a genetic disorder that decreases with time. It is also known as somnambulism.

Which route of drug delivery refers to the drug being absorbed through the skin, and is released into the bloodstream over several hours? (A) Inhalation (B) Intramuscular (C) Injection (D) Transdermal

(D) Transdermal Transdermal drug routes refer to the drug being absorbed through the skin, and is released into the bloodstream over several hours. Ex: Nicotine patches

You will randomly give your cat a treat throughout the day as long as she has continued to avoid scratching the furniture. This example is most illustrative of which partial reinforcement schedule? (A) Fixed-ratio (B) Variable-ratio (C) Fixed-interval (D) Variable-interval

(D) Variable-interval *Variable-interval (VI) schedules* reinforce a behaviour the first time that behaviour is performed after a varying interval of time. After the variable time elapses (e.g. 60s, 30s, 180s, whatever), the behaviour is reinforced the first time it is performed again. Giving your cat a treat at random times is an example of a variable-interval schedule because it is based on a random time table and not on number of actions.

There are three key types of statuses. Compare ascribed, achieved and master statuses

*Ascribed status* is given involuntarily, based on race, ethnic background, gender, family background. (e.g. a male, father) *Achieved status* is a status gained through efforts or choices, such as being a doctor. *Master status* is the most important, all defining status upon which all aspects of that person's life are influenced.

Organizations attempt to achieve maximum through bureaucracies - what is a bureaucracy? Describe the criticism referring to the iron law of oligarchy

*Bureaucracies* are rational systems of political organization, administration, discipline and control, aiming to achieve max efficiency. The *iron law of oligarchy* states that organizations tend to become more bureaucratic over time and naturally shift to a state of being governed only by a select, elite few

Gordon Allport described three basic traits which were of utmost importance in comparison to thousands of others. Describe cardinal, central and secondary traits

*Cardinal traits* - characteristics that influence most of a person's behaviours and ways they organize his life (dominant traits - e.g. selflessness, power-motivation). Cardinal traits influence central and secondary traits. *Central traits* - major characteristics of personality easy to infer, like honesty, shyness or charisma *Secondary traits* - personal characteristics more limited in occurrence - can be described as preferences - e.g. love for modern art, pescatarian or veganism, attitudes

Schizophrenia can also include a positive symptom of disorganized behaviour is catatonia - what does this refer to?

*Catatonia* refers to certain motor movements and tendencies seen in patients with schizophrenia - may include reduction of spontaneous movements, or even rigid posture and refusal to be moved. *Echolalia* (repeating persons words) or *echopraxia* (repeating persons actions) may also be associated with catatonia.

Ivan Pavlov's famous dog experiment was revolutionary in introducing the concept of classical conditioning. Describe classical conditioning and Pavlov's experiment

*Classical conditioning* is a type of associative learning that creates association between unrelated stimuli cues through biological, instinctual responses. Some *unconditioned stimulus* causes an innate, reflexive physiological response (*unconditioned response*) Stimuli that do not result in a response are *neutral stimuli* - they are referred to as *signaling stimuli* if they have the potential to be used as conditioning stimulus. In Pavlov's experiment, he rang a bell (neutral stimuli) before placing meat in the dog's mouths. The dogs initially didn't react to the bell but over time, the dogs began to salivate when hearing the bell - the neutral stimulus (bell) became a *conditioned stimulus* and now produces a reflexive response, a *conditioned response*.

Describe Cultural Lag and how it relates to the concepts of Material and Non-material Culture. Give an example of Cultural Lag.

*Culture lag* results when symbolic/nonmaterial culture is slower to change than material culture, creating some issues. The expansion of tech and devices has definitely led to culture lag!

Different types of governments?

*Democracy* - political voice to everyone *Dictatorship* - one man has all ruling power *Monarchies* - royal ruler with variable levels of power depending on the existence of a constitution *Theocracy* - power held by religious leaders (e.g. partial theocracy for Ayatollah in Iran) Many of these systems of leadership are based around a *charismatic authority* (leader with compelling personality)

Differentiate between *deviance* and *stigma*

*Deviance* refers to any violation of norms, rules or expectations within a society. The term [[deviant]] simply refers to any act that goes against social norms. Deviance is not limited to but also includes acts met with disapproval in society, like pedophilia. Social *stigma* is the extreme disapproval of a person/grouped based on perceived differences from the rest of society, such as beliefs, appearance, behaviours. Examples: Ahmadi's in Pakistan; medical conditions like HIV, obesity, even mental illnesses.

Describe the three theories of deviance: - Differential Association theory - Labelling theory - Strain theory

*Differential association theory* states that deviance can be _learned_ via interactions and exposure to others who engage in deviant behaviour. Differential association is the degree to which one is surrounded by people adhering to social norms vs. people who go against them. "Fallen into the wrong group" *Labelling theory* - a behaviour is deviant if people have judged the behaviour and labelled it as deviant. Labels are given to people which affect not only how people respond to that person, but also the person's self image. This can lead to channeling of behaviour into deviance or conformity. *Strain theory* explains deviance as a natural reaction to disconnect between social goals and resources/structures. Deviant behaviour such as theft may arise as an attempt to reach social goals outside of limiting social structure.

Describe how family can also be a source of violence alongside joy and support.

*Domestic violence* - spousal abuse in form of physical, sexual, emotional or financial abuse. *Child abuse* - neglect, but also physical, emotional or sexual abuse of children *Elderly abuse* - mainly neglecting of elders, but also physical or emotional abuse Physicians are *mandated reporters*, meaning legally obligated to report suspected cases of elder or child abuse.

*Social support* is the perception or reality that one is care for by a social network, and takes many forms. Describe emotional, esteem, material, informational and network forms of social support.

*Emotional support* - listening, affirming and empathazing with someone's feeling (e.g. condolences) - mostly done by family and friends. *Esteem support* - focuses on directly affirming the qualities and skills of a person for confidence. *Material support* - aka *tangible support* - support in the form of finances/materials/goods and resources (e.g. a loan or charity) *Informational support* - providing information that will help someone (e.g. doctors advising patients) *Network support* (aka companion support) - type of support that gives a person a sense of belonging.

*Aversive control* describes situations in which a behaviour motivated by the threat of something unpleasant. Differentiate between escape learning and avoidance learning, two types of aversive control.

*Escape learning* - behaviour with the purpose of reducing the unpleasantness of something that already exists (e.g. leaving a burning building, taking aspirin to treat headache) *Avoidance learning* - behaviour meant to prevent unpleasantness of something that has yet to happen (e.g. Abba avoiding high salty foods to keep his blood pressure normal)

Compare and contrast ethnic identity and national identity

*Ethnic identity* refers to one's ethnic group (members share common ancestry, cultural heritage, language). *National identity* refers largely to our nationality (birth country), and is the result of shared cuisine, history, media, symbols.

Discuss ethnocentrism and cultural relativism.

*Ethnocentrism* refers to the practice of making judgements about other cultures and beliefs based on values/from the perspective of one's own culture. *Cultural relativism* counters ethnocentrism by perceiving and understanding other cultures as different from one's own, but with recognition that cultural values, mores and rules fit into that culture itself.

Different types of statistical studies you should know. Note: the names give a big indication of what the type of study is! Experimental, longitudinal (+ cohort and cross-sectional), case studies, quantitative and qualitative, comparative analyses

*Experimental* - you have a control that allows you to measure the change in one variable in relation to another (always remember though, that correlation is rarely the causation). Independent and dependent variables. *Longitudinal* - a study that follows the same subjects over a long period of time. *Cohort* - a subset of longitudinal study in which the subjects are picked because they have a shared common characteristic or experience within a defined period. *Cross sectional* is a "snapshot" study of a population at a given time. *Case study* - a study in which the subject(s) are hand picked (no random selection) for a detailed analysis. This is useful when encountering a unique subject and intend to gain more information than a regular study. For example, HM was a man with epilepsy and had his hippocampus removed. Researchers studied his unique condition to learn more about the hippocampus and memory. *Quantitative data* - tangible, "hard data", objective information. Examples: height, weight, age, A1c blood glucose test. *Qualitative data* - subjective information (more commonly found in social sciences). Examples: pain rating, mood rating,etc. *Comparative analysis* studies the difference between different populations. Example: how local diet influences health in US vs Japan.

Three factors which contribute to population increase are fertility, mortality and migration. Differentiate between the three.

*Fertility rate* refers to the average # of children born to a woman during her lifetime. In US - declined over time, but still remains above 2, hence contributes to increasing population. *Mortality rate* refers to # of deaths in a population per unit time - measured in deaths per 1000 people per year. Declined in the US with improving healthcare and access, which also contributes to population growth. Decreased mortality rate = _increase in average age_ of population. *Migration* - immigration increases population size, emigration decreases. All rates are measured per 1000 people, so that they can be compared and used to determine overall population growth rate.

Compare secure and insecure attachments

*Secure attachment* is seen when a child has a consistent caregiver and is able to go out and explore with the knowledge of a secure base to return to. Secure attachment is thought to be a vital aspect of a child's social development. Insecure attachments have three types: avoidant, ambivalent and disorganized attachments.

There are several types of anxiety disorders (GAD, phobias, panic disorders, social anxiety disorder, Agoraphobia, PTSD, OCD). Describe Generalized Anxiety disorder (GAD).

*Generalized anxiety disorder (GAD)* - defined as disproportionate and continuous feeling of anxiety/worry and tension about many things for at least six months. People tend to be very tense with an unclear source. Physical symptoms may include fatigue, muscle tension, sleep problems. 2/3 patients are female.

Discuss the last two stages of Erikson's psychosocial theory, which emphasizes emotional development and interactions with the social environment: - Generativity vs. stagnation - Integrity vs. despair

*Generativity vs. stagnation* (40-65) - virtue is care - successful resolution leads to an individual capable of being productive, caring and contributing to society; unfavourable outcome is a sense of stagnation, self-indulgent, self-centered maybe. *Integrity vs. despair* (65+) - virtue is wisdom - favourable resolving leads to wisdom, feeling of a worthwhile life and productivity, dignity, and readiness to face death with a sense of closure; unfavourable outcome is a sense of bitterness, despair and dissatisfaction upon death.

Describe globalization and its causes + effects

*Globalization* - process of integrating the global economy with free market trade and tapping into foreign markets. Caused by improved global communication and transportation technologies, as well as economic interdependence. Positive effects include cultural exchanges and things such as the availability of food, which requires trade with a very large number of world markets. Negative effects include rising prices, worldwide unemployment increase, increased pollution, civil unrest and global terrorism.

A deficiency of iodine or inflammation may result in hypothyroidism, while an excess of T3 and T4 may cause hyperthyroidism (due to a tumor of overexpression) - what characterizes these conditions?

*Hypothyroidism* is characterized by lethargy, decreased body temp, slow respiratory and heart rate, weight gain, cold intolerance. *Hyperthyroidism* is characterized by heightened activity level, increased body temp, increased heart and breathing rates, heat intolerance, weight loss.

Discuss the second three stages of Erikson's psychosocial theory, which emphasizes emotional development and interactions with the social environment: - Industry vs. inferiority - Identity vs. role confusion - Intimacy vs. isolation

*Industry vs. inferiority* (6-12y) - virtue is competence/pride - favourable resolving leads to a child feeling competent, intelligence, high self-esteem and feels able to exercise his/her abilities to serve and affect the world; the unfavourable outcome though would be a sense of inferiority, low self-esteem, and inability to act in a competent manner. *Identity vs. role confusion* (12-20y) - virtue is identity, this conflict encompasses *_physiological revolution_* - favourable outcome is fidelity, sense of identity and personality, integrated person; unfavourable outcome is confusion about identity and amorphous personality. *Intimacy vs. isolation* (20-40y) virtue is love and intimacy - favourable outcomes are love, commitment abilities, intimate relationships; unfavourable outcomes are avoidance of commitment, alienation, distancing

Differentiate between internalization and identification - how do these relate to private and public conformity?

*Internalization* is essentially private conformity - involves changing one's behaviour to fit with a group _while also agreeing internally_ with the ideas of the group internally *Identification* is essentialy public conformity - outward acceptance of others' ideas without personally agreeing.

What is a Lewy Body, and what role does it play in Parkinson's Disease?

*Lewy Bodies* are found in the substantia nigra in Parkinson's Disease, and mainly contain *alpha-synuclein*, a normal protein that will aggregate into these bodies and cannot be cleared. These bodies can be distributed amongst the cortex too, and may or may not be what kills the dopaminergic neurons.

Describe the following age theories: - Life course theory - Age stratification theory - Activity theory - Disengagement theory - Continuity theory

*Life Course theory* - ageing is a socio-psych-biological process that runs throughout life. Biological process of living longer has influenced social process - age based expectations no longer apply. Can't *Age stratification theory* - regulating activities based on age (no drinking till 21, no driving till 18 etc) *Activity theory* - looks at how elders look at themselves (e.g. inability to do some activity may influence morale and well being). *Disengagement theory* - suggests older adults and societies separate - sees people as more self absorbed after leaving work force or major parts of society. *Continuity theory* - people try to maintain same basic structures of their lives over time

Describe the Malthusian theory

*Malthusian theory* focuses on how exponential population growth can outpace growth of food supply, leading to social disorder and degradation. A malthusian catastrophe is the prediction that as a 3rd world nation industrializes and undergoes demographic transition, the pace at which the world population will grow is much faster than ability to generate food - leads to mass starvation.

Material culture is often the tangible embodiment of symbolic culture - can you describe this statement? What are material and symbolic culture?

*Material culture* refers to artifacts and tangible, physical objects given meaning - can include artwork, food, emblems, clothing, buildings, tools, etc. *Symbolic culture* is also called nonmaterial culture - focuses on ideas that represent a group of people - encoded in mottos, songs, catch-phrases, themes, etc. It has cognitive and behavioural components - informs cultural values + beliefs, as well as norms and communication styles. Symbolic culture usually is represented in material culture through these components.

What is McDonaldization?

*Mcdonaldization* is a term used to refer to a shift in focus towards more efficiency, predictability, calculatability and control in societies - stemmed from McDonalds chain but applies so many many more institutions.

Norms vary amongst different societies and cultures, and can change over time. Describe the following terms: - Mores - Sanctions - Folkways - Taboo - Laws

*Mores* are widely observed social norms. While norms are not laws, they are a means of social control. *Sanctions* - penalties for misconduct or rewards for appropriate behaviour, also used to maintain social control *Taboo* - socially unacceptable, disgusting or reprehensible behaviour *Folkways* - norms referring to behaviour considered polite in social interactions (e.g. handshake) *Laws* are norms based under a concept of right and wrong, but with legal backing and punishable sanctions.

Define multiculturalism

*Multiculturalism* is when communities contain multiple cultures - encourages and respects cultural differences, increasing cultural diversity and acceptance within a society. Creates a cultural mosaic or melting pot of cultures (e.g. Singapore, NYC)

Define organizations and the characteristics of formal organizations

*Organizations* are institutions or entities designed to achieve specific goals and have their own structure and culture. e.g. companies, schools, sports teams, music groups, etc. Formal organizations have some characteristics; (1) continue despite departure of individual(s); (2) have common expressed goals that guide the members; (3) have enforcement procedures that controls activities of members; (4) hierarchal allotment.

The stereotype content model classifies groups in four possible combos of warmth and competence - describe: - Paternalistic stereotypes - Contemptuous stereotypes - Envious stereotypes - Admiration stereotypes

*Paternalistic* stereotypes are those where the group is looked down upon as inferior to the in-group. (high warmth, low competence) *Contemptuous* stereotypes are those where the group is viewed with resentment and anger (low warmth and competence) *Envious* stereotypes are those where the group is viewed with bitterness and jealousy (low wamrth high competence) *Admiration* stereotypes are those in which the group is viewed positively with pride (high wamrth high competence)

A variety of social factors influence both prejudice and discrimination. Describe power, prestige and class and how they influence both.

*Power* refers to the ability of people to achieve their goals and control resources despite obstacles. *Prestige* refers to the level of respect shown to a person/group by others. *Class* refers to socioeconomic status. Social inequality (unequal distribution of power, resources, money, prestige) influences prejudice and discrimination by grouping of /haves/ and /have-nots/ - /haves/ may hold a negative view of /have-nots/, and vice versa, leading to attitudes and behaviours.

Mead believed that in young years, we are ego-centric and focus on ourselves, but beliefs of how others perceive us become more important as we grow older in 3 particular stages - describe: - Prepatory stage - Play stage - Game stage

*Prepatory stage* - children interact with others through imitation, which is not real interaction, but towards the end they are focused more on communication through symbols and words unmastered, etc. rather than just imitation. *Play stage* - children become aware of importance of social relationships - pretend play; taking on perspectives of others and acting based on their perceived point of view - mimicing AND creating social interactions *Game stage* - children begin to understand attitudes and behaviours of the "generalized other" (society as a whole) - performance based on how people want to act as well as society's expectations of them

Differentiate between primary and secondary groups.

*Primary groups* include people that share direct interactions with close bonds and enduring relationship. These groups often last a long period and may include core friends circle, family, members of a team, etc. *Secondary groups* include formal, impersonal interactions based on a specific limit and goal - short term and form + dissolve without much significance (e.g. students working on a group project)

Differentiate between the following: - Primary socialization - Secondary socialization - Anticipatory socialization - Resocializaton

*Primary socialization* occurs in childhood where we initially learn acceptable actions + attitudes *Secondary socialization* is learning appropraite behaviour in smaller sections of society, such as schools, church, sports field, etc. *Anticipatory socialization* is process by which a person prepares for future changes in occupations, living situations, etc. *Resocialization* is the process of discarding old behaviours in favour of new ones to make a life change - e.g. army training for obedience

Match the defense mechanism to it's appropriate description: - Rationalization - Displacement - Sublimation (A) Justification of behaviours in manners acceptable to self and society, e.g. smoker believes he is in complete control of amount he smokes and can quit whenever (B) Channeling of an unacceptable impulse in a socially acceptable direction, e.g. pent-up sexual urges expressed in drive for artistic creativity or business success (C) Changing the target of an emotion while maintaining the same feelings; e.g. man angry at his boss holds his tongue and snaps at his spouse when home

*Rationalization* - (A) Justification of behaviours in manners acceptable to self and society, e.g. smoker believes he is in complete control of amount he smokes and can quit whenever *Displacement* - (C) Changing the target of an emotion while maintaining the same feelings; e.g. man angry at his boss holds his tongue and snaps at his spouse when home *Sublimation* - (B) Channeling of an unacceptable impulse in a socially acceptable direction, e.g. pent-up sexual urges expressed in drive for artistic creativity or business success

Match the defense mechanism to it's appropriate description: - Repression - Regression - Reaction formation - Projection (A) Individuals attribute their undesired feelings to others, e.g: /I hate my parents/ turns into /My parents hate me/ (B) The ego's way of forcing undesired thoughts and urges to the unconscious, e.g. to disguise threatening impulses (C) Individuals suppress urges by unconsciously turning them into exact opposites (D) Reversion to an earlier state of development due to stress, e.g: child returns to thumb-sucking or tantrums

*Repression* - (B) The ego's way of forcing undesired thoughts and urges to the unconscious, e.g. to disguise threatening impulses *Regression* - (D) Reversion to an earlier state of development due to stress, e.g: child returns to thumb-sucking or tantrums *Reaction formation* - (C) Individuals suppress urges by unconsciously turning them into exact opposites *Projection* - (A) Individuals attribute their undesired feelings to others, e.g: /I hate my parents/ turns into /My parents hate me/

What are neuroleptics and antipsychotics?

Antipsychotics, also known as neuroleptics or major tranquilizers, are a class of medication used to manage psychosis (delusions, hallucinations, paranoia, disorganized though, even at times bipolarity and schizophrenia).

Define and compare the terms self concept, self schema and identity.

*Self-concept* refers to how we think about, perceive and evaluate ourselves. To be self aware is to have a self-concept. *Self-schema* - a self given label that carries with it a set of qualities; a category we fall under *Identity* refers to the individual components of our self concept related to the groups in which we belong. Whereas we all have one-all encompassing self-concept, we have multiple identities (religious affiliation, sexual orientation, etc).

What are social movements? Differentiate between proactive and reactive social movements.

*Social movements* are organized to either promote (*proactive*) or resist (*reactive*) social change, often motivated by a perceived *relative deprivation*. E.g. of proactive social movements include civil rights, movement, women's suffrage movements, gay rights movements etc. E.g. of reactive include white supremacist movements, anti-immigration movements, etc.

Describe the socioeconomic gradient of health

*Socioeconomic gradient of health* refers to the socioeconomic disparity in health care. Those at the top of the socioeconomic pyramid tend to have better health outcomes than those at the bottom.

Demographic shift refers to changes in population makeup over time, and *demographic transition* is a specific example of demo- shift referring to changes in birth and death rates as a country develops from a preindustrial to industrial economic system. Describe how this transition was seen in the US during Industrial Revolution in its 4 stages.

*Stage 1* - preindustrial society with high birth and death rates *Stage 2* - improvements in healthcare, nutrition, wages, sanitation lowered death rates *Stage 3* - improvements in women's rights, contraception, and shift from agricultural to industrial economy caused birth rates to drop. *Stage 4* - industrialized society with low birth and death rates.

Compare Status to Role. What might the Status and Role of a doctor be?

*Status* refers to a social position used to classify individuals - because in most societies people are not viewed as equal with same amount of respect - some are loved, liked, disliked, even stigmatized. Each status has a *role* (set of beliefs, values, attitudes and norms) that defines expectations of the holder of said status. For a doctor, his status being a doctor; his role would be to help sick people.

Discuss the first three stages of Erikson's psychosocial theory, which emphasizes emotional development and interactions with the social environment: - Trust vs. mistrust - Autonomy vs. shame & doubt - Initiative vs. guilt

*Trust vs. mistrust* is the first conflict (0-1y) where the virtue is hope - resolving of this conflict leads to a child who trusts his environment and self, whereas mistrust winning out results in the child being suspicious of the world. *Autonomy vs. shame and doubt* (1-3y) where the virtue is will - the favourable outcome of resolving this conflict is the feeling to be able to exert control and exercise choice and self-restraint. However, the unfavourable outcome is a sense of doubt, shame, and/or persistent external locus of control. *Initiative vs. guilt* (3-6y) where the virtue is purpose - favourable outcomes include a sense of purpose, ability to initiate activities and enjoy accomplishment; the flipside outcome is inadequacy, child that may unduly restrict himself, or compensate by showing off.

The hypothalamus releases two direct hormones into the posterior pituitary. What are they and what do they do?

- ADH (Antidiuretic hormone, aka vasopressin) increases reabsorption of water in collecting ducts of the kidneys + is secreted in response to plasma osmolarity or increased solute concentations in the blood. - Oxytocin - stimulates contractions during labor + milk letdown during lactation.

The release or inhibition of PTH is controlled by the amount of calcium in our blood. When Ca levels drop, PTH is released - talk about how PTH not only increases Ca levels, but also in terms of phosphorus homeostasis.

- Increases reabsorption/decreases excretion of calcium by kidneys - Increases absorption of calcium in the gut - Increases bone resorption of Ph and Ca - Reduces kidney reabsorption of Ph

What are some of the brain abnormalities in the brain seen in schizophrenics?

- Less tissue and abnormal layering on cerebral cortex - Abnormalities in neuro-pathway that involves the NT dopamine in the frontal and temporal lobes of the brain. One pathway in particular that has attracted attention is the mesocorticolimbic pathway from the VTA to other areas of the brain. - Larger ventricles in the brain due to reduced tissue size of the brain as a whole

What are two problems associated with urbanization?

1) sociologists have found links between urban societies and health issues such as sanitation of water, air quality, environmental hazards, violence, infection, unhealthy diets 2) they are rarely homogenous, and some areas are socioeconomically well off at the expense of others which are impoverished - e.g. the ghettos for racial specific minorities, slums

There are three main underlying premises of the Rational Choice Theory. Describe each of the following premises in terms of choosing between options A, B, and C: 1. Completeness 2. Transitivity 3. Independence of Irrelevant Alternatives

1. Completeness - A, B, C have different values and can thus be ranked as follows: A > B > C 2. Transitivity - If A > B > C then A must also be > than C 3. Independence of Irrelevant Alternatives - If you add in option Z then this new option will not change anything and is not preferable to A, B, nor C.

One way we forget, of course, is brain disorders: What is Alzheimer's disease?

Alzheimer's disease is the most common disorder leading to memory loss - thought to be linked to loss of acetylcholine in neurons. It is marked by progressive dementia (loss of function) and memory loss, and usually happens in a retrograde fashion (new memories lost before distant memories)

Freud believed human psychology and sexuality were inextricably linked - asserted that the *libido (sex drive) is present at birth and hypothesized five distinct stages of psychosexual development - order the 5 stages of Psychosexual Development from earliest to latest: (1) Genital (2) Phallic (3) Anal (4) Oral (5) Latent

4, 3, 2, 5, 1 Oral -> anal -> phallic -> latent -> genital

Differentiate subcultures and countercultures

A *subculture* is subset of dominant culture - a meso-level subcommunity that distinguishes itself from the primary culture and society - has its own symbolic attachment to things, and can be formed based on race, ethnicity, sexuality, gender, etc. They share some characteristics of the dominant culture of the society they're in. *Counterculture* - subset of the dominant culture that opposes and goes to counter the main culture (e.g. cults, motorcycle gangs, etc.)

What is a flashbulb memory, and are they susceptible to reconstruction?

A flashbulb memory is an extremely emotional, vivid memory (positive or negative). They are still susceptible to reconstruction. A person after experiencing something like say, a traumatic experience, will be likely to remember the memory and things done on that day very vividly - this is the concept of flashbulb memories.

For highly addictive drugs, ___________ is common because it is easy for patients to slip and go back to old habits and environments. A) Relapse B) Cross tolerance C) Intoxication D) Withdrawal

A) Relapse

Describe sex cortisol hormones released by the adrenal cortex

Adrenal cortex releases small amounts of male sex hormones (test) - more impactful in females than males, since males have testis. Androgen deficiencies can result in problems in females - excess for e.g. can form ambigious genitals in females.

Describe aggression, forms it can take, and purpose.

Aggression is defined as behaviour, physical or verbal, that intends to cause harm, increase social dominance, or destroy. Aggression can be in the form and range from physical action and harm, to verbal communicaition, and even bodily contact or displays of threat. Threat display is common in both humans and other animals. Aggression offers evolutionary purpose - protection against perceived and real threats - helps ancestors fight off predators, and organisms to gather food and resources, territory or mates + gene propogation.

How do the functions of the external and middle ears differ from the inner ear?

All three portions of the ear are involved in hearing, but the inner ear is also involved in maintaining your equilibrium (proprioception).

As one becomes addicted to cocaine, how does the brain develop a tolerance causing the individual to take more of the drug to get high?

As the brain gets influxes of dopamine consistently, it adapts in order to balance out the unnatural levels of "happiness" or "pleasure" received from the dopamine. Thus, it shuts down some post-synaptic receptors to regulate this. As a result, as one becomes addicted to cocaine, the brain adapts and shuts down some dopamine receptors, leading to the individual needing more cocaine to achieve the high.

At the optic chiasm, how are the nasal and temporal visual fields sorted? What purpose does this serve?

At the optic chiasm, rays from the nasal field cross at the chiasm while those from the temporal do not. This allows all the information from the right visual field to go to the left side of the brain, the from the left visual field to the right side of the brain. Both eyes see both visual fields to a degree, our brain converts it into one image.

Bandura's Social Cognitive Theory states that you are reliant upon four factors that will determine whether or not you will learn something through observation. What are these four factors (Hint: Am I Motivated?)? Describe each and how it might apply to the Bobo Doll Experiment.

Attention - Were the kids paying attention to the video of the man beating up the Bobo Doll? Memory - Did the kids remember what the man did and said? Imitation - Were the kids physically able to kick and punch? Motivation - Were the kids motivated to beat up the Bobo Doll?

How is attention split between multiple tasks, and why does this make multi-tasking less effective?

Attention is not actually split between two tasks simultaneously, but rather only focuses on one at a time. These gaps in attending to each task makes multitasking ineffective.

Differentiate between autocrine, paracrine and endocrine hormones

Autocrine hormones function at the cell Paracrine hormones function regionally, outside the cell Endocrine hormones function at a distance

Read this paragraph about the Prototype WIllingness Model: http://wiki.scetool.nl/index.php?title=Prototype_Willingness_Model What does it mean to say that the Prototype WIllingness Model is a Dual-processing Model? Can you relate this to a child who decides to smoke for the first time?

Basically, this theory says that there are two paths of influence on this child's behavior. The first path is rational and analytical (i.e. "It is okay to smoke because I've never been taught the consequences") . The second path is the social reaction path, which involves prototypes ("I know people who smoke and they are cool") and willingness to engage in the risky behavior ("I'm a daredevil, risk-taker kind of person").

Benzodiazepines, often shortened to "Benzos", can be prescribed for conditions from insomnia to anxiety. If their mechanism is opening chloride channels, how does "Benzos" repress insomnia and anxiety?

Benzos open up GABA (inhibitory NT) activated chloride channels on neurons - allowing more Cl- ions to flow in. The negatively charged anions makes the cell more resistant to excitation - thus they are used to repress anxiety or induce sleep/counter insomnia.

The part of the retina that is directly in front of where the optic nerve exits the eye is known as the _____________.

Blind Spot. The Blind Spot is directly in front of where the optic nerve exits the eye - contains no rods or cones.

What is conversion disorder?

Conversion disorder (formerly hysteria) is characterized by unexplained symptoms with neurological basis that affect voluntary motor or sensory functions. Usually begin after something traumatic or stressful and develops over time - e.gs include paralysis or blindness without evidence of neurological damage. Individuals may be unconcerned about the symptom - la belle indifférence

P.K.U (which codes for a liver protein) disorders can lead to a baby getting a buildup of phenylalanine and suffer neural defects. Giving the baby a phenylalanine-free diet will make the baby asymptomatic. Is P.K.U. caused by nature (genes), nurture (environment), or both?

Both genes and environment. Like many diseases, there is a genetic component (lacking the protein to breakdown Phenylalanine), but altering the environment appropriately (consuming a phenylalanine-free diet) prevents the disease from manifesting.

Compare the functions of the Visuo-Spatial Sketchpad with the Phonological Loop.

Both the Visuo-Spatial Sketchpad and Phonological Loop are part of Working Memory. The Visuo-Spatial Sketchpad is used for visual information (like schedules and maps) whereas the Phonological Loop is used for verbal information (words and numbers). Modality is not the way information is acquired, so whether numbers are heard (echoic) or seen (iconic memory), they will go to the phonlogical loop.

Pablo has been struggling with eating habits, and wants to improve his self control. Give an example of of how each of the following strategies can develop self control: - Changing the environment - Operant conditioning - Classical conditioning

By changing the environment: Pablo could change his environment by limiting the unhealthy options around him. Via operant conditions: Pablo could use operant conditioning by using positive reinforcement, like for every full day he eats healthy, he allows himself to watch sports. Via classical conditioning: Pablo could use classical conditioning by associating his favorite junk food with healthier alternatives, and develop a conditioned response.

____________ refers to drug-specific behavioral and psychological effects on a person when the drug enters the body. A) Tolerance B) Withdrawal C) Intoxication D) Cross tolerance

C) Intoxication

How is collective behaviour different from group behaviour?

Collective behaviour is usually time-limited (short), open to participants, and loose norms Group behaviour is usually long, exclusive and has strongly held norms.

When talking about biases, what is overconfidence?

Confirmation bias contributes to overconfidence, the tendency to interpret ones decisions, knowledge and beliefs as infallible.

How can Marxist conflict theory be applied to healthcare and medicine?

Conflict theory would not deny that healthcare and medicine aim to help people, but it may question and ask who holds the power in the healthcare system? Is it the patient? Doctor? Hospitals? Pharma companies? Insurance companies? The government? The US continues to grapple with this issue!

Differentiate between conformity, compliance and obedience

Conformity is how we adjust our attitudes and behaviours in order to go along with the group. Compliance is doing as you are asked even when you don't want to. (e.g. paying taxes) Obedience describes how we obey authority and follow order - does not involve cognitive factor that conformity does. (e.g. Nazism)

What are some of the emotional behaviour effects of stress?

Depression, anger, anxiety/fear and addiction

A man comes into the ER room wearing steel-toed boots, a mesh hat and a RAM 1500 t-shirt, and is carrying his detached finger with him in a toolbox. Is it more likely that he is a construction worker, or a truck-driving construction worker? How might this be an example of the Conjunction Fallacy?

Despite any context clues, it is statistically more likely that the patient falls under just the label of "construction worker" than both "construction worker" and "truck driver". This is a clear example of the Conjunction Fallacy.

What are some ways people can cope with stress?

Develop a sense of perceived control over a situation Social support Nurturing an optimistic attitude

How does the auditory system break down complex sound waves into its component parts?

Different sound waves of different frequencies travel different distances in the cochlea and move different hair cells. Lower frequencies travel farther (towards the apex) in the cochlea; high at the base.

True or False? Children who engage in lots of Private Speech tend to fall behind in development because they don't spend enough time speaking with others.

False. Children who engage in lots of Private Speech tend to develop faster cognitively. Vygotsky saw speech as an accelerator for learning. He saw thought (Inner Speech) as developing out of Private Speech. Vygotsky believed that thought (which is a higher mental function, independent) emerged from language.

True or False? There is a certain personality type that is more likely to obey and conform than others.

False. There isn't a certain personality type that is more likely to obey and conform than others. However, different socioeconomic and cultural groups may be a little more likely.

Considering the proximity of the receptors for different flavors. How does the Labelled Lines Model account for the 5 flavors not mixing?

Each one of the specialized taste cells has dedicated axons that remain separate all the way to the brain and synapse at dedicated spots in the brain without mixing.

How has the understanding of disease shifted over time?

Entities formerly defined as illnesses like homosexuality and transgenderism are no longer thought of as so. At the same time, formerly neglected problems and disorders have now been *medicalized*, such as the disorders in the DSM-5.

How does Rational Choice Theory relate to Exchange Theory?

Exchange Theory is Rational Choice Theory as applied to social interactions. It is the idea that if actions receive social approval, they are more likely to be repeated.

Becky is one of those people who is never upset. She is always calm and collected. After hanging out with friends, she tends to get worn out. Sometimes she believes that some of her friends are out to get her even though they are not. Hans Eysenck believed that Extraversion, Neuroticism, and Psychoticism were the three major traits of one's personality. How do these three traits apply to Becky?

Extroversion is the degree to which someone is outgoing. Becky would be seen to have little of this trait. Neuroticism is one's emotional stability, which is sounds like Becky has a lot of. Psychoticism is the degree to which reality is distorted in someone's view. In this case, Becky's view of reality is distorted so she would score high in psychoticism.

True or false: Between the central and peripheral processing routes, peripheral processing routes have greater likelihood of inducing long-lasting attitude change.

FALSE - central processing routes have greater likelihood of producing a long-lasting attitude change.

True or false: assimilation can be sped up by the creation of *ethnic envlaves* (locations w/ high concentrations of specific ethnic group).

FALSE: assimilation can be SLOWED DOWN by the creation of *ethnic envlaves* (locations w/ high concentrations of specific ethnic group).

True or false: fundamental attribution error is not influenced by culture and remains the same in individualistic societies as opposed to collectivist.

FALSE: fundamental attribution error IS influenced by culture and INCREASES in individualistic societies as opposed to collectivist.

True or False? The original IQ tests are completely independent of and not affected by the culture of the test-taker.

False.

True or False. Population Dynamics looks at the movement of a population from one region to another as in urbanization.

False. *Population Dynamics* looks at the rise or fall in a population. Is the population growing in size or decreasing?

True or False? According to Relative Deprivation Theory, if Jimmy makes $20,000 per year and feels good about his income, Mary, his next door neighbor must share similar feelings.

False. According to Relative Deprivation Theory, if Jimmy makes $20,000 per year and feels good about his income, Mary, his next door neighbor might be feeling something entirely different depending on who she is comparing herself to. This theory suggests that people join a social movement when they have a sense of RELATIVE deprivation and feel like they can't get what they want through conventional means.

True or False? Because the different personalities in Mulitiple Personality Disorder come from the same body, there are no "physical" changes.

False. Although the different personalities in Multiple Personality Disorder come from the same body, there can be physical changes, including the dominant hand.

True or False? At the end of each projection that the olfactory bulb sends through the cribriform plate to the olfactory epithelium are receptors that sensitive to many molecules each.

False. At the end of each projection that the olfactory bulb sends through the cribriform plate to the olfactory epithelium are receptors that sensitive to ONLY 1 MOLECULE each.

True or False. Attitudes are something innate and unchangeable as they are closely intertwined with one's personality.

False. Attitudes are defined as "learned tendencies."

True or False. Attribution means inferring that someones behavior is caused by external factors.

False. Attribution is the act of inferring the cause of the behavior of others, whether that cause is internal or external. Distributional attribution is inferring it to internal factors, situational attribution is inferring it to external.

True or False? Object Permanence describes how older children can know that two cups hold equal volumes of water, even if the cups are different shapes, if the water is poured in front of them.

False. CONSERVATION describes how older children can know that two cups hold equal volumes of water, even if the cups are different shapes, if the water is poured in front of them.

True or False? Catecholamines are produced in the cortex of the adrenal glands.

False. Catecholamines are produced in the MEDULLA of the adrenal glands. The cortex synthesizes glucocorticoids, like Cortisol.

True or False? Cones adapt to change in illumination very slowly, thus when one walks from a brightly lit room into a dimly light room, one is not able to clearly see color and can't make the transition very quickly.

False. Cones adapt to change in illumination very QUICKLY, thus when one walks from a brightly lit room into a dimly light room, one is ABLE to clearly see color and the make the transition fairly quickly

True or False? Deviance is always a bad thing.

False. Deviance is relative and simply means that you are breaking a norm. Sometimes it can be good to break a norm such as Rosa Parks did during the Civil Rights Movement.

True or False? Emotions are generally considered a voluntary response.

False. Emotions are generally considered an INVOLUNTARY response.

True or false: environmental factors and stimuli cannot induce the turning on of genes to produce proteins and elicit a response

False. Environmental factors and stimuli CAN induce the turning on of genes to produce proteins and elicit a response

True or False? Because both echoic and iconic memory fall under sensory memory, they both last the same amount of time.

False. Even though both echoic and iconic memory fall under sensory memory, the MODALITY (form of acquiring the memory) will determine how long the memory lasts for. For instance, echoic memory lasts for about 4 seconds while iconic memory only lasts for 1/4 of a second!

True or False: Heritability is the percentage for how much of a trait is due to genes.

False. Heritability is the percentage for how much VARIATION of a trait is due to genes. Heritability increases as environment becomes more controlled between subjects; decreases as it becomes less controlled.

True or False? Mania is similar to Action Potentials in the sense that it is an all-or-nothing occurrence.

False. Hypomania is when more mild versions of symptoms of Mania are exhibited, and the symptoms never develop into full-blown Mania.

True or False? In animal models of major depressive disorders, an increase in neuroplasticity has been noticed.

False. In animal models of major depressive disorders, a DECREASE in neuroplasticity has been noticed.

True or False? In the Milgram experiment, when the experimenter held a clipboard, obedience of the participant decreased because they knew the experimenter should have known better.

False. In the Milgram experiment, when the experimenter held a clipboard, obedience of the participant increased due to the increased authority of the person giving them the command.

True or False? In the Milgram experiment, when the experimenter was standing right behind the participant, obedience of the participant decreased because they knew the experimenter more intimately.

False. In the Milgram experiment, when the experimenter was standing right behind the participant, obedience of the participant increased because obedience increases as physical proximity and closeness increase.

True or False? One's Looking Glass Self is extremely accurate due to it being based on the objective opinions of others and not our own thoughts about ourselves.

False. One's Looking Glass Self is extremely INaccurate because it is based on how we THINK others THINK about us. It is very subjective.

True or False. Rational Theory is based on the idea that the best choices are made by individuals who are thinking rationally.

False. Rational Theory is based on the idea that the every choice we make is rational and based on a cost-benefit analysis in which we try and maximize our personal gain.

True or False? Schizophrenia is the diagnosis if a patient exhibits multiple personalities.

False. Schizophrenia is the diagnosis if a patient has delusions and/or hallucinations that affect the way they act, including isolation, disorganization and a flat affect.

True or False? Symbolic Interactionism would say that two people cannot look at the same image and see two different things.

False. Symbolic Interactionism is based on the idea that two people can look at the same image and see two different things. To one person, a cross might sybolize death. To another it might symbolize healthcare. It is dependent upon their perspective.

True or False? The Fundamental Attribution Error is more common in Collectivist Cultures such as Asia.

False. The Fundamental Attribution Error is more common in Individualistic Cultures such as the United States due to a greater focus on individuals and their ability to control their situation. In Collectivist Cultures, successes are much more likely to be credited to external factors, and failures blamed on internal factors.

True or False? Charles Cooley's concept of The Looking Glass Self refers to how you view yourself through your own eyes.

False. The Looking Glass Self refers to how you view yourself through the eyes of others.

True or False? Social Institutions are temporary and may disappear after short periods of time.

False. The actual format of a Social Institution may change but it is still there. For instance, even if the Educational System were to change from public schools to charter schools, it would still be the Educational System, which will endure despite changes.

True or False? The main male reproductive issue regarding chronic stress is decreased levels of testosterone to the point of infertility.

False. The main male reproductive issue regarding chronic stress is impotency, or Erectile Dysfunction (ED), due to peripheral vasoconstriciton.

True or False? The potential causes of Schizophrenia are Genes and Physical Stress only.

False. The potential causes of Schizophrenia are Genes and Physical Stress and Psychosocial factors, like family interaction styles in childhood.

True or False. The removal of a negative factor can replace a reward in Incentive Theory, similar to negative reinforcement and positive reinforcement in operant conditioning achieving similar outcomes.

False. The removal of a negative factor (negative reinforcement) cannot replace a reward (positive reinforcement) in Incentive Theory

True or False. The sooner a reward is given after an accomplishment/action, the less likely it is to occur again. Behavior is best rewarded at random and inconsistently as in variable-ratio conditioning.

False. The sooner a reward is given after an accomplishment/action, the more likely it is to occur again.

True or False? Words that are read will be stored in the Visuo-Spatial Sketchpad of the Working Memory.

False. Words that are read will be stored in the Phonological Loop. Remember, it is the type of information that determines where in the working memory it is stored, not the modality of acquiring the information!

True or false: The thalamus acts as a relay station in the limbic system for all senses.

False: It is the relay station for all senses EXCEPT for the sense of smell!

True or false: consumption of alcohol with benzos or barbituates does not have profound negative side effects.

False: consuming alcohol and overdoses of benzos or barbituates can result in coma or death

True or false: Hypnosis has had little to no success in real world medicine or therapy.

False: hypnosis has been used successfully for pain control, psych- therapy, memory enhancement, weight loss, smoking cessation - the success or effectiveness, however, highly depends on how willing the subjects personality is + lack of skepticism

True or false: the tympanic membrane is part of the middle ear

False: it is considered part of the external ear

True or false: Between these three, semantic encoding is the weakest form of encoding: - Semantic encoding - Visual encoding - Auditory encoding

False: semantic encoding is the strongest while visual is the weakest.

True or false: when someone tells you their IQ score, they are telling you their practical intelligence (from Sternberg's 3 intelligences)

False: the IQ test measures what would be analytical intelligence on Sternberg's theory (or linguistic and logical-mathematical on Gardner's) The Stanford-Binet IQ test is: IQ=mental age/chronological age x100

True or false: extremely vivid and emotional memories are not susceptible to memory reconstruction.

False: these memories (flashbulb memories) are indeed still susceptible to memory reconstruction

True or false: during meditation the EEG sees beta waves from the brain

False: theta and alpha waves are seen, resembling Stage 1 of sleep.

True or false: when we are awake, cortisol levels tend to be lower

False: when we are awake, there is a physiological level of arousal and cortisol levels tend to be HIGHER.

Describe how the family functions as a social institution

Family can be defined by many *patterns of kinship*, not just the small nuclear family perceived as family in the US - varies across cultures. Family is influenced by beliefs, practices, gender, age, race, ethnicity, culture and value systems etc.

What are some agents of socialization?

Family, schools, workforce, peers, ethnic/religious background, mass media Primary agents are personal relationships - for children are family; for adolescents and adults, this can expand to peers and colleagues+bosses (respectively)

Innate behaviours are broken down into simple and complex types. Describe the complex innate behaviours of fixed action patterns, migration, and circadian rhythms.

Fixed action patterns are complex innate behaviours that already ingrained within the organism for a purpose. Example includes mating dance in birds for attracting a mate. Migration is another complex form of an innate behaviour, such as migration of birds to the south. Circadian rhythms are complex innate behaviour relating to sleep cycles. For a bird, example would be waking up early morning to sing

In America, white people generally hand shake while black people fist bump upon greeting. In Japan, people bow when greeting. These are examples of what?

Folkways, and norms varying across cultures and societies.

Ford markets itself as an American Company and has its headquarters in the United States. It also pays taxes to the United States. Can Ford still be considered a Transnational Corporation?

Ford can be considered a Transnational Corporation if is does business or produces goods outside the United States, which it does. Transnational Corporations are those which do business across multiple countries.

If Freddie has a strong attraction to Hannah Montana, how might he use the Reaction Formation Defense Mechanism to deal with this?

Freddie might actively voice disgust with her and say that he hates her music. In Reaction Formation, an individual expresses the opposite emotion to the one they actually have.

Differentiate between free recall, cued recall and recognition.

Free recall is when you recall information without any cues or hints. Cued recall is when you are given some form of a cue to recall a piece of information, e.g. when recalling the word 'planet' from a list and you are given 'pl_____' Recognition is the process of merely identifying a piece of info that was previously learned - far easier than recall itself.

Sigmund Freud's Dream theory: manifest content and latent content

Freud said dreams represent our unconscious wishes, urges, feelings and conflicts. He separated dreams into their manifest content (what one actually sees) and their latent content (the underlying significance/hidden meaning behind these elements). Iceberg analogy - what you see vs. what is underlying.

Many sociological theories converge on key ideas, but there is no one unified sociological perspective. Describe the theories of functionalism, conflict theory, symbolic interactionism

Functionalism - states society is always trying to come to an equilibrium - for society to be stable and function smoothly, it's parts (businesses, institutions, etc) must function and work together in harmony. Conflict theory - focuses on how societies change and adapt through conflict and power, which polarizes the society. Class struggle. Social constructionism - focuses on how people form and put together social constructs and realities - constructs arise from communication and agreement on significance of certain concepts - can be intangible (honor, justice) or even physical objects (money - paper and coinage money have value because we give it value). Symbolic interactionism - concerns interactions of race, ethnicity, immigration - individuals interact through shared understanding of words, gestures, *symbols* (to which we assign meaning).

What causes the decrease in the Mortality Rate? What causes the decrease in the Birth Rate?

Mortality Rate decreases due to higher availability of food and decrease in disease due to better hygiene and healthcare possibilities. Birth Rate decreases due to expanding ability for birth control/contraceptives and due to a higher percentage of women in the workforce.

Emile Durkheim described the theory of Functionalism - elaborate more on this theory and what it entails.

Functionalism is the thought that society is working towards an equilibrium, and for it to work smoothly it's compartments (institutions, individuals, etc) must function together in harmony. Theorists after Durkheim described *functions* as beneficial consequences of people's actions, necessary to keep society in balance. In contrast, *dysfunctions* referred to harmful consequences of people's actions that undermine society's equilibrium.

Compare and Contrast the function of G-Protein Coupled receptors and Ion Channels in terms of the Gustatory System?

GPCR: A sweet, umami or bitter molecule binds a GPCR which causes a conformational change. The G-protein then dissociates which opens an ion channel and thus causes the cell to depolarize and fire an Action Potential. Ion Channel: A sour or salty molecule binds to a receptor on an Ion channel which opens causing the cell to depolarize and fire an Action Potential They end up doing the same thing but the initial trigger is different.

Theories of multiple intelligences also have support. What was Howard Gardner's theory of multiple intelligences? How about psychologist Robert Sternberg's Triarchic theory?

Gardner's multiple intelligences - 7 defined types of intelligence: linguistic, logical-mathematical, musical, visual-spatial, bodily-kinesthetic, interpersonal and intrapersonal - all independent of each other. Sternberg's 3 intelligences: Analytical (IQ test relevant), creative and practical intelligence (to solve ill-defined problems).

Conflict Perspective believes that the Mass Media enhances divisions within society, particularly through Gate Keeping. How might this be so?

Gate Keeping refers to the idea that certain individuals in charge will determine what media gets aired and which media doesn't. It might reject media that portrays blacks as having equal rights to whites, which would reinforce inequality.

Let's say that your mom will sometimes text you before giving you a gift card. How might Generalization vs. Discrimination play out in this scenario?

Generalization would mean that you feel happy when your mom texts you as well as calls you whereas Discrimination would mean that you discern this to be different than a phone call and don't feel the same happiness (conditioned response).

The adrenal glands sit on top of the kidneys and are divided into the adrenal cortex and medulla. The adrenal cortex secretes corticosteroids - three functional classes, what are they?

Glucocorticoids Mineralocorticoids Cortical sex hormones

Describe glucocorticoids and how they work + e.gs

Glucocorticoids regulate blood glucose levels and raise them via gluconeogenesis and decreasing protein synthesis. Examples are cortisol and cortisone - cortisol is responsible for increasing glucose levels for fuel during stress and fight-or-flight modes. Glucocorticoid release is under control of ACTH

Group interactions can shape an outcome. What is groupthink?

Groupthink is the social phenomenon where the desire for harmony and minimizing conflict results in a group of people potentially coming to a poor decision. The desire to eliminate conflict drives consensus decisions without presenting of alternative views/ideas, and the desire to agree causes a loss of independent critical thinking. Many factors influence groupthink, including: illusion of invulnerability, collective rationalization, illusion of morality, pressure of conformity, self censorship, illusion of unanimity, mindguards

Habituation and sensitization (dehabituation) are forms of non-associative learning - what do these two entail in terms of response to a stimulus?

Habituation - reduced response to continuous stimuli Sensitization - increased response to continuous stimuli

Compare and contrast hueristics and algorithms in terms of processing speed and guarantee of correctness.

Heuristics are mental tricks used to break complex problems into simpler problems using mental tricks, but do not guarantee right answers. Algorithms are methodical approaches that are guaranteed to get the correct answer, but can take a very long time.

Describe hypnosis

Hypnosis is an induced state of consciousness, where a person appears to be in control of their actions but is in a highly suggestible state.

Describe briefly the functions of the parts of the endocrine system: - Hypothalamus - Pituitary gland - Thyroid - Parathyroid - Adrenal glands - Gonads - Pancreas

Hypothalamus - control center (ADH and oxytocin) Pituitary gland - master gland Thyroid - metabolism (T3 and T4) Parathyroid - calcium levels (PTH) Adrenal glands - - cortex (cortisol and aldosterone) - medulla (catecholamines) Gonads - sex hormones (test, estrogen) Pancreas - blood sugar (insulin, glucagon)

Put the Auditory Ossicles in order from first to vibrate to last to vibrate? I. Incus II. Malleus III. Stapes (A) I > II > III (B) II > I > III (C) III > II > I (D) III > I > II

II > I > III malleus -> incus -> stapes

You go to Japan and a man bows to you. How might you view this gesture if you have a mindset based in Cultural Relativism? Ethnocentrism?

If you have a mindset based in Cultural Relativism, you might think, "How nice. What a great way to show respect. I might not do that, but I can at least appreciate it." You are seeing Japanese culture from the Japanese perspective. If you have a mindset based in Ethnocentrism, however, you might think, "How strange. What a weird place this is." You are seeing Japanese culture from your own perspective/lens, which you see as superior.

Come up with your own example of Negative Punishment.

In order to have encourage Sarah to do her homework right after school, she is not allowed to go play with her friends until her HW is complete.

Describe prejudice in terms of race and ethnicity, as well as power, social class and prestige.

In terms of race and ethnicity, prejudice can form from dissimilarities amongst in-groups and out-groups of different racial or ethnic backgrounds due to stereotypical factors. In terms of power, groups with larger numbers or majorities have more political power to sway votes their way and keep candidates they like in office to protect their interests. This in turn influences economic and personal factors. In terms of social class, the ideal of the *just world hypothesis* (good things happen to good people, bad things to bad people) comes into play, where people often justify social class inequality by such. In terms of prestige - high prestige jobs like doctors and lawyers are more likely to go to majority group members, while low wage and prestige jobs are more likely to go to minority groups.

Differentiate between inattentional blindness and change blindness.

Inattentional blindness: ere not consciously aware of things that happen in our visual field when our attention is directed elsewhere in that field. "miss something right in front of you" Change blindness: when we fail to notice changes in the environment - "you miss a change between a previous state and current state" - e.g. moms new haircut The difference is subtle but important

Differentiate how the different processing routes of the elaboration likelihood model influences the types of characteristics of persuasion.

Information by this model is first filtered by our target characteristics, and then the content of the message/source characteristics are taken into account. Those in line with the central processing route - target characteristics (of listener) would likely be greater interest in topic, motivation to learn, and finding it important. In terms of message and source characteristics, the information is deeply processed. Those in line with the peripheral processing route - target characteristics might likely include lack of interest in or motivation to learn about the topic information, as well as little importance. In terms of message and source characteristics, the superficial factors like attractiveness have influence on persuasion.

What is the sensory strip?

It is a specific part of your brain's cortex that contains the somatosensory homonculus, which is the map of your own body. It receives sensory input from the entire body (any pain, heat, fingers touching, feet etc). Information from any part of the body will go to one specific part of the sensory strip. There is a designated synapse area for your feet, your face, fingers, etc.

Jack says that he is a Christian, but he doesn't go to church very often and doesn't really follow the teachings of his religion. He thinks of himself as spiritual, however, as he engages in meditation, yoga, and even tai chi. How would you describe the Religiosity of Jack?

Jack has a low amount of Religiosity because religion plays a small role in his life and he is not very engaged with his religion. His spirituality is unrelated to his engagement with religion.

The most common form of depression when referred to is Major Depressive Disorder. What is this disorder characterized by?

Major Depressive Disorder - usually characterized by a feeling of helplessness and worthlessness, low self esteem, persistent depressive mood, social isolation and discouragement/loss of interest in most enjoyable activities (anhedonia) Other physical symptoms such as appetite disturbances, weight fluctuations, sleep disturbances, decreased energy (lethargy) or excessive guilt can also be seen.

Maslow's Hierarchy of Needs is the most well known motivation theory based on needs, and breaks down needs into categories on a pyramid. Describe his theory and hierarchy.

Maslow's Hierarchy of Needs - classifies needs into five groups of different importance on a pyramid, with the bottom being the most important. His needs, in order of importance, were: - Physiological (breathing, food, water, air, sex, homeostasis) - Safety (security of: body, employment, health, property) - Love/Belonging (friendship, family, intimacy) - Esteem (self-esteem, confidence, respect) - Self-actualization (morality, creativity, spontaneity, problem-solving, accomplishment)

Give a few examples of Mass Media?

Mass Media includes things such as books, newspapers, the internet, television, movies, music, and magazines.

Match each of the following to either (A) Collective Behavior or (B) Group Behavior: (1) Long-term (2) Short-term (3) Open-access (4) Exclusive (membership requirements) (5) Strongly Defined Norms (6) Loose Norms

Match each of the following to either (A) Collective Behavior or (B) Group Behavior: (1) Long-term -> B (2) Short-term -> A (3) Open-access -> A (4) Exclusive (membership requirements) -> B (5) Strongly Defined Norms -> B (6) Loose Norms -> A

What might be the function for the Rooting reflex?

May be beneficial for breast feeding

How can McDonalds be considered a part of American Culture?

McDonalds comes with its own objects and beliefs, which contribute to the overall culture of our society. For instance, McDonalds is all about convenience, which has become a value in our society. Culture consists of the beliefs, behaviors, objects, and other characteristics common to the members of a particular group or society.

Sexual and gender minorities are represented under the grouping of the *LGBT community* - what are some health disparities recognized in the LGBT communities?

Most significant LGBT health disparity is seen with HIV/AIDs - disproportionately higher for gay men in urban areas. Lesbians in the healthcare community receive less screening for cervical cancers and may not be screened for other STDs Transgender people have multiple areas of increased risk, including use of "street hormones" without proper guidance and counselling.

DSM-5 Classification.

Neurodevelopmental disorders - distress caused by lack of NS development Neurocognitive disorders - lost function of the NS causing distress Sleep-wake disorders - distress/disability from sleep abnormalities (sleep apnea, insomnia) Anxiety disorders - some specific to phobias, but can be general too - fear and anxiety related (e.g. panic attacks) Depressive disorders - distress from abnormally negative mood Bipolar and related disorders - distress cause by both depressive moods and mania Schizophrenia spectrum + other - distress from psychosis Trauma and stressor related - distress from mentally traumatic or stressful events (e.g. PTSD) Substance-related/addictive disorders - distress from abnormal substance use leading to mental function disorders Personality disorders - distress related to personality (long term mental and behavioural features) Obsessive-compulsive disorders - distress related to obsessive and compulsive behaviour (OCD) Somatic symptom and related disorder - distress from psychologically derived symptoms from illnesses Feeding and eating disorders - distress from food related disorders (anorexia, bulimia) Elimination disorder - distress from urinating or defecating at inappropriate times Dissociative disorders - abnormalities from memory or identity Sexual dysfunction - distress from abnormalities relating to sex performance Gender dysphoria - distress from identifying as a different gender than that identified by society Paraphilic disorders - distress or disability fro sexual arousal from what is considered unusual stimuli Other disorders - one who appears to have a mental disorder causing distress/disability but doesn't fit into another category

By what mechanism does MDMA (ecstasy) function as both a stimulant and hallucinogen?

On the stimulant side, ecstasy increases dopamine and serotonin levels, leading to increased heart rate, CNS activity, blood pressure, dehydration. Overdoing it can lead to damaged serotonin receptors. On the hallucinogen side, excess ecstasy can cause perceptual changes including hallucinations and a feeling of connectedness.

When do on-center bipolar cells turn on? What about off-center bipolar cells?

On- and off-center refer to to the status of a bipolar cell being either ON or OFF. For an on-center bipolar cell, it is turned ON when light shines on the center of the rods (rods turn off). Off-center bipolar cells refer to bipolar cells being turned ON in the absence of light on the center of rods (rod cells turned on). See image for more details.

What differentiates the effects of opiates/opioids vs. depressants?

Opiates can also work as analgesics. Also, the two classes work via different mechanisms: Opiates - stimulate endorphin receptors. Depressants - enhance GABA receptors.

Freddy, as we know, belongs to the town rotary club. It is a very democratic organization and decisions are made jointly between all members. According to the Iron Law of Oligarchy, what will inevitably happen to this structure over time?

Over time, the current members will develop close relationships with one another. These members due to their relationships and previous knowledge and history with the rotary club will have an advantage over new members; thus their voices may begin to matter more. The members that continue with the club may formally claim a higher status than newer members, making it so their power is more permanent in nature. Those with close ties to this inner circle will likely maintain power over time, making this organization more of an oligarchy than a democracy as it once was. This is what happens according to the Iron Rule of Oligarchy, which says that all organizations, despite their origins will eventually become Oligarchies.

Describe peptide hormones

Peptide hormones are made up of amino acids - derived from larger precursor polypeptides that undergo Golgi modification

Mitch agrees to smoke a cigarette when offered it by his friend. He doesn't like the taste of it, and never wants to do it again. His Mom saw him do it and punishes him severely for it. Mitch starts to see himself as a smoker and decides to continue smoking with his friend. How do the terms Primary and Secondary Deviance apply to Mitch's situation?

Primary Deviance is the intial act of deviance and is exemplified by Mitch smoking his first cigarette. Secondary Deviance occurs as Mitch incorporates the smoker identity as his own and continues acts of deviance in the long term.

What is priming?

Priming is the effect where exposure to one stimulus influences response to another stimlus.

Jimmy agrees to eat 10 hot chili peppers with his friends. How would Private Conformity play out in this situation? What about Public Conformity?

Private Conformity would entail Jimmy agreeing inwardly and outwardly that eating 10 hot chili peppers is a good idea. Public Conformity would entail Jimmy eating the 10 hot chili peppers and agreeing that it is a good idea outwardly while feeling it is a bad idea inwardly. A private conformity will actually change your internal beliefs, whereas public would only make you put on a public persona.

What are the main differences between proprioception and kinesthesia?

Proprioception can be thought of as a cognitive awareness of your body in space and is more subconscious Kinesthesia can be thought of as behavioural, as your brain can consciously detect a movement and improve at that movement to better complete the task at hand.

Proprioception spindles are receptors in the muscles, often compared to springs. How do these receptors communicate position to your brain?

Proprioception spindles can sense when the muscle contracts and stretches, and stretches itself. When the protein within gets stretched, it fires a signal to the brain. The ability to know when each muscle in our body is contracted or relaxed gives us our sense of position in space, hence our proprioception.

Compare and contrast the Psychoanalytic Theory and the Humanistic Theory?

Psychoanalytic Theory thinks of us as being controlled by our unconscious. Sees us as animals that have intense desires. Humanistic Theory, however, views us as being in control and focuses on our conscious thoughts and actions. Sees us as inherently good and wanting to improve.

Compare and contrast puberty and adolescence.

Puberty is a 2-year long period for sexual maturation and drastic body changes that ends with the ability of sexual reproduction. Adolescence is a transition between childhood and adulthood seen as starting with puberty and ending once someone is an independent adult (self-supported).

Consequences in operant conditioning have reinforcements and punishments. Describe punishment and differentiate between positive and negative punishments.

Punishment uses conditioning to reduce occurrence of a behaviour. *Positive punishment* adds an unpleasant consequence in response to an undesired behaviour (e.g. chop off hands for stealing in Saudi Arabia) *Negative punishment* reduces frequency of a behaviour when a stimulus is removed (e.g. parent punishes kid for bad behaviour by banning TV)

Consequences in operant conditioning have reinforcements and punishments. Describe reinforcement and differentiate between positive and negative reinforcements.

Reinforcement is the process of increasing the likelihood of a certain behaviour. *positive reinforcers* increase behaviour by providing a _positive consequence/incentive_ following desired behaviour (e.g. money) *negative reinforcers* increase frequency of a desired behaviour but _by removing something unpleasant_ (e.g. increased likelihood of taking aspirin next time you have a headache).

What happens when light strikes a rod cell in the rhodopsin proteins? How are they turned off?

Rod cells have disc like structures within, that contain rhodopsin proteins. Rhodopsin contains a small molecule retinal, which exists in a particular conformation called 11-cis retinal. When light hits rhodopsin, it causes the 11-cis retinal molecule to change conformation to become all trans-retinal, and subsequently rhodopsin also changes shape. Rhodopsin has a molecule called transducin, (a G-protein which has α, β and γ subunits) that then breaks off, and the alpha subunit goes and binds to a protein phosphodiesterase (PDE) on another part of the disc. PDE converts cyclic GMP to GMP. The increase in [GMP] results in inability of Na+ channels along the cell to remain open, as they require cGMP to be bound to remain active. As less sodium enters the cell, the cell hyperpolarizes and causes the rod cells to turn off. These will be divided into other cards - look out for them!

Describe rote rehearsal

Rote rehearsal (or maintenance rehearsal) is the repitition of a piece of info to either keep it within working memory (and not forget) or store it in short-term and eventually long-term memory. This rehearsal is generally not effective in facilitating later recall

Samantha's parents don't let her do things on her own, leading her to lose trust in her own abilities. How old is Samantha? (A) 8 months (B) 2 years (C) 7 years (D) 13 years

Samantha is struggling with Autonomy vs. Doubt, which occurs between the ages of 1 and 2.

Jacob says he has reached self-actualization after winning an Olympic Gold medal in the high jump. He used steroids to achieve this feat and was really only doing it for the fame and fortune. Would Maslow agree with Jacob?

Self actualization revolves around reaching one's maximum potential. If he was really doing it for the fame and fortune only, and not self-improvement, think about it. Self-actualizers are those who reach their full potential and these individuals are true to their own moral principles. Jacob is not focused on reaching his full potential or on obeying his own moral principles. He is most likely trying to meet needs of self-esteem, and is not actually self-actualizing.

Carl Rogers had a humanistic approach to studying the self and stated the self-concept had three components: describe self-image, self esteem and the ideal self

Self-image - the view we have of ourselves Self-esteem - (self worth) - how much value we place on ourselves Ideal self - what we wish or aspire to be

Describe self-serving bias

Self-serving bias is the notion that we ourselves couldn't commit atrocious acts or crimes applying to certain situations, such as the Milgram experiment or the holocaust.

What are the two important classes of steroids in the body?

Sex hormones - estrogen, progesterone, androgens Adrenal cortex steroids - cortisol, cortisone, aldosterone

Why do we dream? No one knows this answer, but what are some of the theories?

Sigmund Freuds theory- dreams are our unconscious desires and needs that we want to fulfill Evolutionary theory- some evolutionists believe dreams evolved as a ways to develop either problem solving and/or threat stimulation Other theories- can include to develop neural pathways, maintain and consolidate memories

Mwape just barely arrives at a new school. She meets two new friends, Becky and Sarah. Mwape starts inviting them over to her house and leading them in several of her favorite games. Does Mwape demonstrate Social Potency or Traditionalism. What about Sarah and Becky? Explain why?

Social Potency is the degree to which a person assumes leadership and mastery roles in a social situation. Mwape is demonstrating this by quickly taking control in her new friend group. Sarah and Becky likely demonstrate Traditionalism as they are quick to follow Mwape's lead.

What does Socialization mean? Describe how attending school might affect the Socialization of a child.

Socialization is the process by which we as individuals acquire a personal identity and learn the norms, values, behavior and social skills appropriate in a certain society. At school, the child will learn from their teachers and peers that it isn't okay to make weird noises with your tongue. They may learn from their teacher that it is isn't okay to speak with someone else is talking. They will also learn what is valued in society by what is seen as "cool" by their peers at school. For this reason, school is an example of an Agent of Socialization as it helps socialize individuals.

How does Culture relate to Society?

Society is the structure. It is a group of people that share a culture. It is like a canvas. Culture consists of what the people of a society value, their believes, objects, and behaviors. It is like the paint on a canvas. Each society is unique in that it has a different culture just as each canvas is unique in that it has a different painting on it.

Compare Somatic Symptom Disorder and Conversion Disorder.

Somatic Symptom disorder can have any type of symptom, specific or general, can have extreme levels of anxiety over these symptoms, and may or may not be medically explained. Conversion Disorder, on the other hand, is characterized by neurological symptoms, like speech, swallowing, seizures, or paralysis. They are incompatible/undetectable with any test/exam.

What are ways religious groups might deal with the struggle to find a place in contemporary society as times change and modernize?

Some religious groups may shift towards modernization within the religion and relax historical practices Some may shift away from religion as society *secularizes* and become more dominant in scientific thinking Some may shift more towards *fundamentalism*, strict adherence to religious code predominating (e.g. Zia's Pakistan)

How do the Parvo Pathway and Magno Pathway compare in terms of: -Spatial Resolution (Detail)? -Temporal Resolution (time)? -Color Vision?

Spatial Resolution - The Parvo pathway can provide much greater detail, necessary for detecting form. Temporal Resolution - The Magno Pathway has faster processing, necessary for tracking motion. Color Vision - Only found in the Parvo Pathway.

Deutsch and Deutsch's Late selection theory

Spawned from the cocktail party effect. This shifted the selective filter aspect to after the perceptual meaning.

Describe symbolic interactionism and how it explains society

Symbolic interactionism looks at a small scale of society and explains it through individuals and their interactions + signing of meanings to decide how to act - through these it can explain social order and change. George Herbert Mead compiled the theory - believed development of individual was a social process and people change through interactions. Herbert Blumer added to it and coined the term symbolic interactionism - 3 tenets: (1) we act on the meaning we give something; (2) we give meaning to things based on social interactions; (3) meaning we give something is not permanent

True or false; diffusion is the spread of cultural norms, ideas, structures etc. across the globe

TRUE; diffusion is the spread of cultural norms, ideas, structures etc. across the globe Both things like capitalism and Mcdonalds are examples of diffusion!

Give an example of how framing a decision can change the choice a person will make.

Take the flu vaccine as an example. If I ask someone if they want to be protected from the flu, they would be likely to get vaccinated; if I ask if they want to receive a shot, they are less likely to get vaccinated. Framing makes a big difference in how someone views their choices!

Kohlberg organized moral reasoning based on participants' responses to hypothetical moral dilemmas into 6 stages ranging from concrete to the abstract. Describe the preconventional phase in terms of the [[Heinz dilemma (stealing an expensive drug to save his wife)]]

The *Preconventional morality* phase is typical of preadolescent thinking and emphasizes consequences of moral choices. Consists of the first two stages. Stage 1 (*obedience*) is concerned with avoiding punishment (/steal the drug, go to jail/)' Stage 2 (*self-interest*) is focused on gaining rewards (/i need to save my wife/), and is often called the *instrumental relativist stage*

Describe the Schachter-Singer theory of emotion. (hint: cognitive appraisal)

The *Schachter-Singer theory of emotion (aka cognitive-arousal theory or two-factor theory)* states that BOTH arousal and the labelling of arousal based on the environment must occur in order for an emotion to be experienced. E.g. "I am excited because my heart is racing and everyone else is happy" This theory uniquely incorporates cognitive appraisal into causation of emotion

What is the social construction of illness?

The *Social construction of illness* refers to how social institutions and ideas of overall societal understanding of the world is forged through a collective process that imbibes that understanding with values of the people within that society. E.g. in America, children eventually move out after getting jobs and become independent of families, whereas in China or elsewhere in Asia children are expected to take care of their family - social constructionist would describe Eastern understanding of family is socially constructed and imbibed with values of a collectivist culture, whereas America would be with values of an individualistic culture.

What does the activation-synthesis theory of dreams state?

The *activation-synthesis theory of dreams* states that dreams are caused by widespread, random activation of neural circuitry in our brainstem - this activation can mimic incoming sensory information, and may consist of stored pieces of memories, current or previous desires, and met or unmet needs. By this theory, dreams have no prominent meaning.

Explain how each of these biological components influence aggression: the amygdala, the prefrontal cortex, and hormones

The *amygdala* is responsible for associating stimuli with corresponding rewards/punishments. In essence it tells us whether or not something is a threat. An activated amygdala increases aggression. Higher order brain structures like the *prefrontal cortex of the frontal lobe* can place brakes on activated amygdala - reducing emotional reactivity and impulsiveness - controls the aggression. Reduced activity of the prefrontal cortex and frontal lobe linked to increased aggressive behaviour. Hormonal control influences aggression too - *high levels of test = more aggressiveness*. Explains why men might be more aggressive than women in general.

According to Freud, what characterizes the anal stage of his theory?

The *anal stage* (1-3yo) - libido is centered on the anus and gratification is gained through elimination and retention of waste. This is the stage where toilet training occurs, and fixation as this stage would lead to either excessive orderliness (anal-retentiveness) or sloppiness in the adult.

According to Freud, what characterizes the genital stage of his theory?

The *genital stage* is characterized by libidinal energy again focused on the genitilia - lasts from puberty through adulthood. According to Freud, if prior development proceeds correctly, the person should have healthy heterosexual relations. However, unresolved earlier stage and fixation in developments can result in homosexuality, asexuality and fetishism.

What is the life coarse approach to health?

The *life coarse approach to health* is exactly what it sounds like - to view health in terms of a patient's entire life history. It takes many factors into consideration instead of just those directly incidental to the illness at hand (thus, a holistic approach).

According to Freud, what characterizes the oral stage of his theory?

The *oral stage* (0-1yo) is the stage where gratification is obtained by putting things in mouth - biting, sucking reflexes; libidinal energy is focused on the mouth, and an orally fixated adult would likely exhibit excessive dependency or aggression (e.g. people that might do nail biting, thumb sucking, cigarette smoking, over-eating)

Why is Cognitive Theory seen as a bridge between Behaviorism and Psychoanalysis?

The *social cognitive* perspective focuses not only on how the environment influences behaviour, but also how we interact with the environment.

John knows that his family has a history of depression, and he has the disease version of the 5-HTTLPR gene (associated with depression in some cases). Knowing this, John avoids long-term stressors when possible and never develops depression. How can this be explained?

The 5-HTTLPR gene is only associated with depression if the person is in stressful situations often.

Contrast The Behavioral Theory and the Psychoanalytic Theory.

The Behavioral Theory says that we are influenced by our environment and is focused on observable behavior. Psychoanalytic Theory, on the other hand, is focused on internal desires and mental states.

What makes the Biologic Theory unique as compared to the Humanistic and Psychoanalytic Theories?

The Biologic Theory posits that our behavior and personality are inherited whereas the Humanistic and Psychoanalytical Theory propose that they develop from our experiences. Certain traits and genes are coupled with certain behaviours, however not necessarily defining.

Some theorists are skeptical of the Authoritarian Personality connection with Prejudice. The Frustration-aggression Hypothesis is an alternative theory for what leads people to exhibit Prejudice. How does this theory explain why one person might be more prejudice than another?

The Frustration-aggression Hypothesis suggests that as someone becomes frustrated they become more aggressive, which in turn will lead them to take out their aggression on easy prey like minority groups, who they may use as Scapegoats and easy targets.

Compare the Halo Effect and the Devil Effect.

The Halo Effect occurs when you have an overall good impression of someone and it skews your evaluation of them toward the positive end of the spectrum. The Devil Effect occurs when you have an overall bad impression of someone and it skews your evaluation of them toward the negative end of the spectrum.

A teacher gives the class the following practice problem, "Jim takes three apples from Mary, which makes her angry, so she throws 2 apples at Jim's face. This makes Jim very sad. Eventually they apologize to each other and each eat one of Mary's apples together. If Mary originally had 13 apples, how many apples does she have now?" How does this relate to the idea of the Hidden Curriculum of Education?

The Hidden Curriculum of Education is the idea that the Education System is more than about just learning information. It is there to Socialize us and teach us things about the world such as manners, morals, and how to make friends. This practice problem doesn't just teach math but it has a hidden curriculum of teaching kids about being kind and forgiving. By the way, Mary now has 6 apples left

Why is the hypothalamus referred to as the "control center"?

The Hypothalamus plays a key role in integrating the CNS and Endocrine System because of its duality as an endocrine gland and as part of the forebrain. Because of this, the hypothalamus can be considered the Control Center.

You are really craving ice cream. Describe how the Id, Ego, and Superego might interact within your mind as you face this temptation.

The Id wants immediate gratification and would drive you to just eat the ice cream NOW. The Superego is moral oversight and represents the values of society, which would tell you to never eat ice cream because it is not good for your body. The Ego acts as a mediator would balance out the two, resulting in a compromise where you eat some ice cream after patiently waiting for it.

What is the Just World Phenomenon?

The Just World phenomenon is the idea that the universe is fair and people must get what they deserve. "Good things happen to good people, while bad things happen to bad people". In terms of Milgram's experiment, some of the participants who felt ashamed for what they did still justified their actions somewhat by sort of blaming the victims, stating if they got more answers correct and were smarter, they wouldn't have gotten shocked as much.

Compare the nervous system and endocrine system, focusing on speed and duration.

The Nervous System can send signals and cause change very quickly, but the duration of these effects tend to be short. The Endocrine System will take much longer to send signals across the body, but the duration of these signals tend to last much longer.

There are a few theories that attempt to explain how attitudes influence behaviour. Describe the Prototype-Willingness model (PWM)

The PWM says that behaviour is a function of six factors - past behaviour, attitudes towards behaviour, subjective norms, intentions, willingness to engage in a behaviour, and prototypes (models) This theory combines the planned behaviour and attitude-behaviour theories and adds onto it.

The Fertility Rate is 2.9. Will the population of this community increase, decrease, or stay relatively the same?

The Population will increase as the Fertility rate is above 2. At exactly 2, the current population will almost exactly be replacing itself. The value of 2 would mean the two children born to the average woman would replace the two biological parents.

Abnormalities in neural pathways for certain NTs may cause increased or decreased activity in certain parts of the brain, leading to depressive symptoms. Discuss these pathways: - Raphe system - Locus Coeruleus - VTA pathway

The Raphe system - pathway begins in the Raphe nuclei of the brainstem, responsible for secreting serotonin into projections leading up to the cerebrum (including frontal lobe and limbic system) - the pathway seems to be abnormal in patients with depression. Locus Coeruleus - also begins in the brainstem, releasing norepinephrine into the cerebrum - also abnormal in patients with depressive disorders VTA - in the brainstem - supplies much of the dopamine to the brain via the mesocorticolymbic pathway - also abnormal in depressive patients. Medications that affect these different NTs often better some of the symptoms of depressive disorders.

As stated before, Sam works at the only lemonade stand in the town. After talking to his buddy from a neighboring town, he realizes that he and his fellow citizens are being ripped off, so he incites his fellow citizens to demand change. What is the Thesis in Sam's town? What about the Antithesis? Why might be the Synthesis?

The Thesis is the idea that the lemonade stand owner (the Bourgeoisie) is in charge and can set whatever wages he wants for his workers. The Antithesis is the idea that the lemonade stand workers (the Proletariat workers) have to overthrow the lemonade stand owner. Because the Thesis and the Antithesis cannot coexist, they will eventually result in a Synthesis. Perhaps in this case, the lemonade stand workers form a union, which results in a legal right to higher wages.

Describe emotional intelligence

The ability to perceive, understand, manage and use emotions in your interactions with others

What is the major function of the basal ganglia (which contains the substantia nigra)?

The basal ganglia coordinates motor movements. It receives dopamine from the substantia nigra to function properly - it controls initiation and termination of movements, as well as sustaining repetitive motor skills and smoothening muscles.

What is the bystander effect?

The bystander effect refers to the inverse relationship between number of people in a group and the inclination for any individual to act during an event which requires action. The more sizeable the group is, the less inclined anyone feels to take action.

Immigrant population in the US is rapidly increasing. What are some barriers that can affect interactions with social structures and institutions?

The complex organization of the US healthcare system and language barriers can both serve as barriers to maximum access to healthcare and making healthcare decisions.

Nativists believe in a certain time frame between the ages of 2 and puberty for learning language, beyond which later training is largely ineffective. What is this timeframe called?

The critical period - period of time in which a child is most able to learn a language. Theoretically, LAD only operates during this period.

Describe the fourth step of the phototransduction cascade. We now have high [GMP] - so what?

The decline in [cGMP] is crucial because sodium channels throughout the rod cells require cGMP to be bound in order to remain open. Now, with them closed, less sodium enters the cell, causing the cell to hyperpolarize and subsequently turn off.

The bystander effect is explained by the Diffusion of Responsibility theory - describe this

The diffusion of responsibility theory - when individuals are in the presence of others, they feel less personal responsibility and are less likely to take action when in a situation where help is needed. Thus, the inverse relationship between # of people and inclination to act, as seen in the bystander effect.

There are a few theories that attempt to explain how attitudes influence behaviour. Describe the elaboration-likelihood model for persuasion

The elaboration-likelihood model theory separates individuals based on their processing of persuasive info - is more cognitive and focuses on the 'why' and 'how' aspects. There are two ways this model is processed: 1) At one extreme, those who elaborate extensively and think deeply - *central processing route* - the degree of attitude change depends on the quality of arguments by the persuader 2) Other extreme - those who do not elaborate much - *peripheral processing route* - looks at superficial and nonverbal persuasion cues (attractiveness, status, expertise)

There are a few theories that attempt to explain how attitudes influence behaviour. Describe the functional attitudes theory

The functionalist attitude theory states that attitudes serve four functions: knowledge, ego expression, adaptation and ego defence. *Knowledge* provides consistency and stability in thought. Attitudes can be *ego expressive* (to communicate and solifidy our identity; e.g. big Eminem fan, big Pats fan) and *ego defensive* (protect self-esteem or justify actions we know are wrong). They can also be *adaptive* - idea that one will be accepted if attitudes are socially acceptable.

The Functionalist Perspective thinks of Mass Media as an enforcer of Social Norms. How might this be so?

The functionalist perspective looks at mass media as an agent of socialization through the enforcement of norms. It also functions to bring people together (e.g. reddit, LGBT groups). Mass Media will also glorify "good" behavior such as winning sport's competitions.

When you eat a piece of cake, how do the glucose molecules interact with taste buds?

The glucose molecule binds to a receptor for glucose in the sweet cell of a taste bud, which triggers a cascade and fires an action potential all the way to the brain. Also: for sweet tastes, the molecule binds to a GCPR and causes the G-protein to dissociate from the receptor, beginning the cascade.

What is the role of the hypothalamus in the limbic system?

The hypothalamus regulates the autonomic nervous system - fight/flight or rest/digest - via secreting hormones into your body (epinephrine, norepinephrine, adrenaline)

How does a cochlear implant work?

The implant is a Receiver and has a tube called a Stimulator that goes all the way and connects to the cochlea. The receiver gets information from its adjacent structure called a Transmitter, which gets electrical information from the Speech Processor. The Speech Processor gets information from it's microphone, which goes to the Transmitter -> Receiver, which sends the information through the Stimulator into the cochlea, which converts that electrical impulse into a neural impulse. This restores some degree of hearing to individuals with nerve deafness

When talking about biases, what is belief perseverance?

The inability to reject a particular belief despite clear evidence to the contrary

Describe the incentive theory of motivation

The incentive theory explains behaviour as motivated by desire to pursue rewards + avoid punishments, rather than by arousal. It is based on the idea that if a behavior is rewarded, it will result in the behavior occurring again.

A smoker who has been smoking for 6 years is aware of how damaging it is to his body, yet he cannot seem to stop smoking. In fact, he claims he smokes more cigarettes today compared to two years ago, when he tried to start quitting due to harm realization. How does the opponent-processing theory explain these phenomenon?

The opponent-processing theory explains that when a drug is repeatedly taken, the body attempts to counteract effects by changing physiology. It counteracts alcohol abuse, for example, by increasing arousal. The problem arises because this reaction lasts longer than the drug - results in withdrawal symptoms which are exact opposing effects of the alcohol or drug. This can create a physical dependence. The same phenomenon runs for nicotine, in cigs - it explains why the smoker smokes more today as he has an increased tolerance.

What is the optimal arousal theory of motivation?

The optimal arousal theory states people perform actions in order to maintain an optimal level of arousal - seek to increase arousal when it falls below their optimal level. The image shows the Yerkes-Dodson law which postulates a U shaped function between arousal and performance - performance is worst at extremely high and low levels of arousal, and best when at optimal levels.

How are sound waves transmitted along in the inner ear?

The oval window (or eliptical window) is attached to the cochlea. After the ear drum vibrates and the stapes causes the oval window to vibrate. This causes fluid inside to go all the way around the cochlea until reaching the tip, and flow back until it hits the circular (round) window and pushes it out. This continues to happen until the energy of the sound wave has dissipated, and hair cells inside the cochlea are being pushed back and forth, which transmits an electrical impulse via the auditory nerve --> brain.

What does the pre-frontal cortex do in relations to emotions?

The prefrontal cortex is the anterior portion of the frontal lobes - associated with planning intricate cognitive functions - problem solving, decision making, expressing personality.

Explain the concept and mechanism of synaptic plasticity and long-term potentiation.

The presynaptic neuron releases NTs to the postsynaptic neuron, which opens channels to allow cations (Na+, Ca^2+) to flow into the postsynaptic charge. Each neuron have a surrounding membrane, with the interior having a different charge than the exterior - diff in charges called neurons potential, and the flow of ions affects that potential. Strength of synapse is measured by how much post synaptic potential changes with presynaptic stimulus. *With practice, the synaptic plasticity improves at opening channels and the strength of synapse increase - this is called Long term potentiation, and this is why we can learn better with practice.*

Remaining in stress and overdrive for too long puts your body in a system of overdrive. What are some of the physical effects of stress on the reproductive system?

The reproductive system requires lots of energy, and so part of acute stress response (like fight or flight) the reproductive system is inhibited or shut down. However, with chronic stress (which accompanies psychosocial stressors) - hormones like LH and FSH, estrogen and progesterone involved in reproduction in women can be inhibited; erectile dysfunction is a problem in men with inhibited testosterone and less blood flow.

Describe the signalling pathway for the sympathetic NS

The signal pathway consists of two neurons - starts with a short preganglionic neuron, whose cell body originates at the spinal cord and the axon extends into the PNS. The preganglionic neuron synapses with a long postganglionic neuron using acetylcholine, which extends to synapse to the effector organ (using epinephrine/norepinephrine) Note: the synapse between pre- and postganglionic neurons uses acetylcholine while the second between the post- and effector organ uses epinephrine or norepinephrine.

There are a few theories that attempt to explain how attitudes influence behaviour. Describe the social-cognitive theory

The social-cognitive theory of attitudes states people learn how to behave and shape attitudes by observing behaviours of others. According to this - behaviour is not learned by trial and error, but develops via direct observation and replication of actions. 3 factors of behaviour, personal factors and environment are not independent, but rather influence each other.

Stereotypes occur when attitudes and impressions are based on limited and superficial information about a person/group - content of stereotypes are attributes that people believe define a group. Describe the *stereotype content model*

The stereotype content model classifies stereotypes with respect to a hypothetical in-group using two dimensions: warmth and competence. Warm groups are those not in direct competition with the in-group; competent groups are those with high status within society.

Clearly differentiate the terms stressor, stress reaction, and stress.

The stressor is the event that is causing the animal to have that stress reaction. The stress reaction is how the animal with the stressor physiologically responds. Stress is the combination of the stressor and stress reaction.

When talking about biases, what is confirmation bias?

The tendency to focus on info that fits an individuals beliefs while rejecting information that goes against them

A reflex is an involuntary behavioural response to a stimulus without higher cognitive input. Infants are equipped with a variety of reflexes that may help and ensure survival. Describe the following primitive reflexes that disappear with age: - Rooting reflex - Sucking and swallowing reflex - Moro reflex - Babinski reflex - Grasping reflex - Galant reflex - Swimming reflex

These primitive reflexes that disappear with age are called neonatal reflexes. Rooting reflex - baby turns its head in response to stroking of cheek (~first few weeks) Sucking and swallowing - when object placed in mouth (~3-4 months) Moro reflex - infant reacts to abrupt movements of the head by flinging out arms and crying (disappears ~ 4 months) Babinski reflex - curling of toes when something touches foot Grasping reflex - wrapping fingers around an object Galant reflex - when stroked on the side, baby will roll to that side Swimming reflex - infants move arms and legs in swimming fashion and hold breaths when placed in water (~6 months)

Electroencephalography (EEG) is used to record average electrical patterns of the brain - there are four characteristic patterns, or waves, associated with different states of waking or sleeping. What are theta and delta waves?

Theta waves are experienced in the drowsy state, essentially Stage 1 of sleep - low frequencies of 4-7Hz, high voltage, irregular waveforms. Delta waves are low frequency (0.5-3Hz), high voltage waves experienced in Stage 3 and 4 of sleep - usually REM sleep or coma.

What is the difference between Exurbs and Suburbs?

They are very similar except that Exurbs are farther away from the city as Suburbs and are often home to very rich individuals.

How might a feminist view the use of the term "policeman"?

They may see it as structural oppression, which puts women in a box that says they can't be a police officer.

Jack looks back on the Holocaust, and thinks to himself, "If I were in that situation, I may have done the same thing." Is this an example of the Self-serving Bias?

This is not an example of the Self-serving Bias. Jack would have said, "I would have never done something like that", asserting his actions or morals would be superior.

True or false: Rods are slow-adjusting to changes in light whereas cones are rapid-adjusting

True

True or false: prolonged beta waves can induce higher stress and/or anxiety and restlessness

True

True or false: in a hypnotized state, brain waves show alpha waves on an EEG

True. Alpha waves are seen showing an awake but relaxed state.

True or False? Babies develop a sense of their Existential Self within 12 to 21 days of birth.

True. Babies develop a sense of their Existential Self (idea that they are a separate, unique entity) within 12 to 21 days of birth. This is evidenced by studies on Imitation.

True or False? Because Nature and Nurture do not act independently, it is more correct to think of the two as "Nature through Nurture".

True. Because Nature and Nurture do not act independently, it is more correct to think of the two as "Nature through Nurture".

True or False? Children in the Pre-operational Stage are often egocentric, not allowing them to consider other's perspectives.

True. Children in the Pre-operational Stage are often egocentric, not allowing them to consider other's perspectives.

True or false: partial reinforcement schedules are generally more resistant to extinction than continuous reinforcement

True. Continuous reinforcement eventually becomes less reinforcing.

True or False? Culture can have an impact on development on different levels: a specific aspect of culture could both help and hinder different parts of development.

True. Culture can have an impact on development on different levels: a specific aspect of culture could both help and hinder different parts of development. An example of this is Doctors now recommending infants sleep on their backs, which delays learning to crawl (slight negative), but prevents babies suffering SIDS (positive). *

True or False? Depression is biologically-supported, and there is evidence that patients suffering from Depression stop making and have reduced responses to serotonin in the frontal cortex.

True. Depression is biologically-supported, and there is evidence that patients suffering from Depression stop making and have reduced responses to serotonin in the frontal cortex.

True or False? Ekman's 6 universal emotions are considered universal because they are expressed by consistent facial expressions, regardless of cultural background.

True. Ekman's 6 universal emotions are considered universal because they are expressed by consistent facial expressions, regardless of cultural background.

True or false: epinephrine and norepinephrine are water soluble, not lipid soluble, and amino-acid derivative hormones

True. Epinephrine and norepinephrine are water soluble, not lipid soluble, and amino-acid derivative hormones

True or false: genetic abnormalities and predispositions can be a cause to contribute towards depressive disorders, along with psychosocial factors

True. Genetic abnormalities and predispositions can be a cause to contribute towards depressive disorders, along with psychosocial factors. Nature and nurture can combine to cause the illness.

True or False? Gestalt Psychologists believe that you cannot understand human psychology by breaking it down into its parts. Rather it must be understood as a whole.

True. Gestalt Psychologists believe that you cannot understand human psychology by breaking it down into its parts. Rather it must be understood as a whole.

True or False? Hypnosis and Meditation are two examples of Induced States of Consciousness.

True. Hypnosis and Meditation are two examples of Induced States of Consciousness.

True or False? In the Asch experiment, when the participant was told to write their answers down, conformity of the participant decreased because they no longer had to state a dissonant answer out loud.

True. In the Asch experiment, when the participant was told to write their answers down, conformity of the participant decreased because they no longer had to state a dissonant answer out loud. When you know you are not being observed, your conformity will decrease.

True or False? Mead believed that very young children are not influenced by others.

True. Mead believed that very young children are not influenced by others. He believed that children see themselves as the center of their own world. Think of this as a parallel to Piaget's egocentrism stage.

True or false: Mental disorders affects higher functions of the NS performed by the brain like cognition, consciousness and emotions. They can even shorten a persons life span through many ways including suicidal tendencies.

True. Mental disorders affects higher functions of the NS performed by the brain like cognition, consciousness and emotions. They can even shorten a persons life span through many ways including suicidal tendencies.

True or false: some antipsychotic drugs for schizophrenia aim to target elevated levels of dopamine neurotransmitters.

True. Schizophrenia can be accompanied with elevated levels dopamine NTs, and some antipsychotic drugs aim to target these increased levels. *Dopamine hypothesis of schizophrenia* - model that attributes symptoms of schizophrenia to disturbed and hyperactive dopaminergic signal - however, does not posit overabundance of dopamine as a complete explanation for schizophrenia

True or False? An Antithesis may come from the Bourgeoisie.

True. The Antithesis can come from any source. What makes it an Antithesis is that it opposed the accepted status quo (the Thesis).

True or False? the Schachter-Singer theory of emotion claims that you must identify the reason behind the physiological response and/or event before registering an emotion.

True. The Schachter-Singer claims you must identify the reason behind the physiological response and/or event before registering an emotion.

True or false: Mead believed that the understanding of the generalized other in the game stage lead to the formation/understanding of the concepts of The I and The Me

True. The concept of the generalized other leads to the concepts of The I and The Me, according to Mead. The Me represents out social self, how the generalized other sees us, learned through interactions with society; "society's view". The I is our response to the social self; "our own view". Mead would say who we are is a balance of the I and the Me

True or false: Rods and cones have structural similarities

True. They both have structural similarities despite just being shaped different. Rods and cones both have membrane-bound optic discs with a form of opsin (internal protein) - in rods, the protein is rhodopsin while in cones, it is photopsin.

True or false: representativeness heuristic can lead to base rate fallacy (also called conjunction fallacy)

True. Using representativeness heuristic can lead to using proto/stereotypical factors while ignoring actual numerical information, which is base rate/conjunction fallacy. E.g. Linda is a smart, studious college graduate and works in anti-discrimination volunteer programs. You would assume she's more likely a feminist bank teller over a bank teller, but the numbers show she is more likely to be a bank teller.

True or False? When experiencing pleasure, the Ventral Tegmental Area sends dopamine to the Amygdala, Nucleus Accumbens, Prefrontal Cortex, and the Hippocampus.

True. When experiencing pleasure, the Ventral Tegmental Area sends dopamine to the Amygdala, Nucleus Accumbens, Prefrontal Cortex, and the Hippocampus. (MESOLIMBIC REWARD SYSTEM)

True or false: At the same time, when we consider our own behaviour, we skew it in the other direction and are more likely to base them on external factors.

True: At the same time, when we consider our own behaviour, we skew it in the other direction and are more likely to base them on external factors. This is known as *Actor-observer bias*

True or false: Kohlberg's theory believes that each new level or stage replaces the thinking of the previous.

True: Kohlberg's theory believes that each new level or stage replaces the thinking of the previous.

True or false: Self-serving bias is more commonly seen in Individualistic N.American and European societies as opposed to Collectivist societies such as Asia

True: Self-serving bias is more commonly seen in Individualistic N.American and European societies as opposed to Collectivist societies such as Asia

True or false: culture is adaptable, builds on itself, and transmitted.

True: culture is adaptable, builds on itself, and transmitted. The beliefs held by an individual are typically based on learned behaviour, expectations and pressure from the group one is in.

Which reinforcement schedule is the best at causing the repetition of behavior?

Variable-ratio is best at causing repetition of behavior, which is why gambling is so addictive.

As sound travels through an ear, how many levels of amplification do we see? Why is this amplification of sound even necessary?

We see three levels of amplification - the first by the pinna capturing sound, the second by the ear drum, and the third by the 3 bones in the middle ear. The reason we need amplification is because in the inner ear sound is transmitted through a fluid, perilymph - sound travels through more resistance in fluid than in air.

Why do some users who take drugs in new locations overdose?

When taking drugs in a new location, the brain does not take external or ritualistic cues and hence has no time to adjust preemptively - thus, the same higher dose that would be needed in a drug users normal location would be too much when the body has not adjusted beforehand, leading to OD.

Sarah wins the Boston Marathon. If she has an External Locus of Control, what might she tell herself about winning the race?

With an External Locus of Control, Sarah might say, "I won because everyone else was having a bad day."

There are no axons that run behind the cones in the fovea, creating an indentation. What is the functional purpose of this lack of axons?

With no nearby axons blocking the light, a higher resolution of light will hit the cones, giving more clarity of vision.

If a Split Brain patient looks at an object on their right visual field, could they verbally describe the object? Why or why not?

Yes, if a Split Brain patient sees an object in their right visual field, it will be processed on the left side of their brain, where the language center is found. If the object were on the left side, however, the patient could not verbally express the object!

Crystal spent the last calendar year noticeably tense and it was affecting her schoolwork. She often had twitching eyelids and a furrowed brow. Could she have been suffering from General Anxiety disorder (GAD)?

Yes. Since Crystal has had a deficit in her work and her symptoms for over six months from an unclear source, this could be diagnosed as General Anxiety Disorder.

You shine a light in a dark room and cockroaches scurry in all directions. What is this an example of?

You shine a light in a dark room and cockroaches scurry in all directions. This is an example of Kinesis.

You are studying the economy of the United States and how it is influenced by job security. Should you approach this research project using a Macrosociology or Microsociology perspective? Why?

You should approach this project with a Macrosociology perspective as it focuses on large groups and society as a whole.

You want to observe how discrimination may play out during job interviews, so you ask your friend if you can shadow them in their job as a manager. Should you approach this research project using a Macrosociology or Microsociology perspective? Why?

You should utilize a Microsociology perspective because Microsociology looks at small scale everyday interactions between individuals and small groups.

Describe how Reciprocal Determinism may have played a role in your decision to become a doctor?

You thought that being a doctor is cool, so you started going to pre-med classes, which led you to be in an environment with others who think alike. This led you to start to behave like a pre-med, which in turn caused you to start thinking like one and so on.

How would your perception change if you only had one eye?

You wouldn't be able to use binocular cues such as convergence or retinal disparity. As a result, you would be less able to accurately perceive depth.

You share an apartment with 4 other college students. Describe how Social Loafing might affect the cleanliness of your apartment.

Your roommates will depend on each other to keep the apartment clean. Because everyone is dependent on each other and feels a sense of shared responsibility, everyone will do less than they should, resulting in a more messy apartment than it would have been if you were each responsible for a specific section of the apartment.

Rods and cones are connected by ________ cells, which directly or indirectly transmit signals from the photoreceptors to __________ cells.

bipolar, ganglion

What are ganglion cells?

retinal ganglion cells are types of neurons located in the inner surface of the retina - receive information from either of two intermediate neuron types, bipolar cells or amacrine cells Info is transmitted --> optic nerve --> brain

What is the equation for Weber's Law?

ΔI/I=K I= Background Intensity K= Weber's Fraction ΔI = incremental threshold By Weber's Law, relationship between intensity and incremental threshold is linear. It quantifies JND for tactile and auditory sensations.

You are at a rock concert and the music is playing at an intensity of 70 Watts. The rock band members turn up the volume just enough that you notice a difference in sound level intensity. If Weber's Fraction is equal to .1 in this case, what is the JND?

ΔI/I=K ΔI = (0.1)*(70W) ΔI = 7W

The fact that a human's eyes are about 2.5" apart allow humans to get slightly different views of objects from each eye and thus results in a perception of depth. Which binocular cue doe this describe? (A) Interposition (B) Convergence (C) Constancy (D) Retinal Disparity

(D) Retinal disparity Retinal disparity allows us to get two different images of something from the 2 eyes, allowing better depth perception. The eyes are ~2.5" apart.

What are the laws of the Gestalt Principles? What is the purpose of the Gestalt Principles?

*Gestalt principles* are a set of principles that account for how humans naturally and subconsciously perceive as the most simple, organized patterns and objects possible. The laws are: similarity, pragnanz, proximity, continuity and closure. Summarized: Law of similarity - similar objects are grouped together. Law of Pragnanz - reality is reduced to its simplest form (5 circles in olympic logo) Law of Proximity - closer objects grouped/viewed together Law of Continuity - lines seen as following the smoothest path Law of Closure - we tend to fill gaps and see objects grouped together as whole

How is a sound wave 'caught' and transmitted through the ear?

1) sound waves are *caught by your pinna/auricle* (big part of external ear) and passed along into deeper ear 2) sound *passes through the external auditory meatus* (auditory canal), connecting into the middle ear --> *auditory waves collide with the tympanic membrane* (aka eardrum), which is the boundary between the two 3) *Ear drum vibrates and passes vibrations to bones in the middle ear (aka tympanic cavity)*, which *amplifies sound waves* when they enter the inner ear *through the auditory ossicles* (3-bone structure of the malleus, incus and stapes - aka hammer anvil and stirrup) 4) The 3 bone structure acts as a chain to *conduct sound through* another membrane, *the superior oval window, which sets fluid in the inner ear into motion, through which sound travels* 5) *inner ear converts sound into electrical impulses* to send to the brain - more on that later.

What is the just-noticeable difference (JND)?

JND is the threshold where you go from not noticing any tiny changes to just noticing a tiny little change in stimulus. Example with lifting weights, 2.2lbs vs. 2.05 (2.2 is just noticeable an increase from 2lbs whereas 2.05 would prob not be).

What is the relative density and location of the rod and cone cells in a human eye?

The cones are very densely populated and centered in the fovea, while scarcely present in the periphery of the retina. The rods are densely populated in the periphery of the retina, but decline rapidly when approaching the fovea. Remember, the macula is AVASCULAR

There are three ways for a neuron to encode timing (when a stimulus starts and stops? Describe each: - Non-adaptive - Slow-adapting - Fast-adapting

Non-adaptive: neurons fire at a constant rate over time Slow-adapting: neuron firing will slowly adapt to stimulus with time - action potentials start incoming fast but become less frequent until finally stopped Fast-adapting: neuron firing will be rapid initially and then stop, and fire again once the stimulus stops

You probably noticed that when you spun around too much like an idiot when a child, you got pretty dizzy after. What goes on in the vestibular system when that happens?

The endolymph in the ear canals doesn't stop moving right when we do when we're spinning around vigorously and then stop. As a result, the shifting endolymph continues to send signals and action potentials to your brain that you are still moving, when when at halt. This results in dizziness. When the fluid eventually stops moving is when the dizziness stops.

You look at a group of lightbulbs and notice that some are brighter than others. You are unable to tell however that two of the lightbulbs have any difference in intensity, seeming to be the same brightness. How does the concept of Just-noticeable Difference (JND) apply in this context?

When you can see a difference in the brightness of some bulbs, the level of brightness for said bulbs in comparison to the baseline has exceeded your just-noticeable difference (JND). When you do not perceive others as brighter, however, the difference in intensity has not passed your JND.


Conjuntos de estudio relacionados

Women's Health and Maternity/Newborn Drugs Level 1

View Set

Anatomy and Physiology II Test 1: Respiratory System

View Set

Marketing Ch 10 Warm-up (Exam 2)

View Set

Chapter 47: Assessment of Endocrine System ANS

View Set

Social Media: Our Connected World Unit 5 Social Media and Interpersonal Communication

View Set

Chapter 6: Auxiliary Verb Phrases

View Set